くだらねぇ問題はここへ書け ver3.14(67桁略)4062

このエントリーをはてなブックマークに追加
1132人目の素数さん
いちいちスレッド建てないで,ここに書いてね.

最重要な数学記号の書き方の例(これを読まないと放置される可能性大)
---------------------------------------------------------------

   ※分数は、分母分子がわかるように括弧を沢山使ってください。
    1+a/bでは1+(a/b),(1+a)/bの2通りの解釈ができます。
    その他解釈の仕方が幾通りもある例がたっぷりあるので気をつけてください。

    これを無視すると放置される可能性が大です。

--------------------------------------------

●足し算 a+b ●引き算 a-b ●掛け算 a*b, ab ●割り算・分数 a/b, a/(b+c), a/(b*c)
※“*”は掛け算の記号です。×(かける)はXx(エックス)と混同してしまうので使わないのが無難です。
※割り算は“÷”を使わず分数の形で表わすのが一般的です。
※分数は、分母分子がわかるように括弧を沢山使ってください。1+a/bでは1+(a/b),(1+a)/bの2通りの解釈ができます。
●指数 a^b, x^(n+1)
●ルート √(a+b), (a+b)^(1/2)
※指数は“^”を使います。「xのn+1乗」は“x^(n+1)”ときちんと括弧でくくりましょう。
※√は“るーと”を変換して下さい。
※さらに詳しい書き方、過去スレはhttp://mathmathmath.dotera.net/にあります

前スレと関連スレは>>2
2132人目の素数さん:2011/10/20(木) 22:34:06.09
【前スレと関連スレ】
くだらねぇ問題はここへ書け ver3.14(66桁略)6406
http://kamome.2ch.net/test/read.cgi/math/1304528303/
雑談はここに書け!【43】
http://kamome.2ch.net/test/read.cgi/math/1317469275/
分からない問題はここに書いてね361
http://kamome.2ch.net/test/read.cgi/math/1318976057/
よくある質問
http://www.geocities.co.jp/CollegeLife-Club/7442/math/index.html
3132人目の素数さん:2011/10/25(火) 03:01:35.02
高校の時に数Bの試験で本気で0点取ったくらい数学が苦手なんですが、
数学の得意な方に質問です。
平均5分間の曲が7000曲入っているiTunesで、
4時間の間に同じ曲がかかる確率ってどのくらいなのでしょうか?
今日そんなことがあって一生懸命考えたんですけれども全く分かりませんでした。
宜しくお願い致します。
4132人目の素数さん:2011/10/25(火) 12:26:57.03
>>3
答だけ書くと15%ぐらいの確率。
5132人目の素数さん:2011/10/25(火) 15:39:48.48
>>4
そんなひどいウソをついてやるなよ。
240分だから、48曲しか再生できないんだぜ?
どうして7000曲のなかの同じものが15%(!)もの高確率で再生されるんだよ!
64ではないが:2011/10/25(火) 15:59:54.39
1 - Π[k=1,48](7000-k+1)/7000 ≒ 15%
7132人目の素数さん:2011/10/25(火) 16:23:17.08
【基本問題】
あるところに、3人の死刑囚がいた
そこへ看守がやってきて、それぞれの囚人の額にスタンプを押して言った

「お前らの額に一つずつ、1から5までの違う数字のスタンプを押した。
 今、お前らには、自分の数字以外のお互いの数字を見えているわけだが・・・
 そこで、自分の額に書いてある数字が、お前ら3人の中で何番目に大きいかを当ててみろ。
 見事に当てたら、釈放してやる。
 ・・・ただし、誰か一言でも発してみろ、即座に3人とも死刑執行だ。」

今、自分の目から見えるのは、二人の同僚の額に書いてある数字「2」と「4」だ・・・

そのまましばらく、誰も動きが無い沈黙の後俺は看守に歩み寄り、
自分がこの3人の中で何番目に大きいかと、数字そのものを言い当て、自由を手にした。
するとその様子を見た仲間も即座に俺に続いてズバリ当て、無事解放されたようだ

さて、自分の数字とその根拠は?
8132人目の素数さん:2011/10/25(火) 17:16:15.67
この手の問題最初に出した人ってどうやってひねり出したんだろうなあ
9132人目の素数さん:2011/10/25(火) 18:13:32.56
>>7
全員違う数字
 →自分の数字は1,3,5のどれか
数字が2の奴が即座に答えない
 →自分の数字が5ではない
数字が4の奴が即座に答えない
 →自分の数字が1ではない

よって自分の数字は3
10132人目の素数さん:2011/10/25(火) 18:33:11.96
【応用問題】
特に書いて無いが、>>7の基本問題の根底にある条件は
・3人ともに同じレベルの思考回路と判断力を持つ
・3人全員で助かりたいと思っている

基本問題において、「4」を付けられたのがちょっと頭の弱い囚人で、「2」と「3」を見て瞬間的に
(残りが1と4と5なら、俺は半分以上の確率で4か5だろ・・・よし、たぶん1番大きい!)
と勝手に判断し、看守に歩み寄っていく姿を見てしまったら、その時点で残り二人はほぼジエンド

このような、他者を蹴落とす意思や確率に頼った運試しとは別に、この問題は、1〜5の数字、どんな組み合わせでも確実に3人とも助かる方法がある

基本問題>>7におけるこの囚人達のとるべき正解は、問題に書いてあるそのもので
・誰も動かないのを確認してから、「3」の人が動く

Q:それでは、下の組み合わせの場合、3人ともに助かるためには何番がどのような行動をとったらいいのだろう?
A. 1、2、3
B. 1、3、5
C. 2、3、5

※ 他人の答えた回答は、看守以外には聞こえない

Aの場合
まず、3は即決しなければならない
そして、その3が即決した様子を見たら、1も即決して出て行かなければならない
(目の前にいるのが2と3で、3が即決できる状況というのは自分が1の場合しか無いから)
その様子を見ることによって2は、自分が2であると分か
1110:2011/10/25(火) 18:34:57.21
あー
・・・答えまで書いてる

超ドミス
12132人目の素数さん:2011/10/25(火) 19:00:00.30

まず、3が即決する
そして3が即決した段階で、残り二人ともじっと動かないで相手の動きを待つ
 →1にとって、目の前にいるのが3と5の場合、3が即決できる状況は自分が1か4の場合
 →5にとって、目の前にいるのが1と3の場合、3が即決できる状況は自分が2か5の場合
という状態だから、つまり二人ともにしばらくじっとしてることをお互い確認できたら、
自分の数字がそれぞれ1と5だと分かる
13132人目の素数さん:2011/10/25(火) 20:24:55.72
>>10
Cが分かった、俺、スゴい!!
自分で自分をほめたくなった
14132人目の素数さん:2011/10/25(火) 22:31:59.15
最近「架空の大学を作る」っていうスレ見てるんだけど、そこで有志が作った過去問が全然解けん。難しい…

飯館村立工科大学(文科)
数学 過去問
http://ux.getuploader.com/MATTARI_Uploader/download/130/%5Bdrip%5Dtest2.pdf
15132人目の素数さん:2011/10/26(水) 04:40:17.84
>>14
(1) b, d はゼロとして一般性を失わない。あとは(0,0) (0,10) 以外の 2交点を求めて
 面積計算だが、平行四辺形の面積と行列式の関係を知っていると少しは楽か。
(2)中心から 100角形の隣りあう頂点に張る角度は 3.6度だから、けっきょく
 100((1/2)(sin(3.6)/(cos(1.8))^2) - (1/2)sin(3.6)) の値を求めればいいわけで…。
(3)第n群は n個の要素をもち、その要素の最初のものを xn, 最後のものを ynとすれば、
 x1 = y1 = 1, yn = xn + 2(n-1), x(n+1) = yn + 1 が成立するので…。
(4)正20角形についての考察なので、正5角形から出発して、半角を 2回。
16132人目の素数さん:2011/10/26(水) 12:11:39.07
第二問と第四問が被ってるのか
17132人目の素数さん:2011/10/28(金) 16:33:20.89
n次元ユークリッド空間について伺います。n次元ユークリッド空間について本を読んでいると、
その説明には少なくとも2つのアプローチがあるようです。1つめは、

1. 実数体Rを係数体とするベクトル空間R^nを考える。
2. 内積を導入する。
3. 内積からノルムを定義する。
4. ノルムから距離を定義する。

というもので、これに関しては特に疑問はありません。しかし、2つめの、

1. 実数のn組をすべて集めてできる集合R^nを考える。
2. R^nの任意の2つの点x=(x_1,…,x_n), y=(y_1,…,y_n)の間の距離を√Σ(x_i-y_i)^2と定義する。

という方法では、2 において√Σ(x_i-y_i)^2を計算する際の演算がどこで定義されているのか
不明です。後者の方法では、他にどのようなことが前提とされているのでしょうか?
18132人目の素数さん:2011/10/28(金) 16:39:04.36
下記の計算,文章って合ってるのでしょうか?
インピーダンスを調べていて分からなくなりました。

---------------------------------
R2 ÷ (R1 + R2) ^ 2   ( ^2 は2乗の意味)
という式の値が、R1 = R2の時に最大になることが証明できればいいわけです。(面倒なのであえて証明はしませんけど。)
適当な数値を代入してみれば判りますが、R1=R2の時が一番数値が大きくなるはずです。
19132人目の素数さん:2011/10/28(金) 18:16:42.17
R1とR2の条件が何も無いんで何とも言えん。
20132人目の素数さん:2011/10/28(金) 22:38:17.79
>>17
実数の全体R^nが完備アルキメデス順序体であること。
21132人目の素数さん:2011/10/28(金) 22:39:02.59
×実数の全体R^nが完備アルキメデス順序体であること。
○実数の全体Rが完備アルキメデス順序体であること。
2217:2011/10/28(金) 22:58:53.71
>>21
ありがとうございます。では、2つめの方法は、おそらく、

1. 実数の全体Rは完備アルキメデス順序体である。
2. n個のRの直積集合R^nを考える。
3. R^nの任意の2つの点x=(x_1,…,x_n), y=(y_1,…,y_n)の間の距離を√Σ(x_i-y_i)^2と定義する。

という話になっているんですね。ありがとうございました。
23132人目の素数さん:2011/10/29(土) 04:05:13.46
一つ目もな。
24132人目の素数さん:2011/10/30(日) 01:37:20.24
>>14 の内容

飯舘村立工科大学 (前)
「入学試験問題」
  数学(文科)
 (配点100点)
平成23年2月25日 14時-15時40分

注意事項(略)
25132人目の素数さん:2011/10/30(日) 01:38:12.54
>>14 の内容

第1問
 xy平面上において、以下の4つの式の全てに囲まれた領域の面積をa,cを用いて表わせ。
ただし、0<a<c であるとする。
  y = ax + b    … (1)
  y = ax + b + 10 … (2)
  y = cx + d    … (3)
  y = cx + d + 10 … (4)


第2問
 半径rの円に内接する正百角形の面積をS_1, 外接する正百角形の面積をS_2 とする。
このとき、S_2−S_1 の値を求めよ。


第3問
 次の群数列{a_n}について以下の問いに答えよ。

 1|2,4|5,7,9|10,12,14,16|17,19,21,23,25|26,28,30,32,34,…

ここで、第n群に属するものの全ての和をf(n)とする。

(1) f(h)>5000 となる最小のhを求めよ。
(2) a_i = 12345 となるようなiは存在するか。
(3) 第n群の奇数項のみの和をg(n)とする。
  g(n)の一般式をnを用いて表わせ。


第4問
 cos 18゚ の値を求めよ。
26132人目の素数さん:2011/10/30(日) 01:57:47.18
>>14 (4)

 sin(18゚) = s とおく。

 sin(3×18゚) = cos(2×18゚),

 3s - 4s^3 = 1 - 2s^2,

 4s^3 -2s^2 -3s +1 = (s-1)(4s^2 +2s-1) = 0,

 s-1≠0 より 4s^2 +2s -1 = 0,

これを解いて s = (√5 - 1)/4,

 cos(18゚) = √(1-s^2) = …
27132人目の素数さん:2011/10/30(日) 02:26:47.87
>>14
第3問
 第n群は n個の要素をもち、その要素の最初のものを x_n, 最後のものを y_n とすれば、
 x_n = (n-1)^2 + 1, y_n = n^2 が成立する。
 g(n) = n・(x_n+y_n)/2 = n・(n^2 -n+1),  (n:奇数)
    = 0   (n:偶数)
28132人目の素数さん:2011/10/30(日) 03:23:30.68
>>25
平行四辺形の面積を求める問題。
底辺と高さを求めてかければいい。
まずはax+bとax+b+10の距離というか幅というか高さというかそれを求めよう。
傾きはtanθ=aと表せる。
1,a,√(1+a^2)の直角三角形をイメージすると
cosθ=1/√(1+a^2)と求まる
すると、高さが10cosθ=10/√(1+a^2)と求まる
次、底辺の長さを求める。
ax+bとcx+d,ax+bとcx+d+10の交点を求めて距離を計算すればいいだけ
それぞれ、x=(b-d)/(c-a),x=(b-d-10)/(c-a)となる
Δx=10/(c-a),傾きがaなのでΔy=10a/(c-a)
底辺=√{(Δx)^2+(Δy)^2}=10√(1+a^2)/(c-a)
よって面積S=底辺*高さ=100/(c-a)

計算あってるか分からないけど考え方はだいたいこうだろうと思われる
29132人目の素数さん:2011/10/30(日) 10:50:40.34
第一問はもっとうまい方法ないのかな、と勘ぐっちゃうけどこれが最短なのかな。
30132人目の素数さん:2011/10/30(日) 14:06:32.58
原点を y=ax+b と y=cx+dに移動させれば、直線の式は A: y=ax, B: y=ax+10, C: y=cx, D: y=cx+10
Aと Cの交点は P=(10/(a-c), 10a/(a-c)), Q= (10/(c-a), 10c/(c-a)).
P, Q を 2×2の行列に並べて行列式を作れば、100/(c-a)。これ(もしくはその絶対値を
とったもの) が求める面積。
31132人目の素数さん:2011/10/30(日) 14:08:10.95
× 原点を y=ax+b と y=cx+dに移動させれば
○ 原点を y=ax+b と y=cx+dの交点に移動させれば
32132人目の素数さん:2011/10/30(日) 14:16:56.57
問3は無理っぽいのかな
33132人目の素数さん:2011/10/30(日) 14:20:17.74
ミスった、第二問だ
34132人目の素数さん:2011/10/30(日) 15:07:27.03
問2は、外接N角形と内接N角形(ただし円の半径 r=1)について、面積差 S = t^3/(1-t^2)
となる。ここで t = tan(π/N)。N=100について、tを解析的に解くのは無理だろう。
数値解でよければ、大きな Nに対して t ≒ π/N として、S ≒ (π/N)^3 = 0.0000310063.
これは真値 0.0000310676 にけっこう近い。
35132人目の素数さん:2011/10/30(日) 15:18:20.59
× 面積差 S = t^3/(1+t^2)
○ 面積差 S = t^3/(1+t^2)
この真値は t = tan(π/100) について、 S = 0.000031006278694459780722
だが、 (π/100)^3で近似したものは S≒ 0.000031006276680299820175
で小数点以下 10桁くらい合う。
36132人目の素数さん:2011/10/30(日) 15:22:05.34
なるほど。参考になります、ありがとう!

第二問は改訂を勧めたほうが良さそうだね。
37132人目の素数さん:2011/10/30(日) 16:23:34.16


nを自然数の定数とする。二元不定方程式
x2+y2=n
の整数解の個数を示せ
38132人目の素数さん:2011/10/30(日) 18:09:22.33
>>37
可算無限個
39132人目の素数さん:2011/10/30(日) 21:38:22.45
>>37

n=1 のとき 4個 (±1,0) と (0,±1)

nが素数pのとき
 p=2 のとき 4個 (±1,±1)
 p=4m+1 のとき 4個
 p=4m+3 のとき 0個

nが合成数のとき、nを素因数分解する。
 p≡3 (mod 4) のベキがすべて0または偶数のとき 4個以上
 p≡3 (mod 4) のベキで奇数のものがあるとき 0個
40132人目の素数さん:2011/10/30(日) 23:13:07.42
>>4 >>6

N=7000 として
 Π[k=1,48] (N-k+1)/N = 0.8508592238124

(近似計算)
 Π[k=1,48] (N-k+1)/N
 = exp{ Σ[k=1,48] log((N-k+1)/N) }
 ≒ exp{ N∫[0, 47.5/N] log(1-x)dx }
 = exp{ N・[ (1-x)log(1-x) + x ] }
 = exp{ N・(1-47.5/N)log(1-47.5/N)) + 47.5 }
 = exp( -0.161526486 )
 = 0.850844
41132人目の素数さん:2011/10/31(月) 01:19:52.16
>>40

(近似計算)その2
 Π[k=1,48] (N-k+1)/N = {N!/(N-48)!}(1/N)^48
  ≒ N^(N+0.5)/{(N-48)^(N-47.5)・N^48} e^(-48) (スターリングの公式)
  = N^(N-47.5)/(N-48)^(N-47.5) e^(-48)
  = {N/(N-48)}^(N-47.5) e^(-48)
  = exp(-0.1615085063)
  = 0.850859294
42自問自答:2011/10/31(月) 18:47:07.72
(問)7000個の区画にコインをランダムに 48個投げる。各区画に、コインが 0個もしくは
 1個である確率を求めよ。
(解)ポアソン分布による。p = 48/7000として、1区画あたりそうなっている確率は
 (1+p)exp(-p) ( = 0.9999766)。全区画がそうなる確率は、それを 7000乗して、
 ((1+p)exp(-p))^7000 = exp(7000(ln(1+p)-p)) = 0.848892.
43自業自得:2011/10/31(月) 22:26:44.60
(問)7000個の区画にコインをランダムに48個投げる。各区画に、コインが0個もしくは
 1個である確率を求めよ。

(解)1つの区画がそうなっている確率は
 1-p' = (1-1/N)^48 + 48・(1/N)(1-1/N)^47
  = 0.0000229198,
1つの区画と他の区画の間には相関があるが、1-p'<<1 なので無視すると、
 (p')^N ≒ 0.851768557
44132人目の素数さん:2011/11/01(火) 02:52:59.97
【自業自得】
〔仏教で〕すべての不結果は、以前に自分が行った良くない行為の報いに基づく
とする考え方。(新明解国語辞典 第五版 三省堂)
45132人目の素数さん:2011/11/01(火) 02:56:02.56
>>43

「相関がある」 = 「独立でない」
ですね。
46132人目の素数さん:2011/11/03(木) 16:18:34.32
24人でボーリングの総当たり戦をします。
1ゲーム4人で回すとき何ゲーム必要ですか?
47132人目の素数さん:2011/11/03(木) 17:10:44.11
>>46
マルチ
48132人目の素数さん:2011/11/03(木) 17:43:18.28
>>46

総当り方式対戦組み合わせソフト 1.28 (09.11.10公開 595K)
http://www.vector.co.jp/soft/winnt/home/se479098.html

BOWLING.Assist ダブルスリーグ用 2.00 (08.10.27公開 797K)
http://www.vector.co.jp/soft/winnt/home/se465017.html
4946:2011/11/03(木) 18:07:50.64
>>48
ありがとうございます。
使ってみます。
50132人目の素数さん:2011/11/05(土) 11:31:46.45
下記の答え 教えてください。

ろくちゃんのコップには、濃度20%のトマトジュース100ccが入って
います。
こうちゃんのコップには、濃度65%のトマトジュース100ccが入って
います。
ろくちゃんのコップから50ccを取ってこうちゃんのコップに入れ、
よくかき混ぜます。
その後に、こうちゃんのコップから50ccを取ってろくちゃんのコップに
戻します。
さらにろくちゃんのコップををよくかき混ぜた後に、ふたたび50ccを
取ってこうちゃんのコップに入れ、
同じようにしてもう一度50ccを取ってろくちゃんのコップに戻します。
ろくちゃんのコップに入っているトマトジュースの濃度は何%に
なったでしょうか?

51132人目の素数さん:2011/11/05(土) 13:31:45.89
算数の問題ですね。

最初に、ろくちゃんのコップから 50cc を取ってこうちゃんのコップに入れると、
原液は 50cc×20%+100cc×65%=75cc あり、
ジュースは 50cc+100cc=150cc あるから、
濃度は 75cc÷150cc=50% になります。

次に、こうちゃんのコップから 50cc を取ってろくちゃんのコップに戻すと、
原液は 50cc×50%+100cc×20%=45cc あり、
ジュースは 50cc+100cc=150cc あるから、
濃度は 45cc÷150cc=30% になります。

さらにに、ろくちゃんのコップから 50cc を取ってこうちゃんのコップに入れると、
原液は 50cc×30%+100cc×50%=65cc あり、
ジュースは 50cc+100cc=150cc あるから、
濃度は 65cc÷150cc=13/30 になります。

最後に、こうちゃんのコップから 50cc を取ってろくちゃんのコップに戻すと、
原液は 50cc×13/30+100cc×30%=51と2/3cc あり、
ジュースは 50cc+100cc=150cc あるから、
濃度は (51と2/3cc)÷150cc=34と4/9% になります。
52132人目の素数さん:2011/11/05(土) 17:37:44.62
最初、ろくちゃんのトマトジュースには20ccの果汁が含まれ、
こうちゃんのトマトジュースには65ccの果汁が含まれている。
まず、ろくちゃんのトマトジュースの50ccには10ccの果汁が
含まれているので、こうちゃんのコップにトマトジュースを移すと、
ろくちゃんのコップにはトマトジュースが50cc(果汁は10cc)、
こうちゃんのコップにはトマトジュースは150cc(果汁は75cc)
入っていることになる。
次にこうちゃんのコップから50cc取り出したとき、果汁は25cc含まれて
いるので、ろくちゃんのコップにトマトジュースを移すと、
こうちゃんのコップにはトマトジュースが100cc(果汁は50cc)、
ろくちゃんのコップにはトマトジュースが100cc(果汁は35cc)入っている
ことになる。
さらにろくちゃんのコップから50cc取り出したとき、果汁が17.5ccの
果汁が含まれているので、こうちゃんのコップにトマトジュースを移すと、
ろくちゃんのコップにはトマトジュースが50cc(果汁は17.5cc)、
こうちゃんのコップにはトマトジュースは150cc(果汁は67.5cc)
入っていることになる。
最後にこうちゃんのコップから50cc取り出したとき、果汁は22.5cc含まれ
ているので、ろくちゃんのコップにトマトジュースを移すと、
こうちゃんのコップにはトマトジュースが100cc(果汁は45cc)、
ろくちゃんのコップにはトマトジュースが100cc(果汁は40cc)
入っていることになる。
これより、ろくちゃんのコップに入ってるトマトジュースの濃度は40%。

53132人目の素数さん:2011/11/05(土) 18:30:09.54
白球3つ、赤球3つ、青球3つを
どの色の球も隣り合わないように並べる方法は何通り?
5453:2011/11/05(土) 18:44:02.69
自己解決しました
取り消します
55132人目の素数さん:2011/11/06(日) 07:22:12.14
「函数:y−β=f(x−α) のグラフは、函数:y=f(x) のグラフを
x軸方向にα、y軸方向にβだけ平行移動したものである」ことを
証明せよ。

56132人目の素数さん:2011/11/06(日) 10:09:29.45
>55
「証明」と大上段に構えたからには、
その函数とやらの定義(おまえの書いたのは単なる等式)と平行移動の定義を明示すべき
57132人目の素数さん:2011/11/06(日) 12:49:52.74
>>55はマルチだよ
58132人目の素数さん:2011/11/07(月) 08:17:43.11
>>51様分 >>52様分 

>>50ですが、どちらが正解なのですか?

59132人目の素数さん:2011/11/07(月) 08:55:44.24
自分で計算すれば分かるだろうが

40%
60132人目の素数さん:2011/11/07(月) 11:39:34.51
>>59

ありがとうございます。
61132人目の素数さん:2011/11/07(月) 15:11:36.10
区別のつかない三個の袋には、RRR、RWそしてWWの玉がそれぞれ
入っている。ただし、Rは赤玉、Wは白玉を意味する。
今、一個の袋を選んでその中から玉を一個取り出したところ、
Rであった。残りの玉がRである確率は?

RR、RW、WWなら2/3なんだが・・・。
RRR、RW、WWならどうなるの?
解法もお願いします。
62132人目の素数さん:2011/11/07(月) 15:26:38.49
まさか
ttp://kamome.2ch.net/test/read.cgi/math/1318976057/706
と同じやつではあるまいな?
63132人目の素数さん:2011/11/07(月) 15:29:41.55
おなじです!
64132人目の素数さん:2011/11/07(月) 15:37:04.38
>>63
反省しろ
65132人目の素数さん:2011/11/07(月) 15:44:36.30
しましたので、教えてください。
お願いします。
66132人目の素数さん:2011/11/07(月) 16:22:02.55
http://yuzuru.2ch.net/test/read.cgi/kouri/1318573136/363
ここにもいるよ、こいつ。
67132人目の素数さん:2011/11/07(月) 16:40:49.36
困っているのです。

詳しく教えてください。
当方文系大卒の老年です。
宜しく。
68132人目の素数さん:2011/11/07(月) 16:54:27.67
ふざけんな、くず
69132人目の素数さん:2011/11/07(月) 16:55:48.20
困ってたら人の道を外れたことをしてもいいと考えるのが老年なのか。
本当かどうかわからんけど。
70132人目の素数さん:2011/11/07(月) 17:03:40.24
ここのヒトは冷たいのですか・・・
71132人目の素数さん:2011/11/07(月) 17:10:47.33
自らを棚に上げて他人を叱責か。
72132人目の素数さん:2011/11/07(月) 17:13:09.21
>>70
他の質問者に対する回答を見てもそう思うかい?
あんただけが叩かれているなら、あんたに叩かれるだけの理由があると思わないのか?
73132人目の素数さん:2011/11/07(月) 17:17:34.03
思わん。
何が悪いのか!!
74132人目の素数さん:2011/11/07(月) 17:21:47.49
何が悪いかって、複数の場所で同じことを質問するのが悪い
マルチポストと言われていて、だいたいの場合まともなレスが付かない
75132人目の素数さん:2011/11/07(月) 17:27:17.12
>>73
じゃあ、>>65では何を反省したんだ?
なりすましだろうけど。
76KingMathematician ◆5lHaaEvFNc :2011/11/07(月) 17:30:15.13
質問です.
集合A,Bに対して二つに共通の要素が存在しないことと同値なことを A∩B=Æ で表せますが,
空集合の表示もせず, 二つの引数を持つ述語記号を用いて表現するにはどうすればいいですか.
77KingMathematician ◆5lHaaEvFNc :2011/11/07(月) 17:33:28.39
空集合の記号のつもりで Æ を書いたが,私がよく知らない文字が出現した.
78132人目の素数さん:2011/11/07(月) 17:38:19.13
>>初めてなので知らなかった・・・。
悪かった。
そんな規則、ルールがあるとは思わなかった。
見てくれるヒトが多いほうがよいと思ったから・・・
79132人目の素数さん:2011/11/07(月) 17:57:10.61
>>78
見てくれる人が多い方がよいのはあなたにとってだけだよね?
回答する側から見たら、よそで解決してたらアホみたいなことをさせられることになるわけだよ。
聞くだけ聞いて放ったらかしにされるかもしれない。
現に回答があるのに放ってあるよね、あんた。
80132人目の素数さん:2011/11/07(月) 18:00:54.31
規則ってより常識だわな。
質問されたから一生懸命考えてたら、そいつはよそへ質問しに行ってた。
普通、不愉快になるよな。ネットに限らず。
どんだけ失礼なやつだよって話だ。
81132人目の素数さん:2011/11/07(月) 21:00:23.91
>>76
∀x(x∈A⇒x∉B)∧∀y(y∈B⇒x∉A)
でいいんでは

Ø は、&Oslash; で出ます
82132人目の素数さん:2011/11/07(月) 22:52:43.53
∅はオースラッシュとは違う字だとおもうんですが…
83132人目の素数さん:2011/11/07(月) 23:22:51.26
512^122 mod 231って1ですか?
オイラー関数よりφ(231)=120で
512と231が素数より512^120≡1(mod 231)として求めたんですが...
84132人目の素数さん:2011/11/07(月) 23:30:41.21
>>83
ちゃう512^122 mod 231は190
85132人目の素数さん:2011/11/07(月) 23:33:15.14
>>83
誤字脱字勘違いのオンパレードにしか見えないんだが
86132人目の素数さん:2011/11/07(月) 23:34:27.45
うるせえ!
87β:2011/11/08(火) 00:26:51.55
ナントカ病院逝って頭にぶっと〜いお注射撃ってもらえ
88KingMathematician ◆5lHaaEvFNc :2011/11/08(火) 01:47:10.97
Re:>>81 ∀x(x∉A∨x∉B).
89132人目の素数さん:2011/11/09(水) 15:23:17.49
2C1 A、Bの2つから1個取り出す。Aが出る確率は0.5
100C50 A、B、C・・・の100個から50個取り出す。Aが出る確率は0.5

例えばそれぞれ10回繰り返したときに、Aが10回続けて出ない確率はどちらも同じですか?
90132人目の素数さん:2011/11/09(水) 15:39:37.40
>>89
2C1とか100C50が何のために書かれているのかわからんけど、同じだよ。
ところでA、B、C・・・の100個ってのはちょびっと違和感がある。
91132人目の素数さん:2011/11/10(木) 06:37:33.86
「仕入れ値の3割増しの定価がつけてある商品を
40円引きして売ったところ、200円の利益がありました。
仕入れ値は何円ですか。」

という問題で本の説明では以下のような式になるのですが

(1.3x-40)-x=200

なぜ-xになるのか解りません。教えて下さい。
92132人目の素数さん:2011/11/10(木) 08:21:33.66
>>91
売値-仕入れ値=利益だから。
9391:2011/11/10(木) 12:34:29.70
>>92
有難うございます。
94132人目の素数さん:2011/11/10(木) 19:03:58.19
古びたお守り1個鑑定1/5400、古びたお守り2個以上鑑定1/2700の護石が出るまでの平均回数教えておくれ

以下データ
エリア3・5・8 採掘合計回数18回 古びたお守り*1:5%
エリア7 採掘回数6回 古びたお守り*1:3%
エリア9 採掘合計回数13回 古びたお守り*2:5%
エリア10 採掘回数6回 古びたお守り*1:7%
95132人目の素数さん:2011/11/11(金) 19:29:13.40
5cmより長いって5cm入りますか?
96KingMathematician ◆5lHaaEvFNc :2011/11/11(金) 20:59:32.53
Re:>>95 5cmより長いとは長さをcm単位で示したときの数量が5より大きいことと思われる.5は5より大きくないので5cmは入らない.
97132人目の素数さん:2011/11/11(金) 22:12:07.65
まずは日本語から練習してくれ
何を言っているのかさっぱりわからん
98132人目の素数さん:2011/11/15(火) 01:37:00.51
情報量についてH(X)とH(X|Y)の関係性を教えてください
99132人目の素数さん:2011/11/16(水) 03:58:23.30
毎時って速さのことですか?
100132人目の素数さん:2011/11/16(水) 04:00:50.90
分子が何かによる
101132人目の素数さん:2011/11/16(水) 06:41:49.09
102132人目の素数さん:2011/11/16(水) 19:00:26.36
103132人目の素数さん:2011/11/16(水) 20:04:18.66
{1,1,-1,1,1,-1,1,1,-1,...}
と、1,1,-1 を繰り返す数列がある。
この数列を生成する漸化式と、この数列の一般項を見つけよ。
104132人目の素数さん:2011/11/16(水) 20:13:09.63
>>103
a_1=a_2=a_3=0, a_{n+3}=a_n
a_n=(4/3)sin((2n+π)/6)+1/3
105132人目の素数さん:2011/11/16(水) 20:14:52.60
あ、間違えた
a_n=(4/3)sin((2n-1)π/6)+1/3
106132人目の素数さん:2011/11/16(水) 20:16:34.36
まだ間違ってる。今日は駄目駄目だわ
a_n=(4/3)sin((4n-3)π/6)+1/3

107132人目の素数さん:2011/11/16(水) 20:25:22.39
>>104-106
まぁ、もちつけ
周期関数を>>104みたいな感じで表したら全く面白くないから、a_(n+2)をa_(n+1)、a_nで表せ位に変更汁
108132人目の素数さん:2011/11/16(水) 21:32:35.90
>>103 これでどうか
a[n+2]=pa[n+1]+qa[n] (n=1,2,...) (q≠0)
n=1,2を代入
-1=p+q, 1=-p+q
p=-1, q=0 ... NG

a[n+3]=pa[n+2]+qa[n+1]+ra[n] (n=1,2,...) (r≠0)
n=1,2,3を代入
1=-p+q+r, 1=p-q+r, -1=p+q-r
--> 1=p+q+r
p=0, q=0, r=1
a[n+3]=a[n] (n=1,2,...), a[1]=a[2]=1, a[3]=-1
a[n+3]+sa[n+2]+ta[n+1]=u(a[n+2]+sa[n+1]+ta[n])
u-s=0, us-t=0, ut=1
u=s, t=s^2, s^3=1
s=1を選択
a[n+3]+a[n+2]+a[n+1]=a[n+2]+a[n+1]+a[n]=-1+1+1=1
a[n+2]+va[n+1]+w(a[n+1]+va[n])=1
v+w=1, v*w=1
v=-ω, w=-ω^2
a[n+2]-ωa[n+1]-ω^2(a[n+1]-ωa[n])=1
b[n]:=a[n+1]-ωa[n]
b[n+1]=ω^2b[n]+1
(b[n+1]-1/(1-ω^2))=ω^2(b[n]-1/(1-ω^2))
b[n]=1/(1-ω^2)+(b[1]-1/(1-ω^2))*(ω^2)^(n-1)
b[1]-1/(1-ω^2)=(1-ω)-1/(1-ω^2)=(1-ω-ω^2+ω^3-1)/(1-ω^2)=2/(1-ω^2)
a[n+1]/ω^(n+1)=a[n]/ω^n+{1/(1-ω^2)}/ω^(n+1)+{2/(1-ω^2)}*ω^(n-3)
a[n]/ω^n=1/ω+Σ[k=1,n-1][{1/(1-ω^2)}/ω^(k+1)+{2/(1-ω^2)}ω^(k-3)]
=...
={1/(3ω^2)}{1/ω^(n-2)-2ω^2-2ω^(n-1)}
a[n]={1/(3ω^2)}{ω^2-2ω^(n+2)-2ω^(2n-1)}
=(1/3){1-2ω^n-2ω^(2n-3)}
109132人目の素数さん:2011/11/16(水) 21:42:57.39
{1+(-1)^n}/2
(1+ω^n+ω^2n)/3
{1+i^n+(-1)^n+(-i)^n}/4

こいつらを使えば周期のある数列はおk

と言いつつ
>>103
a[1]=a[2]=1
a[n]=-a[n-1]*a[n-2] (n≧3)
とかどう?
110132人目の素数さん:2011/11/16(水) 22:02:16.23
連続する三項の積を取ればどこでも-1、だからa[n+2]*a[n+1]*a[n]=-1→a[n+2]=-1/(a[n+1]*a[n])
これに、1/a[n]=a[n]を使えば、a[n+2]=-a[n+1]*a[n]になる
同様に、連続する三項の和を取ればどこでも1、だからa[n+2]+a[n+1]+a[n]=1→a[n+2]=1-a[n+1]-a[n]なんかもok
111132人目の素数さん:2011/11/16(水) 22:49:24.13
>>109

その "対数" をとって
 - a[n] = (-1)^F[n],
とおくと
 F[n] = F[n-1] + F[n-2],
 F[1] = F[2] = 1,
∴ F[n] はフィボナッチ数列。
112132人目の素数さん:2011/11/17(木) 19:24:05.93

ってどういう意味ですか?
113132人目の素数さん:2011/11/17(木) 19:59:48.36
フィボナッチ数列は、1,1,2,3,5,8,13,21,34,...だが、
偶数(e)か奇数(o)かで眺めると、o,o,e,o,o,e,o,o,e,...と
>>103の数列で、1→o、-1→eとしたのと同じになっていると言うこと
114KingMathematician ◆5lHaaEvFNc :2011/11/17(木) 22:46:45.76
Re:>>112 それは monomorphism, 単射の記号か.
115132人目の素数さん:2011/11/18(金) 14:31:12.23
電波テロ装置の戦争(始)
エンジニアと参加願います公安はサリンオウム信者の子供を40歳まで社会から隔離している
オウム信者が地方で現在も潜伏している
それは新興宗教を配下としている公安の仕事だ
発案で盗聴器を開発したら霊魂が寄って呼ぶ来た
<電波憑依>
スピリチャル全否定なら江原三輪氏、高橋佳子大川隆法氏は、幻聴で強制入院矛盾する日本宗教と精神科
<コードレス盗聴>
2004既に国民20%被害250〜700台数中国工作員3〜7000万円2005ソウルコピー2010ソウルイン医者アカギ絡む<盗聴証拠>
今年5月に日本の警視庁防課は被害者SDカード15分を保持した有る国民に出せ!!<創価幹部>
キタオカ1962年東北生は二十代で2人の女性をレイプ殺害して入信した創価本尊はこれだけで潰せる<<<韓国工作員鸛<<<創価公明党 <テロ装置>>東芝部品)>>ヤクザ<宗教<同和<<公安<<魂複<<官憲>日本終Googl検索
116132人目の素数さん:2011/11/18(金) 16:18:31.81
魂は幾何学


誰か(アメリカ)気づいた
ソウルコピー機器


テロ装置の再読願います
117132人目の素数さん:2011/11/20(日) 10:54:03.31
あげ
118132人目の素数さん:2011/11/20(日) 13:08:10.22
区間[-π、π]上の連続関数f(x)、g(x)の内積(f,g)を(f,g)=∫[ーπ→π]f(x)g(x)dxで
定義するとき、1/(√2π)、cosx/(√π)、sinx/(√π)、cos2x/(√π)、sin2x/(√π)、
・・・がこの内積に関し正規直交系をなすことをしめせ。

まったくわかりません。というか問題文の意味がわかりません。
正規直交系をなすってことは内積が0ってことですよね
じゃぁどれとどれの内積をとればいいのですか?
何を計算すればよいのでしょうか?
119132人目の素数さん:2011/11/20(日) 13:10:00.52
問題読め
120132人目の素数さん:2011/11/20(日) 13:13:56.66
読んでわからないから聞いてるんですが
121132人目の素数さん:2011/11/20(日) 13:17:08.73
「正規直交系をなす」の定義を暗唱
122132人目の素数さん:2011/11/20(日) 13:19:21.24
正規直交系の定義に照らして確認するだけじゃん
123132人目の素数さん:2011/11/20(日) 13:19:35.79
(f,g)=0

なにがfでなにがgかが分からないってことです
124132人目の素数さん:2011/11/20(日) 13:20:22.53
>>122

あなたが分からないからナンクセつけてごまかしているんでしょ?
125132人目の素数さん:2011/11/20(日) 13:20:54.46
>>123
自分で質問を書き写していて、fやgとして取上げる関数達にきづいていないの?
126132人目の素数さん:2011/11/20(日) 13:22:58.05
>>124
ワラタ
127132人目の素数さん:2011/11/20(日) 13:23:03.88
いやさすがに
1/(√2π)、cosx/(√π)、sinx/(√π)、cos2x/(√π)、sin2x/(√π)・・・
これらがf、gとは分かります
けど具体的計算としてたとえば
f=1/(√2π)、g=1/(√2π)として片っ端からやっていくのかどうか
具体的な計算の仕方が分かりません
cosnx/(√π) (n=0,1,2,・・・)としてやるんですか?
128132人目の素数さん:2011/11/20(日) 13:23:26.30
>>125

だからー 自分が分からないからと言って、ごまかすのはやめてくださいねw
129132人目の素数さん:2011/11/20(日) 13:23:56.69
>>124=>>126
あからさまないたずら乙
ガキは引っ込んでろ
130132人目の素数さん:2011/11/20(日) 13:24:57.26
誰が見ても>>124=126=128=>>129な件
131132人目の素数さん:2011/11/20(日) 13:28:32.13
結局誰も解けないんですか
口だけのカス共ですね
失笑w
132132人目の素数さん:2011/11/20(日) 13:29:10.28
>>127
f_n(x)=sin(nx)/√2π(n≧1)、g_n(x)=cos(nx)/√2π(n≧0)として
(f_n,f_m)、(f_n,g_m)、(g_n,g_m) を全部計算してみなよ。
分かるから。
133127 ◆.CzKQna1OU :2011/11/20(日) 13:31:47.69
なんかたくさん俺がいるみたいですが127以降全部偽者ですので

>>132
なるほど、それで全部確かめられますね
ありがとうございました。
134132人目の素数さん:2011/11/20(日) 13:32:28.90
まあいいさ
ここはくだらねえ問題を書くスレだ
135132人目の素数さん:2011/11/20(日) 13:36:34.83
>>133
>>132じゃないが、めんどくさがらずにとりあえず
cosx/(√π)、sin2x/(√π)で計算してみるとよかったと思う
136132人目の素数さん:2011/11/20(日) 13:38:02.64
>>132
分母の√2πは書き間違い。適宜修正を。
137132人目の素数さん:2011/11/20(日) 13:44:21.43
フーリエ解析の基礎論ってことじゃないのこれ
138132人目の素数さん:2011/11/20(日) 13:46:45.58
127以降とか言われてもその前までで十分アレなんだが
139132人目の素数さん:2011/11/20(日) 14:05:27.41
定義を確認、という書き込みをナンクセといわれてはなあ
140132人目の素数さん:2011/11/20(日) 14:07:24.63
1/(√2π)、cosx/(√π)、sinx/(√π)、cos2x/(√π)、sin2x/(√π)

1つ目が1/√2πっておかしくね?
141KingMathematician ◆LoZDre77j4i1 :2011/11/20(日) 15:16:51.06
∫_{-π}^{π}(1/√(2π))^2dx=1.
142132人目の素数さん:2011/11/21(月) 20:16:43.68
定数係数って言葉はよく聞きますけど、関数係数って言葉は使われますか?
143132人目の素数さん:2011/11/21(月) 21:19:08.17
多変数多項式関数を考えてみたらいいんじゃないか。

144132人目の素数さん:2011/11/21(月) 21:20:26.77
>>142
あるよ。
145132人目の素数さん:2011/11/22(火) 13:12:35.40
x^2+y^2+z^2=a^2(a>0)から円柱x^2+y^2=axによって切り取られる部分の面積を求めよ
っていう問題が解けません

答えには2(π−2)a^2ってあるんですけどあわないんです
誰か解説お願いします
146132人目の素数さん:2011/11/23(水) 01:04:14.13
>>145
S=2∫_D dS
D:(x-a/2)^2+y^2≦(a/2)^2
zdz=-xdx-ydy
dS=|(dx,0,-xdx/z)×(0,dy,-ydy/z)|
=|(xdxdy/z,ydxdy/z,dxdy)|
=√((x/z)^2+(y/z)^2+1)dxdy
=(a/z)dxdy
x=r*(1+cos(t))
y=r*sin(t)
0≦r≦a/2, 0≦t<2π
(x_r,x_t)=(1+cos(t),-r*sin(t))
(y_r,y_t)=(sin(t),r*cos(t))
dxdy=r(1+cos(t))drdt
a/z=1/√(1-(x/a)^2-(y/a)^2)
=1/√(1-2(r/a)^2(1+cos(t)))
S=2∫[r=0,a/2]∫[t=0,2π]{1/√(1-2(r/a)^2(1+cos(t)))}r(1+cos(t))drdt
=2∫[t=0,2π]{[√(1-2(r/a)^2*2(cos(t/2))^2)]_[r=0,a/2]}/{-(2/a^2)}dt
=(a^2)∫[t=0,2π]{1-√(1-(cos(t/2))^2)}dt
=(a^2)∫[t=0,2π]{1-sin(t/2)}dt
=(a^2)(2π-4)
147KingMathematician ◆LoZDre77j4i1 :2011/11/23(水) 01:14:42.71
Re:>>145 円柱が切り取る部分は円柱の外と円柱の中のふたつがある.曲面をparameter表示して円柱の内側のほうの面積を計算すると2a^2(π-2)になる.
148猫は再起不能 ◆MuKUnGPXAY :2011/11/23(水) 02:40:23.00
>>147
徹底追跡スルさかいナ。



>147 名前:KingMathematician ◆LoZDre77j4i1 :2011/11/23(水) 01:14:42.71
> Re:>>145 円柱が切り取る部分は円柱の外と円柱の中のふたつがある.曲面をparameter表示して円柱の内側のほうの面積を計算すると2a^2(π-2)になる.
>
149132人目の素数さん:2011/11/23(水) 09:31:38.84
哲也しつこい
150猫は再起不能 ◆MuKUnGPXAY :2011/11/23(水) 11:18:13.15
出来る事は全てヤル。ソレが戦いというモノ。


151132人目の素数さん:2011/11/23(水) 13:53:52.78
>量子力学など幾つかの最先端科学分野では、統計学の客観的確率では説明がつかない矛盾が
>生じてしまうと言われ始めており、客観的確率の限界がそこかしこで見られ始めている。

ベイズの理論について調べていたらこのような一文があったのですが
説明がつかない矛盾とは具体的にどのようなことですか?
152132人目の素数さん:2011/11/23(水) 14:03:28.18
>>151
書いた人に聞いて
153132人目の素数さん:2011/11/23(水) 17:59:08.12
数学素人なので言葉づかいがおかしかったらご容赦ください
http://oshiete.goo.ne.jp/qa/2815878.html
について

「一人が女であることがわかっている」ということ以外の条件的事象が

「一人が男であることがわかっている」ということであれば
答えはC

「一人が男であることがわかっている」ということと
「一人も性別がわかっていない」ということであれば
答えはA

「一人が女であることがわかっている」ということ以外の条件的事象は上のどっちですか?
そして、この考え方はあってますか?
154KingMathematician ◆LoZDre77j4i1 :2011/11/23(水) 18:39:46.01
Re:>>150 お前はできることを全てやり自滅するのか.
Re:>>153 少なくとも一人は女であることが判明したか,誰かの性別を調べるとそれが女であることが判明したか.
155132人目の素数さん:2011/11/23(水) 18:45:09.99
>154
ご回答ありがとうございます

私は理系ではあるものの、数学の知識は受験数学で終了しておりますので
素人にわかりやすくご解説いただければ助かります。
156132人目の素数さん:2011/11/23(水) 18:52:03.87
甘えたこと言わず少しは努力しろ
157132人目の素数さん:2011/11/23(水) 19:14:06.28
質問です。
ボジョレー・ヌーボーで「10年に一度の出来」や「50年に一度のなんちゃら」とかありますけど、例えば「10年間の間に『10年に1度の出来』が2回、『50年に一度』が1回訪れる」確率って、どのくらいですか?
知り合いは0.32%って言ってましたが、計算どうやったかは不明です。
158132人目の素数さん:2011/11/23(水) 19:28:52.32
ある区間 0〜X において積分可能で有限値に収束し、
Fmax < f(x) < Fmin であるような関数f(x) があったときに、
ある値 v 以上になる領域の面積を v の関数 F(v)で表せないかなあ、
と思っています。

Fmin < 0 かつ Fmax > 0 としたときに、F(v)はこんなふうになると思います。

v < Fmin のとき
 F(v) = {0 から X まで |f(x)| を積分した値。}
 F(v)の最大値になると思うので、このときの値を Fs とする。
Fmin <= v < 0 のとき
 F(v) = { 0 から X まで、Fs から f(x) の v 以下の部分の面積を引いた値。}
0<=v<=Fmax のとき
 F(v) = { 0 から X まで、f(x) の v 以上の部分の面積の値。}
v > Fmax
 F(v) = 0

F(v) はvに対して単調減少関数になりそうだなー、とか、
これ微分出来るんじゃね?とか思うんですけど……

f(x)に簡単な関数をもってきたときに F(v)を計算する方法、というのは、
どんな本や数学の分野を勉強すれば出来そうですか?
159132人目の素数さん:2011/11/23(水) 19:40:16.77
説明する気があるのはわかるけど説明不足な点が多すぎ
160132人目の素数さん:2011/11/23(水) 19:42:02.88
というか意味不明
161132人目の素数さん:2011/11/23(水) 19:44:47.39
AB=AC=AD=6、BC=CD=DB=6√2
である三角錐に内接する球の半径を求めよという問題なんですが
色々と試したのですがどうやっても答えが出ません
解き方を教えてください
162132人目の素数さん:2011/11/23(水) 19:49:13.59
>>157
ある年のボジョレヌーボーが10年に1度の出来である確率が1/10
50年に1度の出来である確率が1/50、それ以外の確率は1-(1/10)-(1/50)=44/50

10年間のうち2年は10年に1度の出来、1年は50年に一度の出来、残りの7年はそれ以外だから
(10!/2!*7!)(1/10)^2*(1/50)*(44/50)^7≒0.029
だから約3%って感じじゃないかな?
163132人目の素数さん:2011/11/23(水) 20:23:28.19
>>162
10年に1度の出来なら1/10、50年に一度の出来なら1/50として、
100年に1度の出来なら確率はどの程度なの?
n年に1度の出来では?
164158:2011/11/23(水) 21:00:41.00
http://img125.imagevenue.com/img.php?image=48993_sekibunkana_122_389lo.jpg
図にしてみたけど、どうだろう…。f(x)の関数から解析的に F(v)を求めることは出来るんだろうか?
解析学? 確率論? 
なんの教科書に似たような話が載ってる、とか、そういう漠然としたコメントでも大歓迎です。
165132人目の素数さん:2011/11/23(水) 22:09:14.33
f(x)=vを解けばいいんじゃないの。
166132人目の素数さん:2011/11/23(水) 22:25:38.14
「解析的」って何を想定してるのか知らんが
f(x)の具体的な式が与えられていない以上、期待した様なモノは得られないだろう
167132人目の素数さん:2011/11/23(水) 23:13:41.03
>>146>>147
ありがとうございます!
168132人目の素数さん:2011/11/23(水) 23:47:18.35
>>112
inclusion map
169132人目の素数さん:2011/11/25(金) 02:54:24.16
判別式、軸、端点の問題で、このような結果が出ました。

a<-3,a>1・・・@

a<-2・・・A

a>-7/2・・・B

そして、自分の答えは、a<-3,a>1

間違っていました。どのように考えれば正答に納得できますか?
170132人目の素数さん:2011/11/25(金) 03:01:26.76
>>169
@、A、Bを同時に満たすaの範囲を求めたいのか?
そこのところが書かれてないから意味不明。
さらに正答も示されていない。
171猫 ◆MuKUnGPXAY :2011/11/25(金) 03:08:07.71
虚偽院生が大学院の質を下げ、教室を馬鹿の集まりに変えてしまいました。
虚偽院生が大学院の質を下げ、教室を馬鹿の集まりに変えてしまいました。
虚偽院生が大学院の質を下げ、教室を馬鹿の集まりに変えてしまいました。
虚偽院生が大学院の質を下げ、教室を馬鹿の集まりに変えてしまいました。
虚偽院生が大学院の質を下げ、教室を馬鹿の集まりに変えてしまいました。
虚偽院生が大学院の質を下げ、教室を馬鹿の集まりに変えてしまいました。


172169:2011/11/25(金) 03:10:51.74
そうです。答えは、

-7/2<a<-3

-7/2<aは、a>1に食われるのではないかと。
173132人目の素数さん:2011/11/25(金) 03:14:43.96
>>172
数直線かいてみれば分かる

まず@かつAの時点でa<-3だろ?
これとBを合わせれば-7/2<a<-3になる
174169:2011/11/25(金) 03:23:28.84
@とAで、a>1が消えてなくなると考えればいいのでしょうか?
175132人目の素数さん:2011/11/25(金) 03:28:03.49
>>174
@とAを同時に満たすことができるのはa<-3しかないからな
176169:2011/11/25(金) 03:30:49.10
よくわかりました。ありがとうございます。
177132人目の素数さん:2011/11/25(金) 20:24:59.24
ABCDから1つ取り出す時、Aの出る確率は0.25でAが2回続けて出る確率は0.25^2ですが
ABCDのどれかが2回続けて出る確率は(0.25^2)*4ですか?
178KingMathematician ◆LoZDre77j4i1 :2011/11/25(金) 20:52:51.01
確率論における確率とは,標本空間上の事象族上の確率測度のことで,これら三つを考えて初めて成り立つ.
179猫は園児 ◆MuKUnGPXAY :2011/11/25(金) 22:03:04.91
>>178
徹底抗戦で対峙スルさかいナ。



>178 名前:KingMathematician ◆LoZDre77j4i1 :2011/11/25(金) 20:52:51.01
> 確率論における確率とは,標本空間上の事象族上の確率測度のことで,これら三つを考えて初めて成り立つ.
>
180132人目の素数さん:2011/11/25(金) 22:16:21.08
すみません。教えてください!
X=(fi*{N}i)/{C}i のとき
(Xi/嚢i)=1 (i=1,2,3,...n) ...@
はいくらになるんでしょうか?

どうやって変形しても@の左辺が1になり,
1=1となってしまします。
@の左辺をfiの一次式にしたいのです。
{N}iと{C}iはそれぞれ炭素・窒素濃度です。
i=6のときを求めたいのですが,どなたか優しい方教えてください。
おねがいします。 
181132人目の素数さん:2011/11/25(金) 22:19:51.52
次の不定積分を{}内の変数変換を用いて求めてください。

∫1/(coshx+1)dx {t=e^(x/2)}

ほとんど手が出ませんorz
182132人目の素数さん:2011/11/25(金) 22:20:12.23
>>180
記号が曖昧で意味不明。
183132人目の素数さん:2011/11/25(金) 22:21:53.21
>>181
(t+1/t)^2 を計算してみる
184KingMathematician ◆LoZDre77j4i1 :2011/11/25(金) 22:29:04.42
Re:>>181 簡単な式のようで,いろいろな式変形を使う.指数法則,指数函数の導函数,合成函数の微分の公式.これで有理函数の積分になる.
185132人目の素数さん:2011/11/25(金) 22:32:19.60
>>181
 cosh(x) + 1 = 2 cosh(x/2)^2,
なので
 ∫1/{cosh(x)+1} dx = (1/2)∫ 1/cosh(x/2)^2 dx = tanh(x/2),
186132人目の素数さん:2011/11/25(金) 22:53:30.03
181です。返事遅れてすいません。
ありがとうございます。おかげで解決しました。
187猫は園児 ◆MuKUnGPXAY :2011/11/25(金) 23:14:29.19
>>184
オマエはアホか。



>184 名前:KingMathematician ◆LoZDre77j4i1 :2011/11/25(金) 22:29:04.42
> Re:>>181 簡単な式のようで,いろいろな式変形を使う.指数法則,指数函数の導函数,合成函数の微分の公式.これで有理函数の積分になる.
>
188132人目の素数さん:2011/11/26(土) 19:31:34.26
二次関数y=-x^2+2x+4の軸と頂点を求め、そのグラフをかきなさい
おねがいします
189132人目の素数さん:2011/11/26(土) 19:39:09.20
二次関数y=x^2-6x+1 (0≦x≦5)の最大値と最小値を求めなさい。
190132人目の素数さん:2011/11/26(土) 19:47:39.62
@直角三角形ABCにおいて、sinA=1/5のとき、次の値を求めなさい
 (1)cosA (2)tanA

A135°の三角形の値を求めなさい

Bφが鈍角で、sinφ=1/3のとき、次の値を求めなさい
 (1)cosφ (2)tanφ
お願いします
191132人目の素数さん:2011/11/26(土) 20:24:14.87
>>188
ヒント:-(x-1)^2=-x^2+2x-1
>>189
ヒント:(x-3)^2=x^2-6x+9、0<(0+5)/2<3<5
192132人目の素数さん:2011/11/26(土) 20:42:30.21
>>191
ありがとうございます
答えも教えてくれると助かります;;
193132人目の素数さん:2011/11/26(土) 21:51:20.21
ヒントをどう使ったのか、
考えたこと、やってみたことを書かない問い合わせには応じられない。
194132人目の素数さん:2011/11/26(土) 22:33:27.54
丸投げ君に何を言ってもムダですよ
195132人目の素数さん:2011/11/26(土) 22:42:10.10
>>193
そのヒントも理解できないのです
196132人目の素数さん:2011/11/26(土) 22:43:31.69
丸投げ君、こんばんは
197132人目の素数さん:2011/11/26(土) 22:46:32.59
こんばんは
198132人目の素数さん:2011/11/27(日) 02:21:46.44
ヒントを持って先生んとこにGO!
199132人目の素数さん:2011/11/28(月) 15:05:05.62
XBOXのポイントとかアイテムをダウンロードできるコードってのがアルファベットと数字混じりの25ケタなんだけど
これって確率的に適当に当てはめたら通っちゃったりしないの?
XBOXなんて世界中の人がやってるわけだし
200132人目の素数さん:2011/11/28(月) 16:37:13.06
>>199
アルファベットが26文字、数字が10個で合わせて36個
36^25≒8*10^38
世界人口はたかだか7*10^9
人類で手分けして、一人1秒あたりに1コード試しても
全コードを網羅するのに、宇宙の年齢をはるかに超えた時間がかかる
201132人目の素数さん:2011/11/28(月) 20:27:49.59
>>200
マジで!そうなのか…。レスありがとう
数学板って初めて覗いたけどすごいわ
202132人目の素数さん:2011/11/29(火) 00:45:43.74
どなたかいませんかぁ
203132人目の素数さん:2011/11/29(火) 01:10:00.02
チャットは他でやれ
204132人目の素数さん:2011/11/29(火) 01:23:58.65
いた!
In=∫[-1→1](1-x^2)^n dx
を計算したら
In=2n/(2n+1)In-1
になるらしいんですが何度やってもなりません。助けてください
205132人目の素数さん:2011/11/29(火) 01:48:07.82
>>204
x=cosθとでも置換する
後は部分積分。
そしたらI(n)=(2n/(2n+1))*I(n-1)になる
206132人目の素数さん:2011/11/29(火) 02:14:48.64
わざわざ置換しなくても1*(1-x^2)^nすぐ部分積分したらいいだろ。
207132人目の素数さん:2011/11/29(火) 02:29:49.67
ほんやね^p^
サーセン
208132人目の素数さん:2011/11/29(火) 18:01:55.79
どうも丸投げです
>>189(二次関数y=x^2-6x+1 (0≦x≦5)の最大値と最小値を求めなさい。)
を自分なりにやってみました。

最初に変形させたんですがこれであってますかねy=(x-3)^2-8
答えは最大値0 最小値-6になりました
答え合わせお願いします
209132人目の素数さん:2011/11/29(火) 18:08:36.94
>>208
式の変形はあってる。
でも最大と最小がちがう。
図がちゃんと描けてないのかもしれなんな。
210132人目の素数さん:2011/11/29(火) 18:27:31.66
>>208
どう計算するとそういう値になるんだ?
211132人目の素数さん:2011/11/29(火) 19:28:32.17
最大値6 最小値-5
でどうですか・・・?
212132人目の素数さん:2011/11/29(火) 19:36:13.32
>>211
どうですかじゃなくて何をどう計算してるんだ?
213132人目の素数さん:2011/11/29(火) 20:07:41.88
変形したy=(x-3)^2-8は合ってるみたいなので
これをグラフにあてはめて(0≦x≦5)の最大、最小値を出そうとしてますです
214132人目の素数さん:2011/11/29(火) 20:13:05.40
何を言ってるかわからん

最小値はx=3のとき-8
最大値はx=0のとき1だ。
215132人目の素数さん:2011/11/29(火) 20:25:29.88
ありがとうございます参考になりました
最小値は-4
であってますよね・・
216132人目の素数さん:2011/11/29(火) 20:43:05.48
>>215
何をどう計算したのか書け
217132人目の素数さん:2011/11/29(火) 20:44:04.57
次を満たす数列は存在しない事を示せ

・4項からなる
・全ての項は平方数である
・等差数列である
・公差は10!である
218132人目の素数さん:2011/11/29(火) 20:55:46.45
>>215
上のレスで最小値は-8になると書いてるでしょ?
219132人目の素数さん:2011/11/29(火) 21:30:36.38
>>216
計算というかグラフに当てはめただけなので説明が難しいです
>>218
x=5で最小値を求めないといけないんじゃないんですかね;;
問題はこれです
【二次関数y=x^2-6x+1 (0≦x≦5)の最大値と最小値を求めなさい。】
220132人目の素数さん:2011/11/29(火) 21:42:56.82
>>219
上に答え書いたじゃん。
レスしたんだからせめて読んでくれよ;;
221132人目の素数さん:2011/11/29(火) 21:46:39.13
222132人目の素数さん:2011/11/29(火) 21:47:45.27
グラフ書くときに計算するはずなのだが。
当てずっぽうで出してるのか?
223132人目の素数さん:2011/11/29(火) 21:53:07.51
>>219
区間の端で最大最小が起こると思ってるようだね。
224132人目の素数さん:2011/11/29(火) 21:53:32.27
>>220
すいません;;
頭ぐちゃぐちゃしててそれが答えかもわかりませんでした
>>221
このグラフは書けました
ありがとうです;;
>>222
超越した馬鹿ですすいません;;
225132人目の素数さん:2011/11/29(火) 21:54:18.73
>>223
そうです;;
226132人目の素数さん:2011/11/29(火) 22:57:42.83
y=-x^2+2x+4
=-(x^2+2x)+4
=-{(x+1)^2-1^2}+4
=-(x+1)^2-2+4
=-(x+1)^2+2
これあってるよね・・・?
227132人目の素数さん:2011/11/29(火) 23:14:07.47
>>226
2行目でアウト
228132人目の素数さん:2011/11/29(火) 23:40:57.77
う・・・
二行目こうかな?
=-(x^2-2x)+4
229132人目の素数さん:2011/11/30(水) 07:31:16.82
>>228
そうだよ。
だから当然3行目以降も間違ってるよ。
230132人目の素数さん:2011/11/30(水) 09:52:09.11
y=-x^2+2x+4
=-(x^2-2x)+4
=-(x-1)^2-1^2+4
=-(x-1)^2+4
=-(x-1)^2+2
答え合わせおねがいします?
231132人目の素数san:2011/11/30(水) 11:13:27.44
3行目でアウト
232132人目の素数さん:2011/11/30(水) 11:24:46.57
なぜさっきは出来たことが出来ん?
233132人目の素数さん:2011/11/30(水) 11:29:10.65
{}←これつけないとダメってことですか?
234132人目の素数さん:2011/11/30(水) 11:56:52.52
>>233
つけるとどうなる?
235132人目の素数さん:2011/11/30(水) 11:59:19.40
さっきも出来てねえな。
間違いだらけすぎる。
236132人目の素数さん:2011/11/30(水) 11:59:59.37
>>234
う〜ん・・・
わかんないです
なにも変わりませんかね・・
237132人目の素数さん:2011/11/30(水) 13:52:43.81
>>236
(x-1)^2-1^2って、x^2-2xを変形させたものだろ?
だから、
-(x^2-2x)+4のx^2-2xを(x-1)^2-1^2に置き換えることが出来る。
すると、-((x-1)^2-1^2)+4となるが括弧が重なっているので-{(x-1)^2-1^2}+4と表記する。
次に、{ }をはずすが、それは-(x-1)^2-1^2+4ではない。

あと、1^2は2ではない。
238132人目の素数さん:2011/11/30(水) 14:05:13.39
>>237
なるほど
y=-x^2+2x+4
=(x-1)^2-1^2
=-{(x-1)^2-1^2}+4
=-(x-1)^2-1+4
=-(x-1)^2+3
こういうことですね
ありがとでした!
239132人目の素数さん:2011/11/30(水) 14:16:33.05
>>238
違う。
240132人目の素数さん:2011/11/30(水) 14:18:38.92
バギャヤドー!
241132人目の素数さん:2011/11/30(水) 15:04:28.02
テンソル関係の記号を皆さんが何と読んでるか教えて下さい!
・とか×ならドット、クロスと読めると思うんですが、⊗とか∧とか…もう「かける」としか言えないんですが…(後者はハット?)
242132人目の素数さん:2011/11/30(水) 15:10:43.87
テンサーにウェッジじゃ
243132人目の素数さん:2011/11/30(水) 15:18:20.82
⊗ まるぺけ
∧ ぴょこ
244132人目の素数さん:2011/11/30(水) 22:16:55.93
>>239
うがー
違うんですかー
どこらへんから違いますか?
245132人目の素数さん:2011/11/30(水) 22:28:46.42
>>244
{ }をはずすところ。
当然、そのあとも違ってくるが。
246132人目の素数さん:2011/11/30(水) 22:29:46.85
>>244
最後の式を展開してみて
247132人目の素数さん:2011/12/01(木) 08:31:29.92
どうだろ・・
おそらく{}の後で因数分解するんですよね?
y=-x^2+2x+4
=-(x-1)^2-1^2
=-{(x-1)^2-1^2}+4
=-(x-1)^2-1*(-1)+4
=-(x-1)^2+5
248132人目の素数さん:2011/12/01(木) 08:53:50.32
>>247
2行目がなければ合っている。コピペしたときに消し忘れたか?
ただ、そうだとすると、
y=-x^2+2x+4から
=-{(x-1)^2-1^2}+4へ一足飛びに変形しているのに、その次が
=-(x-1)^2-1*(-1)+4ってのは違和感がある。
249132人目の素数さん:2011/12/01(木) 09:07:16.35
>>248
確かにそうかもしれないです
どもです!

皆さんどもでしたー!
250132人目の素数さん:2011/12/02(金) 05:25:28.53
次の関数の最小値を求めよ。
(1)y=|x-5|+|3x-9|

+左右2か所、x=2のとき、x=6のときはなぜ必要なんでしょうか?
+左右2か所が最小値になることがあるんでしょうか?
251132人目の素数さん:2011/12/02(金) 05:55:00.09
>>250
質問が意味不明。

問題集の解説かなにかから抜粋して質問してんの?
「+左右」って何?

ちなみに最小値はx=3のとき2。
252250:2011/12/02(金) 07:28:24.14
黄色い参考書です。パターン34『|(1次関数)|の最大・最小は場合分け不要』。
上の問題の+左右2か所というのはX=3の左でX=2です。X=5の右でx=6です。
参考書には折れ線グラフが書いてあるのですが。
253132人目の素数さん:2011/12/02(金) 11:21:03.34
日本語ダメな人、こっちに来たのか?
254132人目の素数さん:2011/12/02(金) 11:31:06.76
>>252
それはおそらく、yの値が、xが3より左のときはx=3のときよりも大きく(つまり、右下がり)、
xが5より右のときはx=5のときよりも大きい(つまり、右上がり)ことを示すために
その代表としてxが2のときと6のときを明示してるんだろうと思う。

黄色い参考書ってなんだよ。ちゃんと名前を書けよ。
255132人目の素数さん:2011/12/02(金) 17:13:08.54
エスパー試験か
256132人目の素数さん:2011/12/02(金) 19:10:01.70
ブレース中括弧{}の書き方がわかりません
257132人目の素数さん:2011/12/02(金) 19:34:56.46
>>250 >>252

 |x-5| は x=5 で折れ曲がり
 |3x-9| は x=3 で折れ曲がる。

グラフは折れ線。
辺の傾きは、左側より右側の方が大きい。
上に開いた凸多角形。
頂点のいずれか(または一辺)で最小値をとる。
258132人目の素数さん:2011/12/02(金) 23:55:27.94
∫[-a,0]f(x)dx = ∫[0,a]f(-x)dx を証明しろっていう問題なんですが、

x=-t とおくと dx=(-1)dt
xとtの対応は x:-a → 0 t:a → 0 なので、
∫[-a,0]f(x)dx = ∫[a,0]f(-t)(-1)dt = ∫[0,a]f(-t)dt

となったんですが、ここで行き詰りました。
どこか間違ってるところあったらアドバイスお願いします。
259132人目の素数さん:2011/12/02(金) 23:58:23.81
行き詰まってない
それで証明終わり
260132人目の素数さん:2011/12/02(金) 23:59:39.50
>>258
積分は同じもの、変数変換しただけ。
261132人目の素数さん:2011/12/03(土) 00:00:56.53
>>259
最後の式のtをxに変えてしまってもいいんですか?
262132人目の素数さん:2011/12/03(土) 00:02:10.25
すいません、>>260で解決しました。

>>259-260 ありがとうございます。
263あのこうちやんは始皇帝だった:2011/12/03(土) 00:16:19.24

 クソガキども、まだ生きていたのか。

 明日、息の根を止めてやっから、覚悟しとけ!!!!!!
264132人目の素数さん:2011/12/03(土) 00:16:59.04
明日とは言わずに今すぐお願いしますよ
265132人目の素数さん:2011/12/03(土) 02:23:53.60
AとBの2つの封筒がある。
金額は分からないが、いずれかにはもう一方の倍のお金が入っている。
どちらかもらうことができ、片方を開けた後に交換することもできる。

今、Aを開けると10000円入っていた。
ということは、Bには5000円か20000円が入っていることになり、
その期待値は12500円である為、Bに交換する方が得である。

このことは、Aがいくらであっても同じことが言え、
すなわちAの金額を確認する前からわかっていることなので、
最初からBを選べばよいはずである。

しかし、一度Bを開けるとAに交換した方がよいことになってしまう。

結局のところ、最初にどちらを選べばよいのか。
一度開けた後に交換した方がよいのか。
266132人目の素数さん:2011/12/03(土) 07:51:07.03
>>今、Aを開けると10000円入っていた。
>>ということは、Bには5000円か20000円が入っていることになり、
ここまでは正しい。
>>その期待値は12500円である為、
これが間違い。
http://kamome.2ch.net/test/read.cgi/math/1319861681/32-33
267猫 ◆MuKUnGPXAY :2011/12/03(土) 11:16:04.17
>>263
模範解答をせえや。ほんでアンタの公式の五匹目と六匹目も早う出せや。
ワシが観賞したるがな。


268132人目の素数さん:2011/12/03(土) 11:41:49.66
2封筒スレ、4まで行ってるの……
269132人目の素数さん:2011/12/04(日) 13:13:43.59
質問です。問題は、
~~~~~~~~~~~~~~~~~~~~~~~~~~~~~~~~~~~~~~~~~~~~~~~~~~~~~~~
Aは3×3行列で、
0 1 1
1 0 1
1 1 0
です。
3×3の直交行列Oで、O^(-1)AOが次に等しくなるものを求めよ。
2  0  0
0 −1  0
0  0 −1

~~~~~~~~~~~~~~~~~~~~~~~~~~~~~~~~~~~~~~~~~~~~~~~~~~~~~~~
です、O^(-1)はOのインバースということです。これの解き方が分かりません。

今やってみて、(あってるかどうか不明ですが)Aの固有値が2、-1で、
固有ベクトルを(x1,x2,x3)とすると
-1に対する固有ベクトルが、x1+x2+x3=0となるので(a,-a/2,-a/2)、
2に対する固有ベクトルが、x1=x2=x3となるので(b,b,b) (a,bは任意の定数)
まで出せました。
Oは直交行列なので、tO=O^(-1)を利用する気がしまして、
今出した固有ベクトルを使ってOを作るんだと思います。
そこで行き詰ってます。直交行列の定義から各列ベクトルの大きさは1ですよね。
それが満たせなくて・・・
例えばx1+x2+x3=0、√{(x1)^2 + (x2)^2 + (x3)^2}=0
を満たすx1,x2,x3は0しか無いのでは…と思って進めません。

発表でして、自分で調べてここまでたどり着いたんですがこっから分かりません。
できたら詳しい解き方・ヒントをお願いします。
270132人目の素数さん:2011/12/04(日) 13:19:39.60
>>269
>√{(x1)^2 + (x2)^2 + (x3)^2}=0
これは勘違いか書き間違いかどっちだ。
271132人目の素数さん:2011/12/04(日) 13:28:54.62
>>270

あっ
√{(x1)^2 + (x2)^2 + (x3)^2}=1
でした!すいません。
ちょっと考え直してみます。
272132人目の素数さん:2011/12/04(日) 14:33:57.29
うーんやっぱできません・・・
x1+x2+x3=0
√{(x1)^2 + (x2)^2 + (x3)^2}=1
この2つの条件でひとつを任意の定数とおいてやると
ありえない計算量になるので違う気しかしません。

-2に対する固有ベクトルなんて
y1=y2=y3
√{(y1)^2 + (y2)^2 + (y3)^2}=1
からもう必然的にy1=y2=y3=1/√3って決まってしまうような気が・・・


ヒントお願いします!
273132人目の素数さん:2011/12/04(日) 14:38:22.32
なぜそこまでいって解けないのか
274132人目の素数さん:2011/12/04(日) 14:55:28.19
>>273
すいませんできましたw
ありがとうございました
275132人目の素数さん:2011/12/05(月) 13:53:19.32
テスト
276132人目の素数さん:2011/12/18(日) 18:56:53.67
 ●
E @
D A
C B


@を選んでCを選ぶのと、Cを選んで@を選ぶのは同じだから、
2で割るのはわかるんですが、
@を選んでEを選ぶのと、Eを選んで@を選ぶのも同じなのに、
こちらの左右対称の方は2で割らないのがわかりません。
じゅず順列という所の話です。
277132人目の素数さん:2011/12/18(日) 19:07:48.18
>>276
すまんが元の問題をきちっと改変せずに書いてくれんか?
278276:2011/12/18(日) 19:21:20.89
>>277
黒1個、白2個、赤4個の合計7個の玉にひもを通してネックレスを作るとき、
作り方は何通りあるか。
279132人目の素数さん:2011/12/18(日) 19:22:36.44
曲線の接線の傾きを微積使わずに求める方法ってあります?
どんなに回りくどい方法でもいいんで教えてもらえますか?
y=x^2みたいに適当な例を置いて説明してもらえると幸いです。
280132人目の素数さん:2011/12/18(日) 19:24:32.24
>>279
高校の教科書を読む
281132人目の素数さん:2011/12/18(日) 19:25:38.34
>>280
高校の教科書いま手元にないんだ・・・
ってか高校の教科書に書いてある!?
282280:2011/12/18(日) 19:27:41.66
>>281
二次関数の場合なら
283276:2011/12/18(日) 19:29:12.33
すいません。自己解決しました。@とCが白、BとEが白が同じだから、
非左右対称の方だけ2で割るんですね。
284132人目の素数さん:2011/12/18(日) 19:29:23.18
>>282
出来れば方法教えて貰えないだろうか・・・?
285132人目の素数さん:2011/12/18(日) 19:46:18.40
>>284
連立方程式
y=ax^2+bx+c …(1)
y=px+q …(2)
の解(x,y)が一組(重解) ⇔ 直線(2)が放物線(1)の接線
286132人目の素数さん:2011/12/18(日) 19:50:43.13
>>285
あぁ、そういう方法があったか!
ありがとう!!
287132人目の素数さん:2011/12/19(月) 19:40:01.82
初期値が0、微分方程式x''+4x+∫[0,t]x(t)dt=uの式について質問ですが、
これを解くにあたって、x(0)=0、x'(0)=0、x''(0)=0というのは一行目の条件からどのように導くのでしょうか?

x(0)=0、x''(0)=0については、u(0)=0ならば微分方程式の三項目の積分は0から0の範囲の積分となってしまうために0となり、
残りのx''+4xを見ると0にしかならないから0ってことなのか?と考えています。
しかし、これだとあいまいな気がします。

x'(0)=0については、どうかんがえたらこれが示せるのかわかりません。

よろしくお願いします。
288132人目の素数さん:2011/12/19(月) 19:45:23.36
>>287
初期条件:x(0)とx'(0)は与えるもの
289132人目の素数さん:2011/12/19(月) 19:55:23.62
>>288
初期条件としてそう書かれていれば解決だと思うのですが、問題文が>>287の一行目のようになっていたので、
この場合どうやって解くのかなと思って質問させていただきました。

これは「初期値が0」というところをu(0)=0と考えていたのですが、>>288の初期条件と解釈すべきなんでしょか?
それとも>>288の初期値が無いと解けない問題なんでしょうか?
290288:2011/12/19(月) 19:59:36.47
>>289
そうか、t=0を代入すると積分は0
291132人目の素数さん:2011/12/19(月) 21:27:50.83
>>287
問題を正確に書いて
292132人目の素数さん:2011/12/21(水) 01:42:16.92
r=√(x^2+y^2+z^2),G(r)はrのC^2級の関数とするとき、f(x,y,z)=G(r)として、∂^2*f/∂x^2+∂^2*f/∂y^2+∂^2*f/∂z^2をGで表せ

全く手がでません
助けてください
293132人目の素数さん:2011/12/21(水) 07:43:19.27
>>292
調べる気がないのね
294132人目の素数さん:2011/12/21(水) 08:37:21.15
つーかここで答えてる人って何なの?
偉そうなこと言って優越感に浸りたいの?
まあ互いの利害関係は一致してるけどそういう奴らは気持ち悪い
295132人目の素数さん:2011/12/21(水) 08:56:54.81
>>292
これは「ラプラシアンの極座標表示」というベタな問題。「ラプラシアン 極座標」で
検索するといろいろ記述があるよ。ただ、一度は自分の手で計算したほうがいい(オレは
やったことないけど)。
296132人目の素数さん:2011/12/21(水) 09:51:30.38
>>294
> まあ互いの利害関係は一致してるけどそういう奴らは気持ち悪い
気持ち悪い奴と利害が一致してるか、かわいそーに
297132人目の素数さん:2011/12/21(水) 14:10:35.59
酷い読解力だなwwww
どう考えても答えを請う人と教える人で利害関係が一致してるってことだろwwwwww
これが理系か

298132人目の素数さん:2011/12/21(水) 14:47:06.33
文系でしかもばかか
299132人目の素数さん:2011/12/21(水) 17:32:46.03
ほぼ確実に>>296=>>298だなwwww
かわいそーに()
ばかか()
300132人目の素数さん:2011/12/21(水) 18:01:24.65
>どう考えても答えを請う人と教える人で利害関係が一致してるってことだろwwwwww

>>296はどう見てもそういう意味でとってるとしか見えないが
301132人目の素数さん:2011/12/21(水) 18:33:44.42
>>296は主語が抜けてるから正確に理解することはできない
理解してる奴は、それはただの脳内補完で理解したつもりになっただけ
>>296みたいな文章書く奴も、>>297みたいな勝手に理解した気になる奴も死ね
302132人目の素数さん:2011/12/21(水) 18:40:18.82
理解することはできないって誰が
303132人目の素数さん:2011/12/21(水) 18:56:33.06
自殺か
304132人目の素数さん:2011/12/21(水) 18:56:41.39
数学のスレの人って本当に頭悪いんだな
305132人目の素数さん:2011/12/21(水) 18:57:38.18
それはどう考えても書いた本人以外ってわかるだろ
ここはひどいインターネットでつね
306132人目の素数さん:2011/12/21(水) 18:58:21.67
自虐か
307132人目の素数さん:2011/12/21(水) 18:59:11.51
IDなしだと駄目だな
自演多すぎ
まあアホだからバレてないと思ってるんだろうが
308132人目の素数さん:2011/12/21(水) 19:06:11.74
(私には)正確に理解することはできない
309132人目の素数さん:2011/12/21(水) 19:09:33.90
>>308
が煽りたいだけのゴミクズでここには優越感を得るために来ている奴だということはよくわかった
310132人目の素数さん:2011/12/21(水) 19:11:04.05
数学板早く潰れろ
こんな屑共に板用意するならもっと有意義な使え
311132人目の素数さん:2011/12/21(水) 19:11:49.07
>>301は主語が抜けてるから正確に理解することはできない
理解してる奴は、それはただの脳内補完で理解したつもりになっただけ
>>301みたいな文章書く奴も、>>305みたいな勝手に理解した気になる奴も死ね
312132人目の素数さん:2011/12/21(水) 19:13:41.75
有意義な使え()
313132人目の素数さん:2011/12/21(水) 19:14:29.43
やけに大きなゴミ箱だな
314132人目の素数さん:2011/12/21(水) 19:16:14.45
2人で言い争ってるようにしか見えない
315132人目の素数さん:2011/12/21(水) 19:16:29.45
ゴミスレあげ
316132人目の素数さん:2011/12/21(水) 19:17:53.55
NGにも出来ないし本当にいらないな
317132人目の素数さん:2011/12/21(水) 19:22:02.86
ここまで全部俺の自演
318132人目の素数さん:2011/12/21(水) 19:37:54.26
>>314
お前と俺の二人だけ
319132人目の素数さん:2011/12/21(水) 19:39:35.25
言っとくけど>>314は俺じゃないぞ
他は知らん
320132人目の素数さん:2011/12/21(水) 19:41:23.81
>>317は俺じゃない
321132人目の素数さん:2011/12/21(水) 21:22:24.76
322132人目の素数さん:2011/12/21(水) 21:26:43.15
323shishiy:2011/12/22(木) 10:29:11.02
はじめまして。自己紹介です。私は、shishiyと申します。イラストとGデザインで、暮らしてます。
ホームページは、http://www.44y.jp/です。宜しくお願い申し上げます。
324132人目の素数さん:2011/12/22(木) 22:15:02.50
z=(x^2)(y^2)、x=t*cos(t)、y=t*sin(t) のとき、
(d^2)z/d(t^2)  を求めよ。
325132人目の素数さん:2011/12/22(木) 22:17:37.16
なぜ
326324:2011/12/22(木) 23:09:20.24
やっぱりいいです。
何とかなりました。
327132人目の素数さん:2011/12/23(金) 05:11:04.07
"任意の単位的可換環には拡大体が存在する"
は真でしょうか?
偽なら反例を教え下さい。
328132人目の素数さん:2011/12/23(金) 09:09:33.31
体には0でない零因子は存在しない。わかるね?
329132人目の素数さん:2011/12/23(金) 12:14:48.99
反例 mod 6。
拡大しても 4×2=4×5 であることは変わらない
から、一意な除算ができるようにはならない。
330132人目の素数さん:2011/12/23(金) 18:05:20.96
つかぬことを伺いますが
dx/dtのようなxの微分を、xの上に・を付けて表したりしますよね。
あれ、なんて呼びます?「えっくすどっと」とか?
331132人目の素数さん:2011/12/23(金) 18:12:41.39
ニュートン記法というらしいが物理で聞いた方がいいかも
332132人目の素数さん:2011/12/23(金) 18:25:01.87
「エックスドット」、「エックスドットドット」とか、呼んでた
333330:2011/12/23(金) 18:52:26.85
ニュートンさんが考えたとは知らなかったです。
とりあえず間違いではなさそうなので私もドットと呼ぶことにします。
334132人目の素数さん:2011/12/23(金) 23:11:30.01
n変数多項式fおよび0でない有限個の多項式g1,g2,…,gsがあって、

gi(a1,a2,…,an)=0 (1≦i≦s)

なるすべての複素数の組(a1,a2,…,an)に対してf(a1,a2,…,an)=0となるならば、fは恒等的に0に等しい.

という定理の証明において f≠0⇒fg1g2…gs≠0 が使われていたのですが、なぜこうなるのか教えて下さい。
335KingMathematician ◆LoZDre77j4i1 :2011/12/23(金) 23:19:39.31
Newtonは作用反作用の法則で作用と反作用の作用線は同じになることは仮定したか.
Re:>>334 それ以外の数でも0になりうることか.
336KingMathematician ◆LoZDre77j4i1 :2011/12/23(金) 23:22:31.29
Re:>>334 gi(a1,a2,…,an)=0(1<=i<=s)はすべてのiについて成り立つか,少なくとも一つのiについて成り立つか.
337132人目の素数さん:2011/12/23(金) 23:26:26.71
すいません、>>334

gi(a1,a2,…,an)=0ではなくgi(a1,a2,…,an)≠0です。
338132人目の素数さん:2011/12/23(金) 23:27:49.68
全てのiに対して成り立ちます。
339132人目の素数さん:2011/12/23(金) 23:31:15.81
ちなみに>>334は線型代数入門(齊藤)の附録T多項式、[3.2]です
340132人目の素数さん:2011/12/23(金) 23:33:28.08
代数学の基本定理か
> fg1g2…gs
これは
341132人目の素数さん:2011/12/23(金) 23:35:59.33
>>340
f*g1*g2*…*gs です
342132人目の素数さん:2011/12/23(金) 23:39:37.64
>>341
> f≠0⇒fg1g2…gs≠0
背理法だろ、fが恒等的に0でないとすると
343132人目の素数さん:2011/12/24(土) 00:05:09.37
>>342
すいません、よくわかりません
344132人目の素数さん:2011/12/24(土) 00:10:22.80
つまり、恒等的には0ではない多項式の有限個の積もまた、恒等的には0ではないということですよね。

本当なんですか
345132人目の素数さん:2011/12/24(土) 00:12:38.44
>>343
背理法はf≠0を出すまで
346132人目の素数さん:2011/12/24(土) 00:21:32.88
>>345
「f≠0⇒fg1g2…gs≠0」を背理法で示すなら「f≠0かつfg1g2…gs=0」を仮定するのですよね

対偶をとるにしても、導出されるべき結論はf=0ですよね

f≠0を出すとはどういうことですか。
347132人目の素数さん:2011/12/24(土) 00:23:20.86
>>346
> fは恒等的に0に等しい.
348132人目の素数さん:2011/12/24(土) 00:33:00.03
アホに絡まれたせいで混乱を極めてる質問者がかわいそうだ
349132人目の素数さん:2011/12/24(土) 00:34:05.08
>>348
こんばんはアホ
350132人目の素数さん:2011/12/24(土) 00:35:28.42
>>347
定理を導くための背理法で、「f≠0」と仮定して、矛盾を導くという流れはわかっていますが、

「f≠0」と仮定したところで、「fg1g2…gs≠0」が導けないから泣きそうになってるんです。
351KingMathematician ◆LoZDre77j4i1 :2011/12/24(土) 00:42:34.06
人への念の盗み見による介入を阻め。
352132人目の素数さん:2011/12/24(土) 00:48:33.45
>>350
0でない最高次係数の積が0にならないことから明らか
ただし係数環が整域でないときは成り立たないから気をつけること
353132人目の素数さん:2011/12/24(土) 01:14:22.27
>>352
定理の結論は「fは恒等的に0に等しい」ですので、背理法の仮定「f≠0」は、
「ある複素数の組(a1,a2,…,an)が存在して、f(a1,a2,…,an)=0」という意味だと思います。
ですからその論法では明らかではないのでは?
354132人目の素数さん:2011/12/24(土) 01:24:37.33
それを言うなら、
「ある複素数の組(a1,a2,…,an)が存在して、f(a1,a2,…,an)≠0」
だろう。fの係数がすべて0であればこれは成り立たないから
必ず0でない係数が存在する

まあこんなことをごちゃごちゃ言わなくても、f≠0は要するに
多項式として0でないという意味だ
355132人目の素数さん:2011/12/24(土) 01:35:58.22
>>354
また=と≠を間違えました。すいません

それで、そこからどうやって「fg1g2…gs≠0」が導かれるのでしょうか。
356132人目の素数さん:2011/12/24(土) 01:39:33.29
fの最高次係数をa≠0
g_iの最高次係数をa_i≠0
とすればfg_1g_2・・・g_sの最高次係数はa・a_1・a_2・・・a_s≠0
よってfg1g2…gs≠0
ただそれだけのことだよ
357132人目の素数さん:2011/12/24(土) 01:44:08.46
>>356
それは、「恒等的には0ではない」ということで、ある複素数の組に対しては0をとりうるのではないですか?

本当にお騒がせしています、すいません
358132人目の素数さん:2011/12/24(土) 01:50:56.05
「ある複素数の組に対しては0をとりうるのではないのですか」
は、どっちでも良いというか、ここでは全く関係ない問題
359132人目の素数さん:2011/12/24(土) 01:53:57.14
>>357
「fg1g2…gsが恒等的には0ではない」ことと「fg1g2…gs恒等的に0である」ことが
矛盾するというのが定理の論法だが何か納得できない部分があるかね?
360132人目の素数さん:2011/12/24(土) 02:06:57.55
>>358
すいません、途中からこんがらがってしまいました。
全部理解しました!ありがとうございます。
361エトス:2011/12/24(土) 03:27:50.74
>>327
拡大体がつくれないことがあるのは明らかですが、(零因子の個数が理由)
"多項式"を完全分解できるような拡大環(一意性は無し)の存在は示せます。
362132人目の素数さん:2011/12/24(土) 09:55:41.63
長さ12cmの針金を折り曲げて、たての長さxcmの長方形をつくる。
長方形の面積ycm^2の最大値を求めなさい
式のヒントオナシャス
363132人目の素数さん:2011/12/24(土) 10:36:46.55
>>362
よこの長さをxを使って表す
364132人目の素数さん:2011/12/24(土) 13:57:24.34
もう少しオナシャス…
365132人目の素数さん:2011/12/24(土) 14:04:09.56
やるきないのねー
366132人目の素数さん:2011/12/24(土) 14:26:58.93
あるわい!
367132人目の素数さん:2011/12/24(土) 14:29:33.25
なにをやった
368132人目の素数さん:2011/12/24(土) 14:43:31.80
長方形かいてx書いた
式の作り方がわかりませぬ
369132人目の素数さん:2011/12/24(土) 14:47:33.51
となりの辺の長さは?
370132人目の素数さん:2011/12/24(土) 14:49:22.94
とりあえずaにした
371132人目の素数さん:2011/12/24(土) 14:50:34.35
すべての辺を加えると
372132人目の素数さん:2011/12/24(土) 14:53:13.08
12cmになりますん
373132人目の素数さん:2011/12/24(土) 14:56:18.23
xとaであらわすだよ
374132人目の素数さん:2011/12/24(土) 14:59:27.44
y=xa
375132人目の素数さん:2011/12/24(土) 15:01:05.42
362か?
376132人目の素数さん:2011/12/24(土) 15:05:36.98
さようにござる
377132人目の素数さん:2011/12/24(土) 15:06:49.35
長方形の向かい合った辺の長さは?
378132人目の素数さん:2011/12/24(土) 15:13:59.39
同じ
379132人目の素数さん:2011/12/24(土) 15:17:22.06
式がたっただろ
380132人目の素数さん:2011/12/24(土) 15:51:41.36
すいませんまた夜に来ます
すいません
381132人目の素数さん:2011/12/24(土) 16:13:05.56
もうこないで
382132人目の素数さん:2011/12/24(土) 22:02:14.79
スレタイに忠実で、いい投稿じゃないか。
383132人目の素数さん:2011/12/26(月) 11:31:19.99
すんごい下らない質問ですが、教えてください。

今現在、数式(x^,-x等)を含まず、純粋に数字(10進に限らず)のみで表されている最も巨大な意味のある実数(何らかの数学的な理由が存在する数)は何になりますか?
x+1=xを永遠と繰り返すプログラムを走らせたコンピューターがはじき出している最大の数とかでも構いません。
384132人目の素数さん:2011/12/26(月) 11:34:12.92
>>383
>x+1=x
オーバーフロー
385132人目の素数さん:2011/12/26(月) 11:50:39.07
>>384
10 a=0
20 print a
30 a=a+1
40 goto 20
386132人目の素数さん:2011/12/26(月) 11:58:43.94
N88Basicか
387132人目の素数さん:2011/12/26(月) 12:00:24.10
>>383
ナイーブすぎる質問。
無着先生にききなさい。
388132人目の素数さん:2011/12/26(月) 12:10:50.16
>>383
それならとりあえずグラハム数についての感想が聞きたい
389132人目の素数さん:2011/12/26(月) 12:20:55.07
俺か、どうでもいい
390132人目の素数さん:2011/12/26(月) 14:42:30.18
>>388
wikiペ等で見てきました。
此の世で把握出来る数を遥かに超えている数を意味するその数は数字だけで置き換えて表記することが実際に不可能とか。

関数やらを使わずに数字だけで『記された』最大の数と把握されてる数が、どのように存在してるか凄く気になったのです。
それが何処にどのように存在してるのかなぁ?と。
391132人目の素数さん:2011/12/26(月) 18:13:55.36
大きくて、意味のある数といえば、メルセンヌ素数かなあ。
Wikiによれば 47番目まで見つかっていて、これは1500万桁くらいの整数。
392132人目の素数さん:2011/12/28(水) 17:30:20.23
[10^(10兆)π] は数字だけで記されていると言っていいのでは。
393132人目の素数さん:2011/12/28(水) 20:45:41.03
分割方法が決まってるケーキ分割問題を考えようとしたらこんな問題がでてきた

AさんとBさんは共有物としてある物を一つ貰った
それはAさんにとってはa円、Bさんにとってはb円の価値がある
物理的に分割すると無価値になるが、共有したままだと使い勝手が悪いので
話し合いの結果一人が引き取り、もう一人には金銭で埋め合わせをする事にした
誰が引き取って幾ら払うのが妥当か?

単純に考えると平均評価額/人数ずつ引き取らなかった人に配ればよさそうだが
何かもっといい評価方法がありそう
394132人目の素数さん:2012/01/03(火) 01:06:02.98
知り合いがすべての素数は 6m-1 or 6m+1 の形で表せることを発見したのですが、これは何かすごいことなんですかね?
証明も簡単だし、am±b[1] or am±b[2] or ... or ax±b[n] の形ですべての素数を表せる a ならばいくらでもあるので、
正直わたしはとくに意味のあることとは思わなかったのですが、ウィキペディアの素数の項にも載っていたのでぎょっとしました。

am±1 の形ですべての素数を表すことができる最大の自然数 a (1, 2, 3, 6) が 6 だからなのですか?


いちおう引用しておきます
http://ja.wikipedia.org/wiki/%E7%B4%A0%E6%95%B0
「5以上の素数は全て、自然数mを用いて6m+1または6m-1の形で表すことができる(逆は成り立たない)。」
395132人目の素数さん:2012/01/03(火) 01:13:23.54
ウィキペディアに乗ってるとぎょっとするというのがよくわからんな。
どうでもいいと思うなら消せばいいだろ。
396132人目の素数さん:2012/01/03(火) 01:32:45.40
はい
397132人目の素数さん:2012/01/03(火) 02:10:16.01
これは釣りなんだろうなあ。
>>394
2と3はどうなるんだ?
398132人目の素数さん:2012/01/03(火) 07:09:17.47
「エラトステネスの篩」を考えれば当たり前。同様に、
「7以上の全ての素数は、30n±1,30n±7,30n±11,30n±13で表せる」
というものもある。こちらは、素数をメモリに効率的に保存する方法として、
使われる事がある。(1バイト=8ビットだから)
399132人目の素数さん:2012/01/03(火) 15:22:38.83
整数係数2x2行列{{a,b},{c,d}}があり、ad-bc=1
とする。
これを{{x,0},{0,1}},
{{0,1},{1,0}},
{{1,y},{0,1}}
の有限個の積で表せ。
ただし、x=1,-1, yは整数。

わかる方いらっしゃいますでしょうか?
400132人目の素数さん:2012/01/03(火) 15:46:02.44
a,b,c,dについて、条件がad-bc=1だけなら、無理なんじゃない。
a=d=7、b=6、c=8とかは、表現できそうもない。
401132人目の素数さん:2012/01/03(火) 15:50:08.01
可能だよ
上の表記だと {{0,-1},{1,0}}, {{1,1},{0,1}} で十分
どっちにしろ、長くなってめんどくさいのでパス
402132人目の素数さん:2012/01/03(火) 15:51:54.76
>>401 無理だと思うが
403132人目の素数さん:2012/01/03(火) 15:53:12.93
逆行列は取っちゃダメだから {{1,-1},{0,1}} もないとダメか
404399:2012/01/03(火) 16:04:52.46
x=1はあまり意味がありませんでしたので書き直します。

整数係数2x2行列{{a,b},{c,d}}があり、ad-bc=1
とする。
これを{{-1,0},{0,1}}, (第一列の符号逆転)
{{0,1},{1,0}}, (第一列と二列を交換)
{{1,y},{0,1}} (第一列目の整数倍を二列目に加える)
の有限個の積で表せ。
ただし、yは整数。
405132人目の素数さん:2012/01/03(火) 16:27:13.96
表現できそうもないと書いたのは、早とちりのようだ。
ad-bc=1の整数解の表現方法こそが、この問題の本質のようだ。
406399:2012/01/03(火) 16:31:45.51
>>405 すみません、詳しく教えてもらえませんでしょうか?

実数でできる事は知っています。整数の範囲で出来るかどうか知りたいです。
407399:2012/01/03(火) 16:40:25.33
補足。
先の例、7,6;8,7の場合はわかるのですが、
一般の場合が分かりません。
どうやったら良いのでしょうか?
408132人目の素数さん:2012/01/03(火) 16:45:20.21
最初の直感が間違っていたようだと言うことに気づき、
それに伴って新たな直感を書いただけ。

具体的には、二つの整数m,nを使って、ad-bc=1の全ての整数解が
a=fa(m,n)
b=fb(m,n)
c=fc(m,n)
d=fd(m,n)
の形で表すことが出来、そのm,nが、行列表現をする時、どのような順番で掛けていけば
いいかを決めるキーになるだろうという、大ざっぱな解答方針が浮かんだだけです。
409132人目の素数さん:2012/01/03(火) 18:21:46.15
>>404
(1) 第1列の符号逆転
(2) 第1列と第2列を交換
(3) 第1列の整数倍を第2列に加える
以上の操作は行列式の絶対値を変えず、同種類で逆操作にもなる。
だから、積で表す問題を「行列を単位行列に変換する問題」にしても同等である。
まず、(2),(3) の操作を組み合わせると1列と2列でユークリッド互除法ができる。
そこで1行目に着目してユークリッド互除法を行うと有限回で 第1列≠0,第2列=0 となる。
(第1列=第2列=0 は |ad−bc|=1 より否定)
ここで {{a,0},{c,d}} となるが、|ad−bc|=|ad|=1 より a,d は ±1 である。
さらに (1),(2) の操作で {{1,0},{c,1}} となり、(3) で単位行列となる。
410399:2012/01/03(火) 19:47:41.36
大変よく分かりました。
ありがとうございました。
411132人目の素数さん:2012/01/04(水) 00:08:29.84
detA=detB^bdetC^cdetD^ddetE^e=1
412132人目の素数さん:2012/01/04(水) 11:27:33.57
状態無限って何ですか?
413132人目の素数さん:2012/01/04(水) 21:23:59.41
σコンパクトと局所コンパクトって同じですか?
可算個のコンパクトな開集合で被覆されることと、各点近傍にコンパクトな開集合が存在することがそれぞれの定義だと思ってます。
でも後者の定義の場合、被覆が可算個になるとは限らない気がするのですが…
414132人目の素数さん:2012/01/04(水) 21:47:25.35
415132人目の素数さん:2012/01/04(水) 21:50:12.28
σはコセットが存在するってことしょ。
416132人目の素数さん:2012/01/04(水) 22:20:39.08
>>412
ホンダの新しいエンジン
417132人目の素数さん:2012/01/05(木) 11:08:59.47
これどこが間違ってますかね。いくらやってもだめで

θが鈍角で、sinθ=1/3のとき、次の値を求めなさい。
(1)cosθ
sin^2θ=cos^2θ=1
cos^2θ=1-sin^2θ
=1-(1/3)^2
=1-1/9=8/9
cosθ<0より
cosθ=-2√2/3
418132人目の素数さん:2012/01/05(木) 12:00:43.92
>>417
あってるよ
419132人目の素数さん:2012/01/05(木) 12:08:38.75
>>418
やっぱりあってますよね。
教師からペケもらってすごい悩みました・・
ありがとです!
420132人目の素数さん:2012/01/05(木) 12:12:20.59
>>417
>sin^2θ=cos^2θ=1
これがタイプミスでないならペケでも文句言えない
421132人目の素数さん:2012/01/05(木) 12:16:46.71
>>417
> cosθ<0より
これの根拠を書かなかったからとか。
422バカ:2012/01/05(木) 14:02:06.13
円の方程式わからんw
423132人目の素数さん:2012/01/05(木) 14:21:03.94
ある点からの距離が一定
424413:2012/01/05(木) 14:42:00.07
>>414-415
ありがとうございます。やはり違うものということみたいですね。
2つを同じもののように扱っているサイトがあったので気になっていました。
まだよくわかっていないので、414を読んで勉強します。
425132人目の素数さん:2012/01/05(木) 15:14:56.24
数学'Uって何の略ですか?
426132人目の素数さん:2012/01/05(木) 18:04:11.93
「アトラクターであっても不変集合でない」
逆に「不変集合であってもアトラクターでない」
どっちもありえますよね?
427KingMathematician ◆LoZDre77j4i1 :2012/01/05(木) 20:18:18.46
アトラクターとは安定性の問題か.
428132人目の素数さん:2012/01/08(日) 19:50:49.28
質問、
∫1/(x^2+1)dx=arctan(x)+C
ですがこれを実変数複素数値関数と思って
∫1/(x^2+1)dx
=1/(2i)∫1/(x+i)-1/(x-i)dx
=1/(2i)(ln(x+i)/(x-i))+C
と計算しても、理論的には正しいのでしょうか?
Cの自由度を除けば一応値は一致するように思えます。
lnの値域が不安ですが、
定積分もF(b)-F(a) f(x)=1/(2i)(ln(x+i)/(x-i))
で計算できるということでしょうか。
429132人目の素数さん:2012/01/08(日) 19:54:54.59
>>428
>=1/(2i)(ln(x+i)/(x-i))+C
>lnの値域が不安ですが、
高校でやってないのになぜ計算できると思う
430132人目の素数さん:2012/01/08(日) 20:11:08.54
>>429
逆に計算できない理由が思い浮かばなかったのですが、
アウトだとすれば、どこがダメなのでしょう?
431132人目の素数さん:2012/01/08(日) 20:28:24.07
>>430
定義してないだろーーーーーーーーーーーー
432132人目の素数さん:2012/01/08(日) 20:31:25.60
>>431

複素数値関数でも定積分、原始関数は定義できますよね。
(高校数学の範囲外であることはわかっています。)
433132人目の素数さん:2012/01/08(日) 20:36:05.10
>>432
問題はlog(z)だよ
おませさん
434132人目の素数さん:2012/01/08(日) 20:40:16.10
>>429
bakage
435132人目の素数さん:2012/01/08(日) 20:40:33.51
arg(z)の範囲を1つ決めれば、
log(x+i)=log|x+i|+iarg(x+i)も微分可能だと思うのですが。
436132人目の素数さん:2012/01/08(日) 20:41:18.93
>>434
逆ギレか、厨房
437132人目の素数さん:2012/01/08(日) 20:44:36.14
7と8教えて下さい
広義積分です

http://beebee2see.appspot.com/i/azuY2IHBBQw.jpg
438132人目の素数さん:2012/01/08(日) 20:46:07.27
あーくたんじぇんと
439428:2012/01/08(日) 20:47:22.83
>>434
は私じゃありません。
440132人目の素数さん:2012/01/08(日) 20:49:56.56
>>439
ごめん。
434は脊髄反射しかできないカスだっだ
441132人目の素数さん:2012/01/08(日) 20:51:07.94
カジノのルーレットで赤か黒をかけるとき
赤が三回でたら黒にかける(それ以外の時はかけない)というかけかたをした時、
当たる確率はやっぱり50パーセントなんでしょうか?
自分は当たる確率は50パーセント以上になると思うんですが確証がもてません
どなたか教えてください
442132人目の素数さん:2012/01/08(日) 20:56:16.91
443132人目の素数さん:2012/01/08(日) 21:01:18.39
>>442
それがどうした。
444132人目の素数さん:2012/01/08(日) 21:03:37.33
>>441
なぜ50%以上になると思うのか?
誤解の内容が分からないと誤解の訂正もできるわけないだろ。
445132人目の素数さん:2012/01/08(日) 21:03:44.60
446132人目の素数さん:2012/01/08(日) 21:03:55.82
>>441
事象が独立かは統計を取った場合などを除き
数学の外で論じられることじゃないか?
とはいえ普通は十分に事象は独立だろう
447132人目の素数さん:2012/01/08(日) 21:21:48.41
>>444
赤、黒がでる確率はそれぞれ二分の1な訳だから試行回数が増えれば赤、黒がでる回数はほぼ半分ずつにならないとおかしいですよね?
一万回やったら偏りなくほぼ五千回ずつ出るはず・・・
なのでその途中の過程で、出た回数に偏りがあった場合、例えば赤の方が多くでてる場合、その後黒がより多めに出ないとバランスが取れない事になるような気がするんです。
つまり一回目のルーレットで赤が出る確率と百回連続で黒がでた後の赤がでる確率は後者の方が確率的に高くなるように思えるのですがどうでしょうか?
説明下手ですいませんがどなたか教えてください
448132人目の素数さん:2012/01/08(日) 21:28:02.37
百回黒が出た後一億回赤が出て一億回黒が出たら黒のでた割合は0.500000249だな
449428:2012/01/08(日) 21:40:23.98
>>445
1/(2i)log(x+i)/(x-i)
とは微妙に違いますね。
log(x+i)-log(x-i)=log(x+i)/(x-i)
としたのが不味かったのでしょうか
それ以外は成り立つってことでいいんでしょうか?
450数学は式です:2012/01/08(日) 21:41:40.22
>>449
微積分勉強しろ
451132人目の素数さん:2012/01/08(日) 21:44:02.93
>>428,449
>lnの値域が不安
これを解消するように複素関数を勉強すればよろし
452132人目の素数さん:2012/01/08(日) 21:45:28.33
>>429はbakaで>>428は計算間違い
453132人目の素数さん:2012/01/08(日) 21:46:40.19
>>452
おまえは(カス)^2
454132人目の素数さん:2012/01/08(日) 21:58:44.36
どなたか>>447わかる方いませんか?
一応エクセルでシミュレーションしてみたら確率は高くなる事が確認されたのですが
455132人目の素数さん:2012/01/08(日) 22:06:05.26
>>454
どんなシミュレーションをやったのやら
456132人目の素数さん:2012/01/08(日) 22:54:09.56
>>455
「(正しい意味での)高くなることが確認された」と、
「(たまたま)高い値を得た」の区別がつかないのだろう
457132人目の素数さん:2012/01/09(月) 01:06:29.03
仮に高くなったとしても、それを示すのは数学の仕事じゃない
458132人目の素数さん:2012/01/09(月) 01:53:42.11
すいません
エクセルのやつは間違いでした
確率は五分五分でかわりませんでした
459132人目の素数さん:2012/01/11(水) 17:46:09.92
微分可能でテイラー展開できない実関数の例で
f(x)=exp(-1/x)
というのがあると思いますがこの関数なんか名前あるんですか?
460132人目の素数さん:2012/01/11(水) 18:02:36.91
猫は459円関数、とでもしておけば?
461132人目の素数さん:2012/01/11(水) 21:44:54.69
>>459

 ボルツマン分布、アレニウスの式、…
462132人目の素数さん:2012/01/13(金) 05:32:34.90
答えを見てもわからない部分があったので教えて下さい。
(x−3)^2を丁寧に解くと(x−3)(x−3)
=x(x−3)−3(x−3)=x^2−3x−3x+9
=x^2−6x+9
だと思うんですが答えの途中の式を見ると
x^2−2*3*x+3^2
とあるんです。
これの−2*3*xはどこからきたか解りますか?
なんだか昔カッコのついた式のじじょうの簡単なやり方みたいので教わった気もするんですがやっぱりよくわかりません。
よろしくお願いします。
463132人目の素数さん:2012/01/13(金) 07:47:31.65
>>462
君のいう丁寧なと着方をしていないだけ。
(a-b)^2=a^2 -2ab +b^2というのはこれを習った後はそのまま利用してよいとされるので、
それを利用しただけ。
464132人目の素数さん:2012/01/13(金) 11:41:53.66
>>463
なるほど!確かにそんな感じのおぼろげながら習った記憶あります!
久しぶりに数学やってみようと教科書開いてみたんですがちんぷんかんぷんで困ってしまったので本当助かりました。ありがとうございました。きいてよかった…
465132人目の素数さん:2012/01/15(日) 03:22:27.72
行列A={{2,-1,-1},{0,3,1}.{0,-1,1}}のジョルダン標準形ってどんな形になりますか?
wolframAlphaで計算するとJ={{2,0,0},{0,2,1},{0,0,2}}になるんですが
授業のノートにはJ={{2,1,0},{0,2,0},{0,0,2}}と書いてあります。
どちらでもいいのでしょうか?
466132人目の素数さん:2012/01/15(日) 03:30:52.47
どちらでもいいのかダメなのかも授業でやってる。
467エトス:2012/01/15(日) 19:04:51.87
ジョルダン細胞を並べ替えたものは元のものと相似ですからどちらでもO.K.
468132人目の素数さん:2012/01/17(火) 21:35:11.35
問題
a=3+√7の時、α=a^3+4a^2+5a+2を求めよ
回答
a=3+√7より、
a-3=√7
(a-3)^2=7
a^2+6a+2=0 @
ここで、整式の除法を用いて「x^3+4x^2+5x+2」を「x^2+6x+2」で割ると、【◎】
商はx+2で、15x-2余る
したがって、x^3+4x^2+5x+2=(x^2+6x+2)(x+2)+15x-2         【☆】
という式が成立する
ここで、求めるαはこのxにaを代入したものであるが、@より
α=0・(a+2)+15a-2
α=15a-2
α=15(3+√7)-2
α=43+15√7

という問題なんですが、☆に@を代入することは◎において「x^3+4x^2+5x+2」を0で割ることになってしまう為、
☆の式が成り立つのはa^2+6a+2≠0の時のみだと思うのですが、上の回答はあっているのでしょうか?
しばらく余りのある割り算から離れていたせいかどうもしっくりきません
469132人目の素数さん:2012/01/17(火) 21:37:27.37
少し間違っていたので修正します
回答
a=3+√7より、
a-3=√7
(a-3)^2=7
a^2-6a+2=0 @
ここで、整式の除法を用いて「x^3+4x^2+5x+2」を「x^2-6x+2」で割ると、【◎】
商はx+2で、15x-2余る
したがって、x^3+4x^2+5x+2=(x^2-6x+2)(x+2)+15x-2         【☆】
という式が成立する
ここで、求めるαはこのxにaを代入したものであるが、@より
α=0・(a+2)+15a-2
α=15a-2
α=15(3+√7)-2
α=43+15√7
470132人目の素数さん:2012/01/17(火) 22:19:35.51
>>469
x^2-6x+2はゼロ多項式でないのでなんら問題ないです.
多項式と代入してできる実数とを混同されていると思われます.
471132人目の素数さん:2012/01/17(火) 22:21:44.86
多項式と我々が日常で扱っている数は異なる概念です.
多項式に数を"代入"するという操作を行うことで,
その数が含まれる世界の話にすることができます.
472132人目の素数さん:2012/01/17(火) 22:23:37.55
>>471
ha
473132人目の素数さん:2012/01/17(火) 22:37:31.67
R,Sを単位的可換環とし,RからSへの準同型hを任意に1つ与え,
また,任意にα∈Sを取ります.
このとき,ψ(X)=αかつ任意のr∈Rでψ(r)=h(r)を満たすような
ψ:R[X]→S (hom) がただ1つ存在することがいえます.
f∈R[X]に対して,ψ(f)をfのXにαを代入した値といいます.
hにはしばしば恒等写像が採用されます.
474132人目の素数さん:2012/01/17(火) 22:40:00.19
恒等じゃなくて包含といっておきます
475132人目の素数さん:2012/01/17(火) 22:43:11.61
>>473
恒等式がでてこないが
476132人目の素数さん:2012/01/17(火) 22:48:21.21
上記は代入とはなにかを説明したものです
高校では大抵複素数体Cの中で考えるので
多項式環はC[X]の中で考えればいいでしょう
なのでhは恒等写像にすれば十分だとおもいます
477132人目の素数さん:2012/01/17(火) 22:49:06.59
>>475
なんの恒等式?
478132人目の素数さん:2012/01/17(火) 22:49:28.96
>>476
仙石60か?
479468:2012/01/17(火) 22:50:48.14
>>470
確かに、多項式のみで代入して☆の式を得ることは問題ないと思うのですが、そこに0になる値を入れたときも成立するというのがいまいちピンとこないんですよね
実際に最終的には実数になっていますし
似てるか分かりませんが、
y=x/xという式があった時、y=1となるが、x/xにx=0を代入すると分母も0になって計算出来なくなりますよね?
でもx=0においてもyの値は1になるのにどうもイメージが湧かないんですよね
480132人目の素数さん:2012/01/17(火) 22:51:03.27
>>477
468を読んだ上での質問か?
481132人目の素数さん:2012/01/17(火) 22:53:01.87
>>479
なにを代入しても問題ないのですよ
どうぞ好きな数を代入してください
虚数単位でもかまいませんよ!
482132人目の素数さん:2012/01/17(火) 22:53:33.08
>>480
勿論
【☆】のことかなぁ、という気はしなくもないが
483132人目の素数さん:2012/01/17(火) 22:54:13.19
>>482
カス
484132人目の素数さん:2012/01/17(火) 22:55:41.16
>>479
なんら前提がないとすれば
x/x の表記が意味をなすのは xがゼロでないときです
したがって,x=0においても x/x=1 というのは一般に間違っています
485132人目の素数さん:2012/01/17(火) 22:58:03.71
ちなみに,x/xをなんらかの有理関数体の中で考えているとするならば
(平たくいうと2つ多項式の分数で表される多項式全体)
x/x=1 で正しいです
486132人目の素数さん:2012/01/17(火) 23:01:10.13
i486げっと
487132人目の素数さん:2012/01/17(火) 23:03:28.49
たびたび勘違いする人がいますがここでもう一度いっておくと
多項式で使われるX(またはxとかtとかTとか)は不定元と呼ばれる
ものであり,これは実数などとは異なる概念のものです.
したがって,まったく同様には扱ってはいけません.
488132人目の素数さん:2012/01/17(火) 23:04:56.61
>>487
>不定元
たびたびいうがじじの薀蓄はいらない
489132人目の素数さん:2012/01/17(火) 23:06:52.62
なにを代入してもいいといいましたが一応制約があります
しかしながら高校数学に限定するならそこは注意する必要はありませんし
(多項式に多項式を代入したりなど大抵は許されます)
大学にあがったら意味がわかるようになるのでやはり問題ありません
490132人目の素数さん:2012/01/17(火) 23:13:11.10
オートマトンの問題で、
Σ = { a , b }とする。以下の命題の真偽を簡単な理由(証明は不要)とともにのべよ。
(1) Σ上の言語{ a^n b^n | 1000<= n} は正則である。(nは0を含む自然数)
(2)Σ上の言語{ a^m b^n | n + m = 3l を満たすlが存在する} は正則である。
(3)Σ上の言語{ a^n w b^n | w ∈Σ、|w| <= 100 } は正則である。

よろしくお願いします
491132人目の素数さん:2012/01/17(火) 23:14:51.88
>>490
訂正(4)は100 <= |w| でした
492132人目の素数さん:2012/01/17(火) 23:15:40.91
>>491
(4)じゃなくて(3)でしたっ
493132人目の素数さん:2012/01/17(火) 23:15:56.93
【☆】の例で処理してみます
Cを複素数体とします
ψ(x)=aによりCの元を不変にするような
準同型ψ:C[X]→Cが一意的に定まります
x^3+4x^2+5x+2=(x^2-6x+2)(x+2)+15x-2
の両辺にψを取ると
ψ(x)^3+4ψ(x)^2+5ψ(x)+2
=(ψ(x)^2-6ψ(x)+2)(ψ(x)+2)+15ψ(x)-2
(ここで準同型の性質を用いています)

ψ(x)=a だから
a^3+4a+5a+2 = (a^2-6a+2)(a+2)+15a-2 が得られた
494132人目の素数さん:2012/01/17(火) 23:16:06.45
自動羊か?
495132人目の素数さん:2012/01/17(火) 23:24:06.36
>>479
俗に「多項式の割り算」と呼ばれるものがやっているのは、
その例で言えばx/x=1ではなくてx=x(=x*1+0)を作る行為なので。
496132人目の素数さん:2012/01/17(火) 23:26:13.62
>>489
実は理系(非数学)卒の現社会人なんですけど、ちょっと訳ありで高校数学から勉強し直しているところです。
高校の時はこの辺もすんなり受け入れられたのですが、中途半端に余計な知識が付いたせいか今回のような疑問に度々ぶつかります。
いかにいい加減な理解で終わらせていたと思いますが・・・
そこでなんですけど、もしよければ今回の件等も含めて詳しく書かれたお勧めの参考書例があれば教えたいただけませんでしょうか?
497132人目の素数さん:2012/01/17(火) 23:31:35.54
>>496
"本質"を学ばれたいのであれば高校数学は無視して
集合写像(たとえば松坂和夫の集合・位相入門)
とすすんで代数学(たとえば桂利行の代数学T)
をやってみられるといい予感がします
そうすれば代数の言葉もわかるようになるので
とくに代入がなにを意味しているのかも
完全に理解できるようになるとおもいます
498132人目の素数さん:2012/01/17(火) 23:39:37.23
多項式をある形に変形するのが多項式の割り算です
変形ですから多項式の割り算といちいち断る必要は論理的に無いです

一方,除法の原理というものがあります
これはf(X),g(X)を多項式とするとき(g(X)はゼロ多項式でないとします)
f(X)=q(X)g(X)+r(x)かつdeg(q)>deg(r)を満たす多項式q(X),r(X)
の組が一意的に取れるというものです
これはたとえば高校数学でよくあるような
商や余りを問う問題でしばしば有効利用されます
499132人目の素数さん:2012/01/17(火) 23:42:37.30
『非負整数の集合をNで表すとき、環Rを係数とする一変数多項式とは
NからRへの写像fであって殆ど全てのn∈Nに対してf(n)=0となるものである。
gをもう一つの多項式とするとき、fとgの和とはh(n)=f(n)+g(n)で定まる多項式hのことである。
また、fとgの積kとはk(n)=杷(i)g(j) なる k をことである。
ここに狽ヘ、i+j=nを満たすi,j∈N全てを走るときの和を表す。』
500132人目の素数さん:2012/01/17(火) 23:44:52.14
>>497、レスしてくれたみなさん
分かりました、今までの数学の関わりだところどころ「本当にそうなのか?」といった疑問を完全に拭い切れずにやりきれない思いでいっぱいでした。
独学となるのでかなり厳しいことになるかとも思いますが頑張ってみたいと思います
ありがとうございました
こうしてみると数字の概念も理解せずに中・高の数学を教えるというのは何をしているのか疑問を持ちますね・・・
501漁協の方からきました:2012/01/17(火) 23:47:47.32
今日もネラーのミエのためにさまよえる羊が一匹できましたとさ。
めでたし、めでたし
502132人目の素数さん:2012/01/17(火) 23:48:51.11
なんだ、漁協さん密漁を見逃していたのかよw
503漁協の方からきました:2012/01/17(火) 23:51:30.10
漁業共同組合員のために働いております
504132人目の素数さん:2012/01/17(火) 23:56:31.58
乗法単位元を持たない環Rに係数をもつ文字Xの多項式環における一次の単項式とは何か?
505132人目の素数さん:2012/01/20(金) 21:46:17.85
A>0,B>0の場合、

A^s+B^(2-s)を縦軸、sを横軸にした場合

A^s+B^(2-s)の描く曲線はなんという曲線か調べよ。

また、
A^s+B^t=A+B
となる時の、sとtを求めよ。
506132人目の素数さん:2012/01/20(金) 21:48:43.52
507132人目の素数さん:2012/01/22(日) 19:34:06.67
どこで質問をしたら良いのか分からず、他スレにて質問したところスレ違いとの指摘を受けました。
http://uni.2ch.net/test/read.cgi/math/1324646365/541-544
ですので再度、ここで質問させていただきます。

541 自分:132人目の素数さん[sage] 投稿日:2012/01/22(日) 19:03:18.81
当方、中卒です。
先日、友人と1〜9までの数字を3つ選んでその数字と並びを当てるゲームというのをやりました。
例えば、Aは329、Bが251と選んだとします。
AはBの数字とその並びを当てるために最初はあてずっほうで321だろ?と言ったとします。
Bの数字は251ですので、1ヒット(数字の並び位置は間違っているが選んだ数字が入っている場合)
1フォーマ(これは位置も当たっている場合)とAに伝えます。
この場合は、1ヒットは2、1フォーマは1となります。
しかし、Aはどの数字がヒットしていて、どの数字と並びが正解(この場合、1)しているのかはこの時点では分かりません。
次はBがAの選んだ数字と並びを当てるために3つの数字を選び、Aがヒットしているかフォーマしているかを伝えます。

このゲームをした時、最短で何手で正解を導けますか?
また、申し訳ありませんが当方中卒のため、数式で答えられても理解するのは難しいのでえ
最短のセオリーを言葉で説明していただけると大変助かります。
よろしくお願いします。
508507:2012/01/22(日) 20:09:15.09
スレ違いを指摘されたスレにてお答えをいただき、解決しました。
ありがとうございました。
http://uni.2ch.net/test/read.cgi/math/1324646365/545
509132人目の素数さん:2012/01/23(月) 14:49:03.40
期待値の問題です。

確率変数Xは、その値 -2,0,2をそれぞれ、確率0.4,0.3,0.3でとる。次の各確率変数の場合の期待値はいくつになりますか?

1.X 2.X^2 (3) 3X^2 + 5

よろしくお願いします。
510132人目の素数さん:2012/01/23(月) 23:28:24.27
とある書籍で、ある数列a[n]について
  lim_[n→∞]|a[n+1]/a[n]|<1 ならば lim_[n→∞]a[n]=0
ということを証明する際に、最初の仮定として
  |lim_[n→∞](a[n+1]/a[n])|<1
をおいていたのですが、このやり方は正しいのでしょうか?

この証明の仕方だと
  |lim_[n→∞](a[n+1]/a[n])|<1 ならば lim_[n→∞]|a[n+1]/a[n]|<1
というように、「極限値の絶対値」と「絶対値の極限値」が同じであるということを示さなければならないと思うのですが、
書籍ではそのことについて特に触れられてはいませんでした。
511132人目の素数さん:2012/01/23(月) 23:46:52.84
「僕が見てる本のやり方は正しいですか?」とか聞かれても
お前が何を見てるかなんて知らないので答えようがない。
512132人目の素数さん:2012/01/24(火) 10:31:29.83
>>511
すみません、もう少し簡潔な聞き方にします。
ある数列a[n]について、
  |lim_[n→∞](a[n+1]/a[n])|<1 ならば lim_[n→∞]|a[n+1]/a[n]|<1
は成り立つのでしょうか?
すなわち、数列の極限において「極限値の絶対値」と「絶対値の極限値」は同じものとして見て良いのでしょうか?


510の書き込みについては、その書籍で
  lim_[n→∞]|a[n+1]/a[n]|<1 ならば lim_[n→∞]a[n]=0
が結論として示されているのに、実際には
  |lim_[n→∞]a[n+1]/a[n]|<1 ならば lim_[n→∞]a[n]=0
が証明されていたので、上記のような疑問が出てきたということだったのですが、分かりづらくてすみませんでした
513132人目の素数さん:2012/01/24(火) 11:09:50.59
本当にその本に書いてあることをきちんと書いてあるのなら
必要なのはlim_[n→∞]|a[n+1]/a[n]|<1ならば|lim_[n→∞](a[n+1]/a[n])|<1だろ
514132人目の素数さん:2012/01/24(火) 11:44:35.85
>>513
すみません、調べたところどうやら書籍自体に誤植があったようです。
こういった類の書籍を読むのは初めてなので、まさか誤植があるとは思いませんでした……
正しく下らない質問ですみませんでした。
515132人目の素数さん:2012/01/24(火) 16:03:41.37
「関数f(x,y)(←具体的に与えられている)を、3次の項までテイラー展開せよ。」
という問題のとき、剰余項は必要ですか?
516132人目の素数さん:2012/01/24(火) 16:24:41.25
ああつまらん
517132人目の素数さん:2012/01/24(火) 16:42:06.78
固有値を求める方法がわかりません。
L=y^2((∂/∂x)^2+(∂/∂y)^2)
よろしくお願いします。
518132人目の素数さん:2012/01/24(火) 17:22:53.50
>>515
厳密に言えば必要だろうが、
ランダウのoやその他の略記法を用いたり省略したりしていいかどうかは
お前が直面している場面でのローカルルールに従え、
という意味でこんな場所で訊くことが間違ってる。
519132人目の素数さん:2012/01/24(火) 18:57:10.78
>>517
削除します
520132人目の素数さん:2012/01/24(火) 19:03:56.97
521132人目の素数さん:2012/01/24(火) 19:06:37.06
大型ルアーきたな
メッセージコード:0013
メッセージ:セッションの有効期限が切れました。最初から操作してください。

ざんねん><
522猫はイケズ ◆MuKUnGPXAY :2012/01/24(火) 20:24:56.95
>>517
>固有値を求める方法がわかりません。
>L=y^2((∂/∂x)^2+(∂/∂y)^2)
>よろしくお願いします。
その作用素は:
L=y^{-2}((∂/∂x)^2+(∂/∂y)^2)
の間違いやろ。



523132人目の素数さん:2012/01/24(火) 20:28:10.35
>>522
ありがとん
524132人目の素数さん:2012/01/24(火) 23:32:34.88
>>521URLミスりました
http://kaigannsinnsyo857.blog.fc2.com/ここのURLの練習問題2〜14まで
答え合わせしたいんでお願いします
525132人目の素数さん:2012/01/25(水) 00:36:48.67
>>524
おお、答え合わせだな、心得た、任せておけ。
高校の頃、友人とは良くやったものだ。

で、お前の答はどこ?
526132人目の素数さん:2012/01/25(水) 01:20:03.64
>>525
はい 答えなんてありませんよ。 数学まじわかんないもん あほだから
答えが知りたい お願いします
527524:2012/01/25(水) 01:26:35.49
真剣に急いでいるので、早急にお願いします!
もう寝るので、答え合わせは起きてからにしておきます
528132人目の素数さん:2012/01/25(水) 03:10:18.38
雑魚すぎだろwww
529132人目の素数さん:2012/01/25(水) 05:19:34.09
(2)p,q,rを0≦p≦q≦rとp+q+r=6を満たす実数とする。
この時のpqの最大値は「カ」
qrの最大値は「き」
rpの最大値は「ク/ケ」

pq+prの最大値は「コ」
pq+qr+rp=の最大値は「サシ」
530132人目の素数さん:2012/01/25(水) 08:27:45.56
>>524みたいな初めの数項を書き並べるやり方って
数学板とかだと定義されてない扱いにならないのかな
「簡単な」数列って前提を付け足して読めば一意に定まるんだろうけど
531132人目の素数さん:2012/01/25(水) 08:52:00.90
>>524
公務員試験がんばってね
532132人目の素数さん:2012/01/25(水) 15:34:25.89
>>530
必ず誰かはそう言う。
533132人目の素数さん:2012/01/25(水) 16:59:24.02
>>530
「簡単」かどうかはどう判定するんだ?
例えば、数学の素養が無い人にとって、フィボナッチ数列 1, 1, 2, 3, 5, 8, ... は「簡単」なのだろうか?
534132人目の素数さん:2012/01/25(水) 21:02:56.49
一般に,一般項を求める問題はなんらかの条件が無い限り意味不明
なんらかの条件とはたとえば階差数列が定数列になるとか

なんも条件がないならば次にくる数は
ΣΠ(x-a[i])の形を考えればいくらでも捏造できるし
多項式型じゃなくてもグラフをみれば
いくらでも解析的につなげることができるので
いくらでも解析関数が考えられる おまんこ
535132人目の素数さん:2012/01/28(土) 17:30:12.30
h
536132人目の素数さん:2012/01/29(日) 03:49:51.42
1つのサイコロを3回投げるとき、目の最大値が5となる確率を求めよ。
答えは、61/216です。
参考書の解き方は簡単ですぐに理解できました。
ただ、自分のやり方で何が間違っているのか知りたくて質問させていただきたく存じます。

216/216から、最大値が4以下と最大値が6になる確率を引こうと思いました。
最大値4が以下になるのは64通り、これは間違いないと思います。
最大値が6になるのが、1回目が6の場合36通りだから、2回目が6、3回目が6も同様にして、
36×3=108通りとなりました。
108通りのうち1回目、2回目、3回目すべてに6が出るのが重複しているので、106通りになりました。
そして答えは、46/216となってしまいます。つまり、あと15通り重複しているようです。
なにが間違っているのか教えていただけませんか?
537132人目の素数さん:2012/01/29(日) 04:05:00.00
あれやん
6が2回出てその他の目が一回でるパターン
忘れてるやん
538132人目の素数さん:2012/01/29(日) 04:29:15.67
1回目が6で固定して、2回目が6で、3回目が1、2、3、4、5のどれか。
2回目が6で固定して、1回目が6で、3回目が1、2、3、4、5のどれか。
この2つでちょうど5通り重複してますね。よくわかりました。
ありがとうございます。
539132人目の素数さん:2012/01/29(日) 18:32:46.09
電気工事士1級の問題が全然分からないんですが どこか教えてくれるスレはないですか?

今年の試験で絶対受からないといけないので 何方か教えてください
540132人目の素数さん:2012/01/29(日) 18:44:58.23
(a+a^2+...+a^6)(b+b^2+...+b^6)(c+c^2+...+c^6)
5xx,55x,555
1/6*4/6*4/6*3+3C2*4/6^3+1/6^3
48+12+1=61
6^3=36*6=216
61/216
541132人目の素数さん:2012/01/29(日) 20:50:03.33
>>539
電気・電子板と資格全般板にあるけど、調べたのか?
542a:2012/01/30(月) 00:26:50.39
テスト
543543:2012/01/30(月) 00:38:27.77
集合のすごく初歩的な質問です。
集合A1、A2、A3について以下の関係を証明したいです。
(A1∩A2∩A3)∩B=(A1∩B)∩(A2∩B)∩(A3∩B) 自明かもしませんが、いざキチンと示そうとするとどう示せばよいか思いつきません。 よろしくお願いします。
544132人目の素数さん:2012/01/30(月) 00:41:13.88
何かを思いつく必要のあるものではない。やるべきことをやるだけ
545543:2012/01/30(月) 00:49:45.80
すいません。
具体的に示してもらえないでしょうか?
546132人目の素数さん:2012/01/30(月) 01:02:14.92
x∈(A1∩A2∩A3)∩B とすると
x∈A1∩A2∩A3 and x∈B
=> x∈A1 and x∈A2 and x∈A3 and x∈B
=> (x∈A1 and x∈B) and (x∈A2 and x∈B) and (x∈A3 and x∈B)
=> x∈A1∩B and x∈A2∩B and x∈A3∩B
=> x∈(A1∩B)∩(A2∩B)∩(A3∩B)
なので、(A1∩A2∩A3)∩B ⊆ (A1∩B)∩(A2∩B)∩(A3∩B)
⊇も似たり寄ったり
547543:2012/01/30(月) 01:15:27.04
>>546
示していただきありがとうございます。

=> x∈A1 and x∈A2 and x∈A3 and x∈B
=> (x∈A1 and x∈B) and (x∈A2 and x∈B) and (x∈A3 and x∈B)

ここがよくわかりません。この部分はなぜ成り立つと言えるのでしょうか?
548132人目の素数さん:2012/01/30(月) 01:34:35.92
x∈A1 and x∈A2 and x∈A3 and x∈B
=>x∈A1 and x∈A2 and x∈A3 and x∈B and x∈B and x∈B
=>x∈A1 and x∈B and x∈A2 and x∈B and x∈A3 and x∈B
=>(x∈A1 and x∈B) and (x∈A2 and x∈B) and (x∈A3 and x∈B)
または
x∈A1 and x∈A2 and x∈A3 and x∈B
=>x∈Ai and x∈B (for i = 1,2,3)
=>(x∈A1 and x∈B) and (x∈A2 and x∈B) and (x∈A3 and x∈B)
お好きな方で
549543:2012/01/30(月) 02:27:39.26
再びありがとうございます!
前者はよくわかりました。後者は次の部分が成り立つ理由がわかりません。
x∈A1 and x∈A2 and x∈A3 and x∈B
=>x∈Ai and x∈B (for i = 1,2,3)

ここは、x∈Ai(for i = 1,2,3) and x∈B
だと思うのですが上記のようにできる理由は何でしょうか?
550132人目の素数さん:2012/01/30(月) 03:16:40.87
記号遊びしてないで日本語の勉強でもしたらいいと思うよ
551132人目の素数さん:2012/01/30(月) 03:17:05.45
x∈A1 and x∈A2 and x∈A3 and x∈B
=>x∈A1 and x∈B
同じような事を3回やっただけ
552543:2012/01/31(火) 13:03:34.19 ID:W+xAE1pQ
>>551
再びありがとうございます!
同じことを繰り返すというのは盲点でした。
553名無しさん:2012/01/31(火) 14:32:14.29 ID:xmytrYFE
こんにちは。
わからないことがあります。
教えてください。
10/20=0.5です
20がわからない時、知りたい時は、どうやって、答えを出す計算が、ありますか?
よろしくお願いします。
554名無しさん:2012/01/31(火) 14:53:46.57 ID:???
10/0.5
555名無しさん:2012/01/31(火) 14:58:38.83 ID:xmytrYFE
554の人。教えてくれて、ありがとうございました。
とてもよくわかりました。
556名無しさん:2012/01/31(火) 15:49:48.78 ID:???
当たる確率が1%のくじを100回引いて一回でも当たる確率ってなんですか?
当たる確率が1%だから100回引いたら一回は確実に当たると思うので100%だと思ったのですが…
557名無しさん:2012/01/31(火) 16:17:32.26 ID:???
>>556
五分五分の勝負を二回やったら連敗は絶対にしない?
558名無しさん:2012/01/31(火) 17:24:41.49 ID:???
文章問題だが
一辺2cmの正四面体がある
@この立体に内接する球の半径を求めよ
Aこの立体に外接する球の半径を求めよ
うろ覚えだから日本語おかしいかもしれんが頼んます
559556:2012/01/31(火) 17:59:08.03 ID:???
>>557
それは絶対じゃないですね…
確率って難しいですね
560名無しさん:2012/01/31(火) 18:00:10.15 ID:???
>>558
wikipedia、正四面体のページ
561名無しさん:2012/01/31(火) 18:00:58.24 ID:???
>>560
おおスマン
562名無しさん:2012/01/31(火) 19:24:43.89 ID:Xdzro9mC
s∈Q ∧ s^2<2 と仮定したとき、
∃t∈Q s^2<t^2<2 を証明したくて、

具体的に、
(2-s^2)/2 > 1/(n^2), (2-s^2)/(4s) > 1/m を満たす n,m∈N, n,m>=1 を取り、
Min{1/n, 1/m}=c t=s+c と置けば良いとあるのだが、
どのようにしてこのような(アルゴリズム?)やり方が出てきたのかが分かりません。
教えて下さい。
563名無しさん:2012/01/31(火) 20:59:10.27 ID:9nNP5MeH
スレ違いならすいません
これはある数値の表です
ちなみに作ったのも張ったのも私ではないです
これは第三者に見せても表として機能しますか?
某スレでこれが張られた時に荒れたので聞きに来ました
Dの列が-2+8=10となるのはおかしいだろ馬鹿、という理由で変らしいです
私にはDの列は10-2=8と見るのが普通だと思うのですがどうですか?

A     -2
B     -8
C     0
D 10 -2 8
E 9 -3 6
F 10 -1 9
G     -7
564名無しさん:2012/01/31(火) 21:02:56.22 ID:???
下手くそが手を加えてしまってズレまくってますがもっと綺麗な表として見てくれたら助かります
565名無しさん:2012/01/31(火) 21:05:34.30 ID:Rgd7ATxi
超能力の試験か何かでしょうか?
566名無しさん:2012/01/31(火) 21:09:10.37 ID:QOjeZjSl
黒魔術
567名無しさん:2012/01/31(火) 21:26:57.09 ID:???
>>563
マス割りもしてないし見た目汚いしそうなりますよね…
同一スレで「表にしたいんなら8(10−2)って書け」というレスがあったんですがこれはどうですか?
私には解が64の計算式にしか見えません
568名無しさん:2012/01/31(火) 21:46:42.48 ID:???
エスパーさんの降臨を求む
569名無しさん:2012/01/31(火) 22:01:17.77 ID:???
どこのスレ?
570名無しさん:2012/01/31(火) 23:26:04.20 ID:???
>>569
ここです
スレ汚して申し訳ありませんでした
ttp://kohada.2ch.net/test/read.cgi/csaloon/1327861864/
571名無しさん:2012/02/01(水) 00:12:23.12 ID:???
そのスレ荒れすぎてて何がなんだか
572名無しさん:2012/02/01(水) 14:05:04.05 ID:???
表の定義とは?
総計は右端でないとダメとかあんのか
573132人目の素数さん:2012/02/01(水) 16:04:37.04
「閏年ならば、夏季オリンピックがある」は真だが、
「夏季オリンピックがある年は、閏年である」は真か?
574132人目の素数さん:2012/02/01(水) 17:16:20.13
近代オリンピック始まってからグレゴリー暦の100年に一回閏年抜く
ルールの年に一度も掛かってないからな。2000年は100年ルールの
400年に一度の例外だったし。

2100年にならんとわからんが、夏季オリンピックを8年飛ばすとは思えん。
575132人目の素数さん:2012/02/01(水) 19:23:08.09
自然数の定義についてわからないのですが、ご回答頂ければ幸いです。

ひとつめ
a≠0ならばb+1=aとなるbが存在することを証明せよ。

ただし使う定理は帰納法と0と1があること、a+0=aであることのみ。

ふたつめ
積の簡約法則を証明せよ。
c≠0 かつ ac=bc ならばa=b

当方数学初心者なのですが、考えれば考えるほどこんがらがってしまって困っています。
よろしくご教授お願い致します。
576132人目の素数さん:2012/02/01(水) 20:14:29.77
同値関係における対称律の必要性について教えて下さいm(_ _)m

x~y⇒y~x
X~Y、Y~Z⇒X~Z
より、x~x

とはならない理由です
577132人目の素数さん:2012/02/01(水) 20:33:32.85
すいません、>>576は解決しました
578132人目の素数さん:2012/02/01(水) 21:08:10.11
>>575
明日テストなんです(;_;)なんとかこれだけでも理解したいのですが、、うう
579132人目の素数さん:2012/02/01(水) 21:52:27.80
>>575
帰納法の第一段
a=1のとき、規定により0+1=1であるからb=0とすれば、b+1=aである。
即ちa=1のときb+1=aとなるbが存在する。
帰納法の第二段
a=kのとき、b+1=kとなるbが存在するとすれば
k+1の項kに帰納法の仮定を適用すればk+1=(b+1)+1である。
ここでb+1を改めてbとおくことにすればb+1=k+1である。
即ち、a=kのときb+1=aとなるbが存在するとすればb+1=a+1となるbが存在することが示せた。
以上から数学的に帰納法により、任意のa(≠0)に対しb+1=aとなるbが存在する。
580132人目の素数さん:2012/02/01(水) 22:12:16.09
>>579
超ありがとうございます!徹夜で理解します!!!本当にありがとうございます!
581132人目の素数さん:2012/02/01(水) 22:13:38.54
>ここでb+1を改めてbとおくことにすればb+1=k+1である。
即ち、a=kのときb+1=aとなるbが存在するとすればb+1=a+1となるbが存在することが示せた。

これがオカシイと思えるのは俺だけ?
b+1をbと置きなおすってことは、b∈N⇒b+1∈N っての暗に使ってない?
582132人目の素数さん:2012/02/01(水) 22:19:48.54
餡に、って、「次の数」は自然数だろ。
583132人目の素数さん:2012/02/01(水) 22:21:09.31
「↑」の右半分みたいな記号、なんて呼ぶんですか。

fのcへの制限f↑cみたいな
584132人目の素数さん:2012/02/02(木) 01:20:48.28
>>581
うん、多分お前だけ。
585132人目の素数さん:2012/02/02(木) 14:54:52.52
>>573
どちらも偽。暦によって閏年の定義は違うし、グレゴリオ暦に限っても
1892年以前の閏年には夏季オリンピックはなかった
1900年は閏年ではないが夏季オリンピックがあった
586132人目の素数さん:2012/02/02(木) 20:13:11.37
電気工事しで質問した者です 教えてくれた方ありがとうございます 遅くなってしまい申し訳ないです(汗
587132人目の素数さん:2012/02/03(金) 20:22:17.91
フラクトゥールを手書きする時はどう書けば良いですか?
588132人目の素数さん:2012/02/05(日) 10:25:50.63
589132人目の素数さん:2012/02/05(日) 13:47:01.02
>>562
t=s+cと、sを補正する方向を探るのは自然な発想
t^2=s^2+2sc+c^2なので、
(1) 2sc<(2-s^2)/2
(2) c^2<(2-s^2)/2
を両方満たすように正の有理数cを決められれば、tは有理数でありs^2<t^2<2
590132人目の素数さん:2012/02/06(月) 00:35:40.16
多項式どうしの割り算で着目する文字によって答えが変わることがあるというやつなんですが、
これってどういうことなのでしょうか?
式自体は変わらないのに答えが変わるというのがよく分からないのですが・・
どなたか¥教えていただけますでしょうか
591KingMathematician ◆LoZDre77j4i1 :2012/02/06(月) 00:44:27.04
Re:>>590 一部の例外を除き,剰余類は一元集合にはならない.
592132人目の素数さん:2012/02/06(月) 00:49:35.16
そういう話じゃないんでね?
593132人目の素数さん:2012/02/06(月) 01:28:33.86
>>590
自然数同士の余りのある割り算の場合、
a÷b=c…dとするとdはb未満の非負整数と考えることでc,dは一意に定まる。

1変数の多項式同士の余りのある割り算の場合、
AをBで割った商がCで余りがDとするとき、
Dの次数はBの次数より小さいと考えることで、C,Dは一意に定まる。

しかし、A,Bがx,yの2変数からなる多項式だとすると、
xに着目した場合はDのxの次数をBのxの次数より小さくしなければならず、
yに着目した場合はDのyの次数をBのyの次数より小さくしなければならないので
結果が異なるのは当然。
594590:2012/02/06(月) 12:32:24.59
分かりました。
出た答えの変数に実際に数を代入してみると、余りがまだ割れることがありますね。
それで割れなくなるまで計算してみると結局着目した変数が違っても結果が同じになりました。
代入後の式の値が同じなのであれば納得です。
レスいただいた方ありがとうございました。
595猫は本書く派ではない ◆MuKUnGPXAY :2012/02/06(月) 12:37:47.49
>>591
コラ、低能は黙れや。



>591 名前:KingMathematician ◆LoZDre77j4i1 :2012/02/06(月) 00:44:27.04
> Re:>>590 一部の例外を除き,剰余類は一元集合にはならない.
>
596132人目の素数さん:2012/02/06(月) 15:25:56.64
a=b ⇒ ac=bc

これを教えて下さい
597132人目の素数さん:2012/02/06(月) 15:44:15.41
acにa=bを代入するんじゃね
598132人目の素数さん:2012/02/06(月) 20:51:38.43
(b+x)^2≦a となるようにxをとるときに、x=Min{b,(a-b^2)/3b}

とありましたが、このMin{b,(a-b^2)/3b}はどうやってつくったのですか。
思考回路教えてほしいです。
599132人目の素数さん:2012/02/06(月) 20:58:52.41
書いてあるだろ。
600132人目の素数さん:2012/02/06(月) 21:11:03.40
>>599そういう啓発的なやつじゃなくて、詳細な感じのやつがいいです
601132人目の素数さん:2012/02/06(月) 21:13:49.69
お前がまずお前の脳内を詳細にかけ
602132人目の素数さん:2012/02/06(月) 21:18:24.69
それが出てきたところに書いてあることが成り立つように作ったんだよ。
603132人目の素数さん:2012/02/06(月) 21:28:11.02
>>598
-b-√a≦x≦-b+√aを満たすxのなかから
値がMin{b,(a-b^2)/3b}となるようなものを取る、といっているのだから、
答えて貰うには情報が足りない。
あえて書けば 4b^2=aが満たされているときに何かが起こっているんじゃないの?
604132人目の素数さん:2012/02/06(月) 21:36:45.13
正の数aに対してb^2<aとなる正の数bがあるとき
(b+x)^2=b^2+(2b+x)x<=b^2+3bx<=b^2+(a-b^2)=a
とやってbより大きいz^2<=aとなるzがあることを示そうとしてるんでしょ。
605132人目の素数さん:2012/02/06(月) 21:48:41.14
すみませんでした。詳細に書きます

A={x∈X|x≧0,x^2≦a}, b=supA⇒b^2=a

の証明において、b^2<aならばε=Min{b,(a-b^2)/3b}とおいて
(b+ε)^2≦aとなるから、b<b+ε∈Aで矛盾

という流れの中に出てくるものです

>>604その通りです!
606132人目の素数さん:2012/02/06(月) 22:24:51.18
悪漢に追いかけられている人たちが橋を渡る。
橋はもろく、人間を2人までしか支えられない。
当たりは暗く、一行は灯りをひとつ持っており
灯り無しでは渡れない。

また、2人で渡るには一緒にわたる必要があり
かつ、渡るのが遅いものに合わせなければならない。

渡るのは、男ー5分 友人ー10分 女ー20分 老人ー25分

悪漢は1時間でこの橋に到着する。1時間以内に全員が渡るにはどうすればいいか

いまいち確信が持てない。誰か〜
607132人目の素数さん:2012/02/06(月) 22:38:09.40
確信が持てないって何
608132人目の素数さん:2012/02/06(月) 22:40:35.86
反則みたいな答えになったから
609132人目の素数さん:2012/02/06(月) 22:45:34.37
何が反則?
610132人目の素数さん:2012/02/06(月) 22:52:28.84
まず、男と友人が一緒にわたって5分たつと
橋の中央にいることになる。ここからが反則のようなんだけど
橋の中央に男を置き、灯りを男に渡して友人をそのまま渡すという行為

橋の中央に灯りをもった誰かがいればほかの人も渡れる・・・反則?
611132人目の素数さん:2012/02/06(月) 22:55:00.15
-> 5,10.
<- 5(10).
-> 20,25.
<- 10(5).
-> 5,10.
612132人目の素数さん:2012/02/06(月) 22:55:15.00
そんなん正解なわけないやろアホちゃうか
613132人目の素数さん:2012/02/06(月) 22:56:11.59
どう見ても正解だ
614132人目の素数さん:2012/02/06(月) 23:02:43.42
>>611
最初男と友人がわたって、次は?
615132人目の素数さん:2012/02/06(月) 23:16:23.95
これで正解?
616132人目の素数さん:2012/02/06(月) 23:31:22.41
>>7の問題って自分から見える同僚の番号を伏せといても答え1つになるよね、つまり
【基本問題】
あるところに、3人の死刑囚がいた
そこへ看守がやってきて、それぞれの囚人の額にスタンプを押して言った

「お前らの額に一つずつ、1から5までの違う数字のスタンプを押した。
 今、お前らには、自分の数字以外のお互いの数字を見えているわけだが・・・
 そこで、自分の額に書いてある数字が、お前ら3人の中で何番目に大きいかを当ててみろ。
 見事に当てたら、釈放してやる。
 ・・・ただし、誰か一言でも発してみろ、即座に3人とも死刑執行だ。」

今、自分の目から見えるのは、二人の同僚の額に書いてある数字「●」と「●」だ・・・

そのまましばらく、誰も動きが無い沈黙の後俺は看守に歩み寄り、
自分がこの3人の中で何番目に大きいかと、数字そのものを言い当て、自由を手にした。
するとその様子を見た仲間も即座に俺に続いてズバリ当て、無事解放されたようだ

さて、自分の数字とその根拠は?
617132人目の素数さん:2012/02/07(火) 12:03:47.81
>>616
>>7と同じで2と4を見て3と答えた。
それ以外は、即座にわかるか、他の2人のどちらかが先に答えることになる。
618132人目の素数さん:2012/02/07(火) 14:40:19.63
ほぼ等辺の三角形の面積をsinとcosを使って
求めるために角度は20秒の測定誤差
619132人目の素数さん:2012/02/07(火) 20:34:33.88
>>616
元の問題からしておかしくね?
分かったら1分以内に当てないと死刑、くらいはつけてあるとする。

432なら、4のやつは
「自分が1なら3の奴がすぐわかる。そうでないのだから5か4で自分が1番大きい」
と(3の奴が分かったかどうかに関係なく)わかる。2の奴も同様。
だから、3の奴の様子をみてわかった、というのは変で、出題1分後に3人ほぼ同時にわかったといった、だろう。

その上で、532,431の可能性もある(これら以外の場合は誰かが直ちにわかる)。
だから、自分の数字は何かという問題の答は3。

別解:解がただ1つあるなら対称性から3でなければならない。
620132人目の素数さん:2012/02/07(火) 20:39:00.41
621132人目の素数さん:2012/02/07(火) 20:44:11.26
>>620
何を堂々とマルチしてんだよ
622132人目の素数さん:2012/02/07(火) 21:00:15.47
9人を3つのグループに分ける方法を考える。
 (i)4人、3人、2人の3グループに分ける
 (ii)3人、3人3人の3グループに分ける
623132人目の素数さん:2012/02/07(火) 21:55:56.36
>>614
戻る時には時間が掛からないようにするのが味噌か。
624132人目の素数さん:2012/02/07(火) 23:00:06.50
A,B2つのさいころを同時に1回投げ、出た目をそれぞれa,bとする。

(1) A,Bの目の出方について

|a - 3|+|b - 3|=3となるのは(ウエ)通りある。

どのように計算すればいいですか?
625132人目の素数さん:2012/02/07(火) 23:06:57.20
>>624
くそマルチ
626132人目の素数さん:2012/02/07(火) 23:43:46.85
-7-8×(-2)=
中学生の問題なんだけど教えていただけますか?
627132人目の素数さん:2012/02/08(水) 03:13:43.50
>>605
(b+ε)^2 = b^2 + 2bε + ε^2 = b^2 + (2b+ε)ε
だから、もし0<ε≦bなら (b+ε)^2 ≦ b^2 +3bε
さらにε ≦ (a-b^2)/3b なら(b+ε)^2 ≦ a
ということで、ほとんど >>604 が書いたまんま

ていうかXが何なのかわからないとb+εがXの元かどうかわからないし、bが0でないことに言及しないのかよ全然詳細に書いてねーじゃねーか
628132人目の素数さん:2012/02/08(水) 20:30:49.51
一辺の長さが1である正方形ABCDを点Aを中心として点D側に角θ(0゜<θ<90°)だけ回転して、正方形AB'C'D'に移す。
C DとB'C'の交点をE、B'を通りBCと平行な直線とAB、DCとの交点をそれぞれF、Gとする。

また△AB'Dの面積をS、△B'EDの面積をTとおく。



(1)Sをcosθを用いて表せ

(2)B'GとB'Eに長さをそれぞれθを用いて表せ

(3)Tをθを用いて表せ

(4)S=3Tとなるときのθの値を求めよ

(5)S+T=1/2となるときのsinθの値を求めよ
629132人目の素数さん:2012/02/08(水) 21:01:56.48
たぶんマルチw
630132人目の素数さん:2012/02/08(水) 22:19:25.39
A∈M3(R ) で A^(2) ≠ E かつ A^(3) = Eとなるものを1つ具体的に求めよ
631132人目の素数さん:2012/02/08(水) 22:26:50.35
[a 0 0]
[0 1 0]
[0 0 1]
a = exp(i*2π/3)
632132人目の素数さん:2012/02/08(水) 22:27:53.05
あ、Rかw
ちょっといじって回転行列にすればおk
633132人目の素数さん:2012/02/12(日) 19:00:35.10
ε-δ論法に関する下の例題で、最後に「これらをまとめて〜〜を得る。」という記述がありますが、
具体的に、どの式をどのようにまとめると|x^2-1|<δ(δ+2)が出てくるのでしょうか?

関数f(x)=x^2のx=1における連続性を確かめる。
正数εを一つ定め、任意のxについて
  |x-1|<δ, |x^2-1|<ε
を満足するδを見附ければよい。絶対値の性質より
  |x^2-1|=|(x+1)(x-1)|=|x+1||x-1|
となる。ここで三角不等式:
  |a+b|≦|a|+|b|, |a|-|b|≦|a-b|
を用いて上式を変形する。先ず
  |x+1|≦|x|+1, |x|-1≦|x-1|
であり、x-1<δを用いて
  |x|-1<δ
を得る。上式の両辺に2を加えて
  |x|+1<δ+2
となる。これらをまとめて
  |x^2-1|<δ(δ+2)
を得る。……
634132人目の素数さん:2012/02/12(日) 20:00:04.43
>>633
|x^2-1|はどこにある?
635132人目の素数さん:2012/02/12(日) 20:24:49.48
>>633
あまりいい解答じゃないなー
636132人目の素数さん:2012/02/13(月) 04:55:25.36
>>633
回り道省くと
|x+1| = |(x-1)+2| ≦ |x-1| + |2| ≦ δ+2
だから
|x^2-1| = |x-1| |x+1| ≦ δ(δ+2)
637132人目の素数さん:2012/02/13(月) 04:57:39.43
≦ のとこ適当に < に直しといて
638132人目の素数さん:2012/02/13(月) 08:50:50.33
>>634-637
ありがとうございます
レスを参考にしつつ、解答では下のようなことが言いたかったのだろうと一応納得しました
もっとも、これを式番号も示さずに「まとめて」とだけ言うのはやや無理な気はしますが

|x^2-1|=|x-1||x+1|
<δ|x+1|   (∵|x-1|<δ)
<δ(δ+2)  (∵|x+1|≦|x|+1 かつ |x|+1<δ+2)
639635:2012/02/13(月) 14:37:05.55
>>638
無理っていうほどの感じはないな
「あまりいい解答じゃない」って書いたのは、論理の流れがあまりよくないからだよ

>>633で十分だし、ああいう書き方は実際によくある
でも、初学者が手本にするにはちょっとなー、っていう感じ
採点していると、>>633みたいなのを真似して失敗してる答案が非常に多い
640132人目の素数さん:2012/02/13(月) 15:55:54.80
δ=min{1-√(1-ε),√(1+ε)-1}
のような回答を想像したが
641132人目の素数さん:2012/02/13(月) 16:10:47.13
>>640
√ってx^2の連続性無しで定義できたっけか
δ=min{1,ε/3}くらいが普通じゃないか?
642132人目の素数さん:2012/02/13(月) 16:13:48.11
何言ってんのこいつ
643132人目の素数さん:2012/02/13(月) 16:14:22.45
二次方程式を解いているだけとおもうが
644132人目の素数さん:2012/02/13(月) 16:44:40.27
>>643
「任意の非負数aに対してr^2=aをみたす非負数rがただ一つ存在する」というのも証明が必要だよ
それをどう証明するか、が問題で、x^2 (x>0) の連続性と単調性と非有界性から証明するなら、ここで使うわけにはいかない
645132人目の素数さん:2012/02/13(月) 17:17:06.04
>>644
ま、俺の問題ではなのでどうでもいい
646132人目の素数さん:2012/02/13(月) 17:19:51.98
といいながら、ε/3はどこから出てきた?
647132人目の素数さん:2012/02/13(月) 20:37:02.35
>>644
つっこみどころ満載
648132人目の素数さん:2012/02/13(月) 20:54:22.29
>>647
じゃあ突っ込め
649132人目の素数さん:2012/02/14(火) 01:38:03.19
>>646
δをかならず1以下になるように定めればδ+2≦3
最初からこうしてδをminで制限してしまうのは、よくある方法
650132人目の素数さん:2012/02/14(火) 02:38:00.14
距離空間ですから
651132人目の素数さん:2012/02/15(水) 15:35:00.68
zとaが複素数の時z^aが微分可能かどうか、可能なら導関数の求め方を教えて下さい
652132人目の素数さん:2012/02/15(水) 23:17:06.21
>>651
z^aの定義は知ってるの?
653132人目の素数さん:2012/02/15(水) 23:26:48.98
>>652
e^(a(log|z|+iArgz)) でしょうか?
654132人目の素数さん:2012/02/16(木) 00:47:02.93
>>653
そう
それが微分可能かどうか(あるいは、そもそも連続かどうか)考える
655132人目の素数さん:2012/02/16(木) 00:54:05.89
>>654
そうすると途中で|z|の微分が出てきて上手くいきません
どう考えたらうまくいくのでしょうか?
656132人目の素数さん:2012/02/16(木) 01:59:57.03
>>655
w^a - z^a = z^a (w^a / z^a - 1)

w^a / z^a = e^( a(log |w| + i Arg w) - a(log |z| + i Arg z) )
= e^( a( log |w| - log |z| + i(Arg w - Arg z) ) )

w→zのとき、zの値とwの近づき方によってはArg w - Arg zは0にならずに2πになる
657132人目の素数さん:2012/02/16(木) 20:00:38.03
>>656
これを使えばz^aの導関数を導けるということですか?
658132人目の素数さん:2012/02/16(木) 21:57:54.69
>>657
logの導関数は計算したことある?
659132人目の素数さん:2012/02/16(木) 22:46:59.08
>>658
log(x)の導関数は1/xですよね?
660132人目の素数さん:2012/02/17(金) 09:16:40.91
噛み合ってないな。
有無を尋ねてるのに答えはこうですよねって…
661132人目の素数さん:2012/02/17(金) 14:28:30.45
つまり、logxの導関数を求めるのと同じ手法を使うということですか?
662132人目の素数さん:2012/02/17(金) 17:41:58.93
微分が自明じゃねーって事にも気付いてないなら無理ってこと。
663132人目の素数さん:2012/02/17(金) 17:48:03.45
z^aの微分が自明なのですか?
どうしてそう言えるのですか?
664132人目の素数さん:2012/02/17(金) 18:04:06.97
自明じゃないことに気づいていない、と書いてある。
665132人目の素数さん:2012/02/18(土) 16:47:17.55
y=x^2の上の3点を結ぶ三角形と同じ面積の座標を求める問題は台形の公式か
頂点から平行線と放物線の座標を求めるか
直交する直線に放物線の座標を入れて交点までの距離を出すか。
中学の模範解答はどの方式がいいか?
666132人目の素数さん:2012/02/18(土) 16:53:50.26
>>y=x^2の上の3点を結ぶ三角形と同じ面積の座標を求める問題
667132人目の素数さん:2012/02/18(土) 16:54:47.04
好きにしろ
668132人目の素数さん:2012/02/18(土) 17:08:50.51
放物線上の2点は共通でその他の点で放物線上にある点の座標ね。
あと頂点は共通の2点より下にある放物線上の点。
669132人目の素数さん:2012/02/18(土) 17:16:33.20
好きにしろ
670132人目の素数さん:2012/02/18(土) 17:27:44.80
>>668
そういう質問者の解釈の入った戯言はどうでもいいからさ、問題文全文を正確に書き写してみな。
671132人目の素数さん:2012/02/18(土) 17:30:12.00
>>668
共通の2頂点を結ぶ直線と平行であって、3番目の頂点を通る直線と放物線のもう一つの交点を求める。
672あぼーん:あぼーん
あぼーん
673132人目の素数さん:2012/02/19(日) 23:29:39.52
テトリスの全部のピース1つずつ使って4*7マス全部埋めるにはどう配置すればいいか
674132人目の素数さん:2012/02/20(月) 00:06:51.37
2×1のピースで埋める問題の市松模様に塗り分ける考え方をピースの方に適用させると
T字だけ1:3(他はすべて2:2)になるので不可能、という結果になった
675132人目の素数さん:2012/02/22(水) 22:41:25.48
平面上にn個の点からなる集合Aが与えられているとする。
Aのどの2点の距離も1より小さければ、Aのどの点も内部に含む
半径√(2)/2の円が存在することを示せ。
676132人目の素数さん:2012/02/23(木) 00:05:58.17
>>675

 x座標が最小の点をP1 (x1, *) 最大の点をP2 (x2, *) とする。
 |x1-x2| ≦ P1P2 < 1,

 y座標が最小の点をQ1 (*, y1) 最大の点をQ2 (*, y2) とする。
 |y1-y2| ≦ Q1Q2 < 1,

∴ n個の点はすべて1×1の正方形の内部にある。


 y座標の最大値y1, 最小値y2,
とする。
677132人目の素数さん:2012/02/23(木) 04:38:23.80
どなたかお分かりになるかたいらっしゃいましたらよろしくお願いします。

次の条件を満たす放物線をグラフとする2次関数を求めよ。
頂点が(2,1)で、点(3,2)を通る。

答えは解っているのですが途中式が解らず困っていますどうかよろしくお願いします
678132人目の素数さん:2012/02/23(木) 04:41:22.25
高校1年1学期の内容だろ?教科書読めよ
679132人目の素数さん:2012/02/23(木) 04:43:27.03
(´;ω;`)
680132人目の素数さん:2012/02/23(木) 04:45:36.38
>>677
色んなスレに問題投下して丸投げしてるな
681132人目の素数さん:2012/02/23(木) 12:29:24.61
どうぞ

任意の正整数kに対してS={2^(1)-1,・・・,2^(2k)-1)}の元の少なくとも一つは
2k+1で割り切れることを示せ。
682132人目の素数さん:2012/02/23(木) 22:12:56.81
>>681
どうも
683132人目の素数さん:2012/02/23(木) 23:12:39.62
>>681

背理法による。
Sの要素を2k+1で割った余りは、いずれも0でないとする。
それらは 2k でもない。(題意から)
∴ 余りは {1,2,・・・・,2k-1} のいずれか。
Sの要素は2k個あるから、鳩ノ巣原理(引出し論法)から、
いずれか2個の余りが一致する。
ある i<j に対して
 0 ≡ (2^i - 1) - (2^j - 1) = 2^i - 2^j = (2^j){2^(i-j) - 1}, (mod 2k+1)
 0 ≡ 2^(i-j) - 1 ∈ S  (mod 2k+1)  (← 1≦ i-j ≦ 2k-1)
これは仮定に反する。(終)

 
>>671
 y = a(x-2)^2 +1, (a≠0)
と桶。
684132人目の素数さん:2012/02/23(木) 23:43:52.73
〔類題〕
d次元空間にn個の点からなる集合Aが与えられているとする。
Aのどの2点の距離も1より小さければ、Aのどの点も内部に含む
半径(√d)/2の超球面が存在することを示せ。
685132人目の素数さん:2012/02/24(金) 00:04:32.12
>>683
> >>671
>  y = a(x-2)^2 +1, (a≠0)
> と桶。
これは何?
686132人目の素数さん:2012/02/24(金) 01:59:11.28
>>677
 y = a(x-2)^2 +1, (a≠0)
と桶。
これが点(3,2) をとおるので...
687132人目の素数さん:2012/02/24(金) 02:37:09.45
>>681
2k+1は2k+1で割り切れる
688132人目の素数さん:2012/02/24(金) 22:12:45.70
誰か途中の計算式も含めて教えて

@2(x+1)(x-2)=0

A2(x+1)^2=3

どなたかお願いします
689132人目の素数さん:2012/02/24(金) 22:17:28.66
@とか何を計算すれば満足するんだろうか
690132人目の素数さん:2012/02/24(金) 22:25:02.62
あちこちマルチすんなボケ
691132人目の素数さん:2012/02/28(火) 20:04:18.74
2次式の因数分解 数U

ax^2+bx+c=a(x^2+b/ax+a/c)
=a{x^2-(α+β)+αβ}
=〜

という公式を導くための途中式があるのですが、
-(α+β) のマイナスがよくわかりません。

なぜマイナスを付けるのですか?

692132人目の素数さん:2012/02/28(火) 20:49:35.10
>>691
2解がα、βとなる2次方程式が
a(x-α)(x-β)=0
と書けるから
693132人目の素数さん:2012/02/28(火) 20:52:28.02
て、マルチかよ
694132人目の素数さん:2012/02/28(火) 21:00:40.18
とりぷるち
695132人目の素数さん:2012/03/04(日) 08:38:24.62
とんからり


って何ですか?
696132人目の素数さん:2012/03/04(日) 11:12:27.39
>>695
素数の歌
697132人目の素数さん:2012/03/05(月) 03:58:28.51
>>695

大魔王さんが怪物を退治してる音ぢゃね?

http://www.youtube.com/watch?v=ioucgknkuH4
http://www.youtube.com/watch?v=Ez-duJ2kVGU (歌詞)
698132人目の素数さん:2012/03/08(木) 07:17:26.49
iPhone4 は300超で Retina を名乗っていたのですがPPI264でRetiaを名乗れる理屈を教えて下さい。

a = 2 tan(-1乗)* h / 2d らしいのです。ちなみに成績は2です。
699132人目の素数さん:2012/03/08(木) 14:56:38.44
国語の成績は1か
700132人目の素数さん:2012/03/08(木) 15:22:42.58
二次方程式 X^2-2(a-1)x-a+3=0 が重解をもつように、定数aの値をもとめ、その重解を求めよ。

わからないです。誰か解いてください・・・
701132人目の素数さん:2012/03/08(木) 16:47:45.59
その問題が分からないとなると、「判別式とは何か?」というところの説明から始めないといけないのか
ちょっとしんどいな
702132人目の素数さん:2012/03/08(木) 17:23:05.15
>>700
与えられた方程式の解は
 x=a-1±√(a^2-a-2)
  =a-1+√(a^2-a-2), a-1-√(a^2-a-2)
となるが、この2解が一致するとき、与えられた方程式は重解をもつことになる。よって、
 a-1+√(a^2-a-2)=a-1-√(a^2-a-2)
なるaが求めるべきaである。整理して、
 2√(a^2-a-2)=0
 √(a^2-a-2)=0
 a^2-a-2=0
 ∴a=-1, 2 [答]
上の変形からも見て取れるように、今求めたaは√(a^2-a-2)=0を満たす。これを利用して重解を求めると、
 a=-1のとき、x=(-1)-1=-2 [答]
 a=2のとき、x=2-1=1 [答]

ちなみに、教科書の判別式のあたりをしっかり読めばもっと簡潔な解答が出来る。
703132人目の素数さん:2012/03/08(木) 19:24:17.08
(1) x^2-3|x-1|=7

(2)|2x-1|>1

(3)2|x+1|<x+2

優しいひと解答おねがいします。
704132人目の素数さん:2012/03/08(木) 19:27:04.50
|A|=B ⇔A=±B
|A|<B ⇔-B<A<BかつB>0
705132人目の素数さん:2012/03/08(木) 19:29:49.88
Q.判別式とは何か?
A.実係数2次方程式の場合はルートの中の式のことだ。
706132人目の素数さん:2012/03/08(木) 19:30:40.11
ルートというのは解の公式のルート
707132人目の素数さん:2012/03/08(木) 19:36:33.96
Z(t)=(2z/a)t  (0≦t≦a/2の時)
   2z-(2z/a)t  (a/2≦t≦aの時)

のフーリエ変換を計算して教えてください優しい人
708132人目の素数さん:2012/03/08(木) 19:43:33.48
>>703
くそまるち
709132人目の素数さん:2012/03/08(木) 19:50:51.77
>>707
アホマルチ
710132人目の素数さん:2012/03/08(木) 19:54:12.16
マルチマルチうるせえ奴だ

それを言うためにこんなとこ来てんのかwwww
711132人目の素数さん:2012/03/08(木) 19:56:24.09
もちろんそうよ
712132人目の素数さん:2012/03/08(木) 19:57:44.13
暇人乙wwwwwwwwwwwwwww
もっと有用な時間の使い方白
713132人目の素数さん:2012/03/08(木) 20:16:27.42
まともな質問には回答することもあるでよ
714132人目の素数さん:2012/03/08(木) 21:30:16.43
>>705
根の差積の平方のことだよバカ。
715132人目の素数さん:2012/03/08(木) 22:03:35.76
よくも東大生にバカとほざいたな?
おまえはどこ大のバカだ?
716猫vs運営 ◆MuKUnGPXAY :2012/03/08(木) 22:05:56.48
>>715
東大生でも馬鹿が居るのは衆知の事実。従ってオマエは文句を言えない。


717132人目の素数さん:2012/03/08(木) 22:08:32.53
>>715がFランだと見抜ける程度にそこそこの大。
718132人目の素数さん:2012/03/08(木) 22:10:54.50
今夜はカツ丼とサラダを食べました。
みんなの晩御飯は何でしたか?
719猫vs運営 ◆MuKUnGPXAY :2012/03/08(木) 22:13:02.09
>>715
オマエのそういう考え方は極めて卑しい。もし文句があれば掛かって来いや。
ワシはオマエと戦う準備は何時でもアルのや。エエな。


720猫vs運営 ◆MuKUnGPXAY :2012/03/08(木) 22:15:08.88
>>718
無関係な話をして、話を逸らすなや。思いっきり打ち下すゾ。


721132人目の素数さん:2012/03/08(木) 22:16:14.95
>>718
うちはすき焼きだよ
722猫vs運営 ◆MuKUnGPXAY :2012/03/08(木) 22:17:03.87
>>715
ワシはオマエみたいな考え方の奴が大嫌いなんだヨ。徹底的に潰したる
から出て来いや。


723猫vs運営 ◆MuKUnGPXAY :2012/03/08(木) 22:18:04.15
>>721
コラァ、叩くゾ。


724132人目の素数さん:2012/03/08(木) 22:20:26.78
>>718
偶然だな、うちもすき焼きだったよ!
725猫vs運営 ◆MuKUnGPXAY :2012/03/08(木) 22:21:29.82
名門大学の准教授が、講演ついでの痴漢行脚で逮捕された。

徳島県警阿南署は5日、県迷惑行為防止条例違反容疑で、東京都足立区、筑波
大准教授の増田哲也容疑者(50)を逮捕した。増田容疑者は4日、JR牟岐
線の山河内〜阿南間の列車内で、午後4時20分から5時10分にかけての約
50分間、専門学校生の女性(21)にささやき、言い寄りながら、足や胸を
服の上から触るなどした疑い。女性は阿南駅で降り、通報。届け出を受け、特
徴が似ている男を捜した結果、徳島東署員が翌5日の午前零時25分ごろ、徳
島駅にいた増田容疑者を発見した。
 増田容疑者は「好みのタイプだった」と話し、容疑を認めている。高知大で
講演し、次の講演場所に向かう途中だったと言う。
 筑波大ホームページによると、増田容疑者は大阪大基礎工学部を卒業後、京
大大学院に進学。フランスの数学者、アラン・コンヌ氏に心酔し、フランス留
学で同氏に直接師事、帰国後は筑波大の数理物質科学研究科で数学を専攻して
いた。

ケケケ猫
726132人目の素数さん:2012/03/08(木) 22:22:14.16
砂糖としょうゆで焼いて食べるのがいい
ぐつぐつ煮るのは邪道
727猫vs運営 ◆MuKUnGPXAY :2012/03/08(木) 22:22:43.23
>>724
そういう下らん話をする奴は痛め付けたる。


728猫vs運営 ◆MuKUnGPXAY :2012/03/08(木) 22:23:43.56
>>726
オマエも虐められたいんかァ。


729132人目の素数さん:2012/03/08(木) 22:27:38.55
>>704
なんでB>0が要るの?
730132人目の素数さん:2012/03/08(木) 22:27:46.31
すき焼きは関西風の勝ち
731猫vs運営 ◆MuKUnGPXAY :2012/03/08(木) 22:33:28.68
名門大学の准教授が、講演ついでの痴漢行脚で逮捕された。

徳島県警阿南署は5日、県迷惑行為防止条例違反容疑で、東京都足立区、筑波
大准教授の増田哲也容疑者(50)を逮捕した。増田容疑者は4日、JR牟岐
線の山河内〜阿南間の列車内で、午後4時20分から5時10分にかけての約
50分間、専門学校生の女性(21)にささやき、言い寄りながら、足や胸を
服の上から触るなどした疑い。女性は阿南駅で降り、通報。届け出を受け、特
徴が似ている男を捜した結果、徳島東署員が翌5日の午前零時25分ごろ、徳
島駅にいた増田容疑者を発見した。
 増田容疑者は「好みのタイプだった」と話し、容疑を認めている。高知大で
講演し、次の講演場所に向かう途中だったと言う。
 筑波大ホームページによると、増田容疑者は大阪大基礎工学部を卒業後、京
大大学院に進学。フランスの数学者、アラン・コンヌ氏に心酔し、フランス留
学で同氏に直接師事、帰国後は筑波大の数理物質科学研究科で数学を専攻して
いた。

ケケケ猫
732132人目の素数さん:2012/03/08(木) 22:58:06.77
>>729
B≤0は無意味だから。
733132人目の素数さん:2012/03/08(木) 23:00:48.49
>>732
ああ、分ってない。
734132人目の素数さん:2012/03/09(金) 13:46:59.70
-B<A<Bなら-B<Bだから当然B>0だろう
735132人目の素数さん:2012/03/09(金) 20:46:42.64
数字(0,1,2,3,4,5,6,7,8,9)のみで書き表せる文字列(長さ1以上)のうち、
「2012」を含む文字列はどれくらいの割合で存在しますか?
736132人目の素数さん:2012/03/10(土) 02:41:38.83
それベルトランの逆説的な問題なんじゃなかろうか
737132人目の素数さん:2012/03/13(火) 13:36:06.49
計算を便利にするといわれる自然対数の底eですが、πと違っていまだに便利さがピンときません。
指数・対数の本を見てもいまひとつ…(図解の入門書だったから?)という感じでeが重宝される
理由がわからなかったので、少し難しそうな不思議な数eの物語という本を買いました。
歴史みたいな出だしで読みやすく面白い!買ってよかった!
と思ったのもつかの間で6ページ目で理解できなくなって…そこで質問です。前置き長くてすみません。
-------------- ここから引用 --------------
シュティーフェルは指数が整数の場合しか考えなかったのに対して、ネーピアの考えはそれを連続的な
値域にまで拡張することだった。
彼の考えの道筋は次のようであった: 任意の正の数を、ある与えられた数(後に底と呼ばれる)の累乗
として書くことができるなら、数のかけ算、割り算はその数の指数の足し算、引き算に等価である。
さらに、ある数をn乗する(すなわちn回その数を掛ける)ことは、その指数をn回足す――すなわち
指数をn倍する――ことに等価である。そして、ある数のn乗根を求めることはn回続けて引く――すなわち
nで割る――ことに等価である。
-------------- 引用ここまで --------------
はじめの二つは指数法則の
(A^n)・(A^m)=A^(n+m) と (A^n)/(A^m)=A^(n-m)
のことを書いていて、三つ目に書いてあることは
(A^m)^n【Aのm乗のn乗は】=A^(m+m+…+m)【元からあった指数mをn回足す】=A^(nm)【だから指数はnmになる】
で…たぶんここまでは理解できてると思うのですが。
四つ目の意味がわからなくてモヤモヤしてしまいました。わたしが本の言葉通りにすると
(A^m)【のn乗根は】[n]√(A^m)=A^(-m-m-…-m)【今度はn回引く??】=A^(-nm) 【nで割る形にならない!】
こうなっちゃう!
たぶん(いっこ前に読んだ本の似たような部分から察すると)[n]√(A^m)=A^(m/n) のことだと思うんですけど
『ある数のn乗根を求めることはn回続けて引く』という部分と結びつかなくて理解できなくなってしまいました。
どこで間違ってしまったんでしょうか。面白そうな本なので読み続けたいです。
738132人目の素数さん:2012/03/13(火) 15:13:48.90
数のかけ算{(A^m) * (A^n)}は(=)その数の指数の足し算{A^(m+n)}に等価である。
数の割り算{(A^m) / (A^n)}は(=)その数の指数の足し算{A^(m-n)}に等価である。

ある数をn乗する{(A^m)^n)}ことは(=)、その指数をn回足す―
―すなわち指数をn倍する{A^(m*n)}――ことに等価である。

ある数のn乗根を求めること{(A^m)^n=A^mn}ことは(⇔)、
n回続けて引く(注)[A^0=A^(mn-n-n-...-n),引き算の回数はm回]―
―すなわちnで割る[n√{(A^m)^n}=A^(mn/n)}――ことに等価である

注:割り算を定義する原始的な方法に、0になるまで減算する回数とするものがあるが、
恐らく本来はそのことを言いたかったのだろうと推測する。

だが間違ってても知らね
739132人目の素数さん:2012/03/13(火) 23:59:27.02
数学板の現総ねらーは何人か
740猫vs運営 ◆MuKUnGPXAY :2012/03/14(水) 00:16:45.48
>>739
最後の残党はもう僅かな筈です。


741132人目の素数さん:2012/03/14(水) 01:11:53.89
東亜は一日で一週するから、一人平均3レスするとしてだいたい
340人必要。3,4スレの回転率だと1400人はいることに・・・

ここは十数人かな・・・どうでしょう?
742あぼーん:あぼーん
あぼーん
743132人目の素数さん:2012/03/18(日) 04:51:51.79
行列I=[[0,-1],[1,0]]とするとI^2=-Eとなるということですが、
このIというのは虚数単位のiとまったく同じものなんでしょうか?
それとも単にiと似た性質を持った行列に過ぎないんでしょうか?
744132人目の素数さん:2012/03/18(日) 04:54:11.23
後者
745132人目の素数さん:2012/03/18(日) 12:10:14.51
>>743
Eを単位行列とするとき、
集合{aE+bI|a,b∈R}を複素数{a+ib|a,b∈R}と同一視することができる。
746132人目の素数さん:2012/03/24(土) 11:18:47.29
D^n(fg)=Σ[m=0,n](C(n,m)*D^(n-m)(f)*D^m(g))
を証明するときに、数学的帰納法で上式がn以下の自然数で成り立つとき
D^(n+1)(fg)=D(D^n(fg))
=D(Σ[m=0,n](C(n,m)*D^(n-m)(f)*D^m(g)))
=Σ[m=0,n](C(n,m)*(D^(n+1-m)(f)*D^m(g)+D^(n-m)(f)*D^(m+1)(g)))
って書いてあったんですが微分とΣって勝手に入れ替えていいんですか
747132人目の素数さん:2012/03/24(土) 12:21:54.61
有限和だろ。
748132人目の素数さん:2012/03/24(土) 15:27:19.68
Σというのは、例えばa[0]+a[1]+a[2]+a[3]+...+a[n]をΣ[m=0,n]a[m]と書くと約束しただけだから
(Σ[m=0,n]a[m])' = (a[0]+a[1]+a[2]+a[3]+...+a[n])' = (a[0])'+(a[1])'+(a[2])'+(a[3])'+...+(a[n])'
=Σ[m=0,n](a[m])'
749あぼーん:あぼーん
あぼーん
750132人目の素数さん:2012/03/26(月) 13:58:44.32
さよか
751あぼーん:あぼーん
あぼーん
752あぼーん:あぼーん
あぼーん
753あぼーん:あぼーん
あぼーん
754あぼーん:あぼーん
あぼーん
755あぼーん:あぼーん
あぼーん
756132人目の素数さん:2012/04/02(月) 10:10:20.86
tg
757132人目の素数さん:2012/04/17(火) 12:34:04.59
dr
758あぼーん:あぼーん
あぼーん
759132人目の素数さん:2012/04/19(木) 20:48:49.14
a,bを複素数として、
{exp(a)} {exp(b)}
= Σ(k=0,∞) ( a^k / k ! ) Σ(m=0,∞) ( b^m / m ! ) ・・・(A)
= Σ(n=0,∞) ( 1 / n ! ) Σ(k=0,n) n ! a^k b^(n - k) / { k ! (n - k) ! } ・・・(B)
= Σ(n=0,∞) ( a + b )^n / n ! ・・・(C)
= exp( a + b ) ・・・(D)

という計算をしている本があって、(B)→(C)は二項定理なのでよいのですが
(A)→(B)の変形が分からないのでどう考えればいいのかヒントを下さい。
kをnとおいて、mをn-kとおいているみたいに見えますが・・・。
760132人目の素数さん:2012/04/19(木) 21:28:45.30
>>759
n=k+mとおいて、(k,m)の組み合わせをnの値ごとにまとめた
761132人目の素数さん:2012/04/19(木) 21:39:26.62
なるほど!この段階ですでに組み合わせを考慮してるんですね!
もうちょっと考えてみます。
762132人目の素数さん:2012/04/19(木) 23:31:11.91
ある本に、命題論理の公理として
(A→(B→C))→((A→B)→(A→C))
というのが挙げられていたのですが、私はこれを見てもなんだか納得できませんでした
→を「ならば」と読んで素朴に考えてみても、なんだかあまり「当たり前のこと」には見えず、
なんというか、本当に当たり前な事実(例えばA→Aとか¬¬A→Aみたいなもの)とはひとつ壁を隔てた側にあるようなものに見えるんです
「あくまで公理なんだから頭で納得する必要性は皆無」という向きもあるとは思いますが、
よく分からなくてもとにかく受け入れろ、と言われるのはなんだか気持ち悪いような気がするので……

皆さんは上の論理式を見てすぐに「当たり前だなぁ」と思えますか?
763132人目の素数さん:2012/04/19(木) 23:48:33.71
>>762
見た瞬間にではないけどまあわかるかなあ。
(A→(B→C))かつ((A→B)ならってのと同じだから。
764132人目の素数さん:2012/04/20(金) 01:10:00.37
ABCに具体的なものを入れてみるとか…

(総理大臣が→(露出狂なら→逮捕))→(総理大臣ならば→露出狂)ならば→(総理大臣は→逮捕)

…総理大臣ごめん
765132人目の素数さん:2012/04/20(金) 08:23:27.02
俺も一度日本語にしないとダメだな。
AならばB→Cが成り立つとき、AならばBならAならばCと読むとそのまんまじゃんと思う。
766132人目の素数さん:2012/04/20(金) 14:34:05.56
様々なご意見ありがとうございます
それをもとに考えてみて
A→(B→C)というのは、B→Cが成り立つのにAという前提が必要である、ということを言っていると見て、かの論理式を
(Aであり(B→C)) ならば ((AでありB)→(AでありC))
と見ると、前提であるAをB,Cのそれぞれに分配してやっただけだ、という感じで納得できました
767132人目の素数さん:2012/04/20(金) 19:15:37.13
>>766
> A→(B→C)というのは、B→Cが成り立つのにAという前提が必要である
それは違うのでは?
「Aならば(B→C)」と言っているだけであって、「Aでなければ(B→C)でない」とは言っていない。
Aでないときに(B→C)であっても構わない。
768132人目の素数さん:2012/04/20(金) 19:37:29.51
総理大臣でなくても→(露出狂なら→逮捕かもしれない)
769132人目の素数さん:2012/04/20(金) 20:38:45.61
>>767
あ、Aは偽にもなるってことを忘れてました
とすると「Aであり」という書き方では間違いですね
でも、「A→」を分配する、という感覚で捉えるのはなんとなく分かったと思います
770132人目の素数さん:2012/04/21(土) 10:48:43.85
わかってる気がしない
771132人目の素数さん:2012/04/21(土) 11:13:42.27
>>766
こういう「納得」をしてしまうから、その公理が当たり前とは思えないんだろうね。
772132人目の素数さん:2012/04/21(土) 12:54:59.10
すみません、皆さんの反応が芳しくないので自分の考え方を反省してみたら、
考えていた実例が特殊だったせいで勘違いしてたみたいでした
うーん、どうすればいいんでしょう
773132人目の素数さん:2012/04/21(土) 22:13:02.43
ユークリッドの互除法の証明でつまずいてます

http://www.cwo.zaq.ne.jp/bfaby300/math/gojyo.html
の【互除法の原理の証明】で
r=(a'+b'q)G
r=r'G'
とあらわせるところまではわかりました
ここからどうして G が G' の約数であることが導かれるのかわかりません
G が r の約数であることならわかるんですが
たとえば r = 2*3*5*7 のとき
G = 2*3*7, G' = 2*5*7 のようになることはなぜありえないんですか?
774132人目の素数さん:2012/04/21(土) 22:51:34.92
775132人目の素数さん:2012/04/21(土) 23:03:51.26
>>773
説明がおかしいように思う。

Gはaとbの最大公約数であるから当然bの約数。
r=(a'+b'q)GなのでGはrの約数。
従ってGはbとrの公約数。
bとrの最大公約数がG'であるので、GはG'の約数。

最大公約数と公約数の関係を利用しているのだが、そのサイトの説明ではそのことに言及していない。
776132人目の素数さん:2012/04/22(日) 23:47:26.45
>>774-775
ありがとうございます。とてもわかりやすいです。
777132人目の素数さん:2012/04/23(月) 05:13:42.54
>>772
複雑な事を考えるのに慣れてないみたいだから、経験値を積むこと。
778あぼーん:あぼーん
あぼーん
779132人目の素数さん:2012/04/23(月) 08:44:35.96
>>772
最初のほうできちんと学んでいないのだと思う。
日常の言葉で「AならBである」と言えば、「AでなければBでない」と言う意味を含む場合がある、
というかそのほうが多いくらいだと思うが(そうでなければ「Aなら」と言う意味がない場合が多いから)。
例えば、「明日なら行けます。」は「明日以外は難しい。」と言う意味を含んでいる場合がある。
しかし、論理学での「Aなら」には「Aであるとき」と言う意味しかないので、「Aでないとき」については一切関係がなく、
例えば、常にBが成り立つ場合であっても「AならBである」も真。
必要条件、十分条件は混乱しやすいところなので、ややこしい問題をやる前にきちんと理解すべき。
780阿呆:2012/04/23(月) 12:02:41.63
線型と線形の違いがわかりません
781132人目の素数さん:2012/04/23(月) 16:33:43.55
しつこくてすみません
今調べたらP→QはPが真ならQの真偽と一致して、Pが偽ならQにかかわらず常に真なんですね…
ということは、(A→(B→C))→((A→B)→(A→C))について
Aが真のときは
(真→(B→C))→((真→B)→(真→C))すなわち(B→C)→(B→C)
Aが偽のときは
(偽→(B→C))→((偽→B)→(偽→C))すなわち真→(真→真)
になる、という理解でいいでしょうか…?
782132人目の素数さん:2012/04/23(月) 17:45:11.85
>>781
真偽表を書けばいいんでないの?
783132人目の素数さん:2012/04/23(月) 18:02:27.32
公理なんだろ
784132人目の素数さん:2012/04/23(月) 20:52:09.26
>>781
真偽なんか関係ない
785132人目の素数さん:2012/04/23(月) 21:44:15.55
いやあるだろ
786132人目の素数さん:2012/04/23(月) 21:53:00.78
俺もちょっとややこしく感じるときは真理値表を書き上げるけどな
書けば恒真になることはすぐに確認できる
ぱって理解できる人がうらやましい
787132人目の素数さん:2012/04/23(月) 22:35:50.39
>>782
実は真偽表を書いて確かめることはしたのですが、
でもわざわざ真偽表を持ち出さなければ理解できないようなことが公理として採用されるだろうか、
本来ははもっと当たり前に理解できるものだからこそ公理なのではないのだろうかと思ったんです
781では、私はどうも前提が偽になるときの「ならば」に弱いようなので、
Aが偽になるときを真になるときと分けて考えてみようと思ったのですが、
でもこんな面倒なことやってる人いないですよね…
やはりご指摘の通り、私自身がこのようなことに関する思考に慣れていないからあの公理が納得できなかったのだと思います

一応尤もらしさは真偽表で確かめられたので、とりあえずそれで公理を受け入れることにして、
これから論理式を運用していくことでいつかは「当たり前」と思えるだろう、というぐらいの心構えでいたいと思います
長々と失礼しました
788132人目の素数さん:2012/04/23(月) 23:13:26.59
rank(A)=rank(A^t)=rank(AA^t)=rank(A^tA)

の証明を教えてください
789132人目の素数さん:2012/04/23(月) 23:32:44.02
「{A→(B→C)}→{(A→B)→(A→C)}」って、「[{A→(B→C)}かつ(A→B)]→(A→C)」ってことだろ?
「[(A→B)かつ{A→(B→C)}]→(A→C)って当たり前だと思うけど。
Aの真偽を考える必要はないんじゃないの?
790132人目の素数さん:2012/04/24(火) 00:25:03.63
>>788
その問におけるrankの定義は?
791あぼーん:あぼーん
あぼーん
792132人目の素数さん:2012/04/24(火) 08:26:06.61
A の行(列でも同じ)をいくつか取ってきて、1 次独立になる最大個数

A に行と列の基本変形を施して、左上端に単位行列 E 、他の成分が
全て 0 になるようにしたときの E の大きさ

で定義します。
当然ですがA^tはAの転置です。
793132人目の素数さん:2012/04/24(火) 11:05:22.56
基本変形は左右から正則行列を掛けること。
794132人目の素数さん:2012/04/24(火) 11:06:03.88
行列Aを
1 i
0 0
とすりゃAA^t=O なので成り立んわな
795132人目の素数さん:2012/04/24(火) 17:56:26.90
Aの成分が虚数以外なら成り立ちますか?
796132人目の素数さん:2012/04/24(火) 22:03:10.74
>>795
標数2の体を係数に持つ行列Aを
1 1
0 0
とするとAA^t=0
797あぼーん:あぼーん
あぼーん
798132人目の素数さん:2012/04/25(水) 21:46:14.12
読みたい数学書が4冊ほどあるんですが
凡人が4冊同時に読み進めようとしても大丈夫でしょうか
それとも1冊ずつやった方がいいですか
799132人目の素数さん:2012/04/25(水) 22:54:25.19
集中できる方でやれば
800あぼーん:あぼーん
あぼーん
801132人目の素数さん:2012/05/05(土) 13:35:56.56
>>798
能力云々はさておき、頭の働き方には個性があるので
一般論なんてないだろ。

数学者(国内のまあ一流数学者たち)の座談会で司会が
「数学の勉強法ってどうですか」なんて話を振ったんだが
数学の勉強法って何がいいのか未だに悩んでますみたいな事を言っていて
へぇって思ったな。

あと、岡本清郷だかが誰かから聞いた話だったか(うろ覚え)
年配の数学者から「数学はね、ノートに鉛筆を使って丁寧に書いてくのが一番ですよ」
と言われたなんて話も印象的だな。(書き損じを修正できないペンではなく消しゴム使って
いつも綺麗な状態を保ち続けるというような意味らしい)。
802132人目の素数さん:2012/05/13(日) 01:29:00.38
大したことではないのだが、これの証明がいまひとつうまくできないんだ。

f:X→Y を局所Noetherスキームの間の固有射であるとし、
X , Y の構造層 , の間には f*( )= が成り立つと仮定する。
このとき、任意の y∈Y に対し、 f^-1(y) は、空でなく、連結である。
803132人目の素数さん:2012/05/13(日) 15:06:42.52
>>801

岡本さんみたいな3流に云われてもなw
804あぼーん:あぼーん
あぼーん
805132人目の素数さん:2012/05/13(日) 21:16:44.05
f^2=(f-d)^2+r^2 という式を立てて、f、rからdを求めようと、
f^2=f^2-2fd+d^2+r^2 とかやったら、糞詰まってしまいました。

結局求めたかったのは、d=|f-√(f^2-r^2)| のようなんですが、
2行目の式からこの式に持ってくる術はありますか?
(f-d)^2を展開しちゃっても、頑張ればなんとかd=の形に持ってこれるか
どうかを知りたいんです。中学数学レベルでお願いします。
806132人目の素数さん:2012/05/13(日) 22:55:27.95
中学生かな?
2行目はd^2-2fd+r^2=0と書けるよね。
この式は、いわゆる平方完成という方法を使って(d-f)^2-f^2+r^2=0とすることができる。(中学で習うっけ?)
この式を(d-f)^2-(f^2-r^2)と書くと、
(d-f)^2-(f^2-r^2)
=(d-f)^2-{√(f^2-r^2)}^2
=[(d-f)+{√(f^2-r^2)}][(d-f)-{√(f^2-r^2)}]=0
だから、
[(d-f)+{√(f^2-r^2)}]=0 “または”(これ大事) [(d-f)-{√(f^2-r^2)}]=0
これよりd=f+√(f^2-r^2) “または” d=f-√(f^2-r^2)
まとめて表すとd=f±√(f^2-r^2)

こんなもんでどう?
ちなみにこのやり方は2次方程式の解の導き方そのものだよ。
807132人目の素数さん:2012/05/13(日) 23:50:54.14
(b-c)a^2-(b-c)(b+c)a+bc(b-c)
から
(b-c){a^2-(b+c)a+bc}

に変わる所があるのですがどの公式を使ってるんでしょうか
808132人目の素数さん:2012/05/14(月) 00:47:29.71
AC+BC=A(B+C)

という分配法則を使ってます。
これは数学のもっとも土台となる法則の一つであって、何からも導かれません。
導かれないのですが、A,B,Cに色んな数値を代入してみて下さい、この式が成り立ちますよね。
このように、何からも導かれないけどどう考えても成り立ってるとしか思えないような式が「法則」と呼ばれている式です。
公式や定理はこれらの「法則」から導かれます。
法則を色々組み合わせていったものが公式や定理というわけです。

(b-c)a^2-(b-c)(b+c)a+bc(b-c)の場合、
分配法則より(b-c)a^2-(b-c)(b+c)a=(b-c){a^2-(b+c)a}なので、
(b-c){a^2-(b+c)a}+bc(b-c)と書くことができますよね。
もう一度分配法則を使いましょう。
すると(b-c){a^2-(b+c)a+bc}を導くことができます。
どうですか?法則から導かれましたよね。
このように、すべての公式や定理は法則から導かれます。
809132人目の素数さん:2012/05/14(月) 22:10:35.26
>>806
ありがとう。
結局、(f-d)とか(d-f)とかの状態にしとけ、ってことに気付かないとダメなんですね。
「平方完成」なんてのも、言われないと出てこない朧気な記憶だし。

> 中学生かな?
はい。
はるか遠い昔の、ね。
810132人目の素数さん:2012/05/14(月) 23:57:48.51
>>809
>結局、(f-d)とか(d-f)とかの状態にしとけ、ってことに気付かないとダメなんですね。

実は、「平方完成」のやり方さえ知っておけばおのずとその形になるんです。
(x+a)^2=x^2+2ax+a^2 ですよね。
平方完成とはこの左辺の項を(無理やり)作り出すことです。
d^2-2fd+r^2=0を平方完成して(d+なんとか)^2の項を無理やり作り出してみますね。
覚えておくべきは(x+a)^2=x^2+2ax+a^2を書き換えた式である「x^2+2ax=(x+a)^2-a^2」です。
d^2-2fdをこの式にならって変形してみますと、d^2-2fd = (d-f)^2-f^2となります。
(d-f)の状態になりましたね。
よって、d^2-2fd+r^2=0 を(d-f)^2-f^2+r^2=0と書くことができます。
後は>>806の通りです。

ちなみに、どんな数も「2かける何とか」の形にできますので(例えば3 = 2かける3/2)、
x^2+5x+8を平方完成しようと思ったらx^2+2・(5/2)x+8というように、xの項を無理やり(2かける何とか)xという形にします。
そうすると、x^2+5x+8 = x^2+2・(5/2)x+8 = (x+5/2)^2-(5/2)^2 + 8 = (x+5/2)^2 + 7/4
というように平方完成することができます。
「x^2+2ax=(x+a)^2-a^2」がキーポイントなんです。
811あぼーん:あぼーん
あぼーん
812あぼーん:あぼーん
あぼーん
813あぼーん:あぼーん
あぼーん
814あぼーん:あぼーん
あぼーん
815あぼーん:あぼーん
あぼーん
816132人目の素数さん:2012/05/24(木) 10:43:41.39
化学の参考書スレで起きている偶然はどれくらいの確率か?
817あぼーん:あぼーん
あぼーん
818132人目の素数さん:2012/05/26(土) 16:45:43.21
ペアノの公理について質問です。

0を自然数に含めるとして。公理1〜5を満たすのを自然数と定義するなら、
「0以上の偶数の集合」 なども自然数になる気がするんだけど、なぜダメなのかが知りたいです。

1.自然数 0 が存在する。
2.任意の自然数 a にはその後者 (successor)、suc(a) が存在する(suc(a) は a + 1 の "意味")。
3.0 はいかなる自然数の後者でもない(0 より前の自然数は存在しない)。
4.異なる自然数は異なる後者を持つ:a ≠ b のとき suc(a) ≠ suc(b) となる。
5.0 がある性質を満たし、a がある性質を満たせばその後者 suc(a) もその性質を満たすとき、すべての自然数はその性質を満たす。

suc() を +2 と考えれば、偶数の集合は1〜5全てを満たしてるような…。
819132人目の素数さん:2012/05/26(土) 18:21:53.31
それにはまだ乗法の単位元が入ってないね
820132人目の素数さん:2012/05/26(土) 18:37:13.13
どういうことでしょうか?
821132人目の素数さん:2012/05/26(土) 19:16:00.44
同型を除いてない
822132人目の素数さん:2012/05/26(土) 19:17:08.85
まだそれには加法や乗法とか入れてない段階だから
公理を満たすならなんだっていいのさ
まあ一般常識的な四則演算や単位元などが
よく知られたものとして指名されて揃ってる自然数じゃないから
断り書きは必要なんだろうけど
823132人目の素数さん:2012/05/26(土) 19:21:21.82
つまりこの段階では、「0以上の偶数の集合」 も 「自然数」 の定義の1つということでしょうか?
ここでいう自然数ってのは、一般的に言う {1,2,3,4…} とは違う意味なのでしょうか。
824132人目の素数さん:2012/05/26(土) 19:41:53.75
そこでの2って何よ?ということだね。
825132人目の素数さん:2012/05/26(土) 22:05:42.73
子犬のワルツ
826あぼーん:あぼーん
あぼーん
827132人目の素数さん:2012/05/27(日) 21:44:21.34
>>823
自然数に0を含むとする流儀もある。
828132人目の素数さん:2012/05/27(日) 21:50:20.65
>>823
> 「自然数」 の定義の1つということでしょうか?
モデルの一つ
829132人目の素数さん:2012/05/28(月) 15:55:43.20
>>823
その段階での「偶数」の定義とは?
830132人目の素数さん:2012/05/29(火) 21:02:00.62
高校1年です。
全ての 5以上かつ偶数 の数 の否定は、
ある 5未満または奇数 の数

任意の 両方とも2でない素数 の否定は、
ある どちらかが2である素数

でいいでしょうか? よろしくお願いします。
831132人目の素数さん:2012/05/29(火) 22:19:04.58
ダメだろ。そもそも命題になってない。
832132人目の素数さん:2012/05/29(火) 22:53:03.14
833132人目の素数さん:2012/05/29(火) 23:08:05.99
>>832
全体集合が実数の集合なの?
834132人目の素数さん:2012/05/29(火) 23:11:54.80
>>833 すみません 整数ですf^_^;)
835あぼーん:あぼーん
あぼーん
836あぼーん:あぼーん
あぼーん
837132人目の素数さん:2012/05/30(水) 00:37:32.50
>>834
Uは実数ってのは間違いなんだね?

>>832
Pの補集合を求めたいということなら、
>>830の「全ての」は適切じゃないし
>>832じゃ「かつ」と「または」が不明瞭。
838132人目の素数さん:2012/05/30(水) 00:56:37.85
>>830
>任意の 両方とも2でない素数
曖昧な表現だなあ。

ともに2ではない2個の素数 を 任意に取る

ということか?

その整数をx,y と表すと
xとy はともに2ではなく、かつ ともに素数である。

この否定は

xとy は 少なくとも一方は2であるか、または、少なくとも一方は素数ではない

839KingMathematician ◆LoZDre77j4i1 :2012/05/31(木) 19:56:55.54
Re:>>777 経験値とは何か. Wizardry の日本語訳か.
840132人目の素数さん:2012/06/01(金) 03:11:11.78
会社の社長たちにインタビューしたところ、彼らの90%が歯を磨いていた。歯を丈夫に保てば、乾坤一擲、ここ一番というときに歯をくいしばって頑張ることができるから成功を収めたのだ。歯を磨くのが成功のもと!

 この本を読んだ、某大学のある「残念な」学生A君は、一念発起し、歯を磨くことにしたそうです。そのような「残念な」学生にならないために、以下の問いに答えて下さい。

<問1>
 「会社の社長たちにインタビューしたところ、彼らの90%が歯を磨いていた。」という情報を、確率の記号を使用して表現して下さい。

<問2>
 A君は「歯を磨いている人が社長になる確率」が高いのではないかと考えたことになります。この確率を、確率の記号を使用して表現して下さい。

<問3>
 某大学の学生が社長になる確率=0.01%、日本人のある人が歯を磨いている確率=80%、とします。与えられた情報を基にすると、<問2>で表現した確率は、何%になるか、計算して下さい。
841132人目の素数さん:2012/06/01(金) 13:25:55.67
<問1>は 確率で表わせる情報じゃない。
もうちょっと知ってから問題作りましょう。
842132人目の素数さん:2012/06/10(日) 06:15:38.31
          __ノ)-'´ ̄ ̄`ー- 、_
        , '´  _. -‐'''"二ニニ=-`ヽ、
      /   /:::::; -‐''"        `ーノ
     /   /:::::/           \
     /    /::::::/          | | |  |
     |   |:::::/ /     |  | | | |  |
      |   |::/ / / |  | ||  | | ,ハ .| ,ハ|
      |   |/ / / /| ,ハノ| /|ノレ,ニ|ル' 
     |   |  | / / レ',二、レ′ ,ィイ|゙/   私は只の数ヲタなんかとは付き合わないわ。
.     |   \ ∠イ  ,イイ|    ,`-' |      頭が良くて数学が出来てかっこいい人。それが必要条件よ。
     |     l^,人|  ` `-'     ゝ  |        さらに Ann.of Math に論文書けば十分条件にもなるわよ。
      |      ` -'\       ー'  人          一番嫌いなのは論文数を増やすためにくだらない論文を書いて
    |        /(l     __/  ヽ、           良い論文の出版を遅らせるお馬鹿な人。
     |       (:::::`‐-、__  |::::`、     ヒニニヽ、         あなたの論文が Ann of Math に accept される確率は?
    |      / `‐-、::::::::::`‐-、::::\   /,ニニ、\            それとも最近は Inv. Math. の方が上かしら?
   |      |::::::::::::::::::|` -、:::::::,ヘ ̄|'、  ヒニ二、 \
.   |      /::::::::::::::::::|::::::::\/:::O`、::\   | '、   \
   |      /:::::::::::::::::::/:::::::::::::::::::::::::::::'、::::\ノ  ヽ、  |
  |      |:::::/:::::::::/:::::::::::::::::::::::::::::::::::'、',::::'、  /:\__/‐、
  |      |/:::::::::::/::::::::::::::::::::::::::::::::::O::| '、::| く::::::::::::: ̄|
   |     /_..-'´ ̄`ー-、:::::::::::::::::::::::::::::::::::|/:/`‐'::\;;;;;;;_|
   |    |/::::::::::::::::::::::\:::::::::::::::::::::::::::::|::/::::|::::/:::::::::::/
    |   /:::::::::::::::::::::::::::::::::|:::::::::::::::::::::O::|::|::::::|:::::::::::::::/
843132人目の素数さん:2012/06/12(火) 07:12:34.72
1
844132人目の素数さん:2012/06/12(火) 08:32:18.15
          __ノ)-'´ ̄ ̄`ー- 、_
        , '´  _. -‐'''"二ニニ=-`ヽ、
      /   /:::::; -‐''"        `ーノ
     /   /:::::/           \
     /    /::::::/          | | |  |
     |   |:::::/ /     |  | | | |  |
      |   |::/ / / |  | ||  | | ,ハ .| ,ハ|
      |   |/ / / /| ,ハノ| /|ノレ,ニ|ル' 
     |   |  | / / レ',二、レ′ ,ィイ|゙/   私は只の数ヲタなんかとは付き合わないわ。
.     |   \ ∠イ  ,イイ|    ,`-' |      頭が良くて数学が出来てかっこいい人。それが必要条件よ。
     |     l^,人|  ` `-'     ゝ  |        さらに Ann.of Math に論文書けば十分条件にもなるわよ。
      |      ` -'\       ー'  人          一番嫌いなのは論文数を増やすためにくだらない論文を書いて
    |        /(l     __/  ヽ、           良い論文の出版を遅らせるお馬鹿な人。
     |       (:::::`‐-、__  |::::`、     ヒニニヽ、         あなたの論文が Ann of Math に accept される確率は?
    |      / `‐-、::::::::::`‐-、::::\   /,ニニ、\            それとも最近は Inv. Math. の方が上かしら?
   |      |::::::::::::::::::|` -、:::::::,ヘ ̄|'、  ヒニ二、 \
.   |      /::::::::::::::::::|::::::::\/:::O`、::\   | '、   \
   |      /:::::::::::::::::::/:::::::::::::::::::::::::::::'、::::\ノ  ヽ、  |
  |      |:::::/:::::::::/:::::::::::::::::::::::::::::::::::'、',::::'、  /:\__/‐、
  |      |/:::::::::::/::::::::::::::::::::::::::::::::::O::| '、::| く::::::::::::: ̄|
   |     /_..-'´ ̄`ー-、:::::::::::::::::::::::::::::::::::|/:/`‐'::\;;;;;;;_|
   |    |/::::::::::::::::::::::\:::::::::::::::::::::::::::::|::/::::|::::/:::::::::::/
    |   /:::::::::::::::::::::::::::::::::|:::::::::::::::::::::O::|::|::::::|:::::::::::::::/
845132人目の素数さん:2012/06/12(火) 12:04:41.49
          __ノ)-'´ ̄ ̄`ー- 、_
        , '´  _. -‐'''"二ニニ=-`ヽ、
      /   /:::::; -‐''"        `ーノ
     /   /:::::/           \
     /    /::::::/          | | |  |
     |   |:::::/ /     |  | | | |  |
      |   |::/ / / |  | ||  | | ,ハ .| ,ハ|
      |   |/ / / /| ,ハノ| /|ノレ,ニ|ル' 
     |   |  | / / レ',二、レ′ ,ィイ|゙/   私は只の数ヲタなんかとは付き合わないわ。
.     |   \ ∠イ  ,イイ|    ,`-' |      頭が良くて数学が出来てかっこいい人。それが必要条件よ。
     |     l^,人|  ` `-'     ゝ  |        さらに Ann.of Math に論文書けば十分条件にもなるわよ。
      |      ` -'\       ー'  人          一番嫌いなのは論文数を増やすためにくだらない論文を書いて
    |        /(l     __/  ヽ、           良い論文の出版を遅らせるお馬鹿な人。
     |       (:::::`‐-、__  |::::`、     ヒニニヽ、         あなたの論文が Ann of Math に accept される確率は?
    |      / `‐-、::::::::::`‐-、::::\   /,ニニ、\            それとも最近は Inv. Math. の方が上かしら?
   |      |::::::::::::::::::|` -、:::::::,ヘ ̄|'、  ヒニ二、 \
.   |      /::::::::::::::::::|::::::::\/:::O`、::\   | '、   \
   |      /:::::::::::::::::::/:::::::::::::::::::::::::::::'、::::\ノ  ヽ、  |
  |      |:::::/:::::::::/:::::::::::::::::::::::::::::::::::'、',::::'、  /:\__/‐、
  |      |/:::::::::::/::::::::::::::::::::::::::::::::::O::| '、::| く::::::::::::: ̄|
   |     /_..-'´ ̄`ー-、:::::::::::::::::::::::::::::::::::|/:/`‐'::\;;;;;;;_|
   |    |/::::::::::::::::::::::\:::::::::::::::::::::::::::::|::/::::|::::/:::::::::::/
    |   /:::::::::::::::::::::::::::::::::|:::::::::::::::::::::O::|::|::::::|:::::::::::::::/
846132人目の素数さん:2012/06/12(火) 20:20:58.95
http://beebee2see.appspot.com/i/azuYoZXSBgw.jpg
この問一を教えてください
847132人目の素数さん:2012/06/12(火) 20:52:25.62
848132人目の素数さん:2012/06/12(火) 20:58:20.49
>>846
x→0 を先にするか y→0 を先にするかで極限が違うから存在しない。
849132人目の素数さん:2012/06/12(火) 23:34:56.23
>>848
xを先に0にしたら0でyを先に0にしたら1であってますかね?
850132人目の素数さん:2012/06/13(水) 00:33:36.99
あってるよ
ついでに
x=c^(1/t)
y=t
と置くと
lim_[t→+0](x,y)=(0,0)
だけど、
lim_[t→+0](x^y)=lim_[t→+0]((c^(1/t))^t)=c
になる
851132人目の素数さん:2012/06/14(木) 16:34:18.73
次の関数が分布関数となるにはa<=b<=1-a,0<=a<=1/2であることを示せ
F(x)=0 (x<-1)
=ax+b (-1<=x<1)
=1 (1<=x)

0<=F(x)<=1よりa<=b<=1と0<=aは出るのですがa<=1/2が出てきません
非減少か右連続であることを使うのでしょうか?
どうすればいいのか分からないです
ご教授お願いします
852132人目の素数さん:2012/06/14(木) 22:15:03.93
b+a<=1、a<=b
→2a<=b+a<=1
853132人目の素数さん:2012/06/15(金) 14:25:47.51
          __ノ)-'´ ̄ ̄`ー- 、_
        , '´  _. -‐'''"二ニニ=-`ヽ、
      /   /:::::; -‐''"        `ーノ
     /   /:::::/           \
     /    /::::::/          | | |  |
     |   |:::::/ /     |  | | | |  |
      |   |::/ / / |  | ||  | | ,ハ .| ,ハ|
      |   |/ / / /| ,ハノ| /|ノレ,ニ|ル' 
     |   |  | / / レ',二、レ′ ,ィイ|゙/   私は只の数ヲタなんかとは付き合わないわ。
.     |   \ ∠イ  ,イイ|    ,`-' |      頭が良くて数学が出来てかっこいい人。それが必要条件よ。
     |     l^,人|  ` `-'     ゝ  |        さらに Ann.of Math に論文書けば十分条件にもなるわよ。
      |      ` -'\       ー'  人          一番嫌いなのは論文数を増やすためにくだらない論文を書いて
    |        /(l     __/  ヽ、           良い論文の出版を遅らせるお馬鹿な人。
     |       (:::::`‐-、__  |::::`、     ヒニニヽ、         あなたの論文が Ann of Math に accept される確率は?
    |      / `‐-、::::::::::`‐-、::::\   /,ニニ、\            それとも最近は Inv. Math. の方が上かしら?
   |      |::::::::::::::::::|` -、:::::::,ヘ ̄|'、  ヒニ二、 \
.   |      /::::::::::::::::::|::::::::\/:::O`、::\   | '、   \
   |      /:::::::::::::::::::/:::::::::::::::::::::::::::::'、::::\ノ  ヽ、  |
  |      |:::::/:::::::::/:::::::::::::::::::::::::::::::::::'、',::::'、  /:\__/‐、
  |      |/:::::::::::/::::::::::::::::::::::::::::::::::O::| '、::| く::::::::::::: ̄|
   |     /_..-'´ ̄`ー-、:::::::::::::::::::::::::::::::::::|/:/`‐'::\;;;;;;;_|
   |    |/::::::::::::::::::::::\:::::::::::::::::::::::::::::|::/::::|::::/:::::::::::/
    |   /:::::::::::::::::::::::::::::::::|:::::::::::::::::::::O::|::|::::::|:::::::::::::::/
854132人目の素数さん:2012/06/15(金) 16:56:26.07
∂{∬_[D]f(x,y)dxdy}/∂x = ∬_[D](∂{f(x,y)}/∂x)dxdy

ってどんな条件で成立しますか?
855132人目の素数さん:2012/06/15(金) 17:11:07.79
          __ノ)-'´ ̄ ̄`ー- 、_
        , '´  _. -‐'''"二ニニ=-`ヽ、
      /   /:::::; -‐''"        `ーノ
     /   /:::::/           \
     /    /::::::/          | | |  |
     |   |:::::/ /     |  | | | |  |
      |   |::/ / / |  | ||  | | ,ハ .| ,ハ|
      |   |/ / / /| ,ハノ| /|ノレ,ニ|ル' 
     |   |  | / / レ',二、レ′ ,ィイ|゙/   私は只の数ヲタなんかとは付き合わないわ。
.     |   \ ∠イ  ,イイ|    ,`-' |      頭が良くて数学が出来てかっこいい人。それが必要条件よ。
     |     l^,人|  ` `-'     ゝ  |        さらに Ann.of Math に論文書けば十分条件にもなるわよ。
      |      ` -'\       ー'  人          一番嫌いなのは論文数を増やすためにくだらない論文を書いて
    |        /(l     __/  ヽ、           良い論文の出版を遅らせるお馬鹿な人。
     |       (:::::`‐-、__  |::::`、     ヒニニヽ、         あなたの論文が Ann of Math に accept される確率は?
    |      / `‐-、::::::::::`‐-、::::\   /,ニニ、\            それとも最近は Inv. Math. の方が上かしら?
   |      |::::::::::::::::::|` -、:::::::,ヘ ̄|'、  ヒニ二、 \
.   |      /::::::::::::::::::|::::::::\/:::O`、::\   | '、   \
   |      /:::::::::::::::::::/:::::::::::::::::::::::::::::'、::::\ノ  ヽ、  |
  |      |:::::/:::::::::/:::::::::::::::::::::::::::::::::::'、',::::'、  /:\__/‐、
  |      |/:::::::::::/::::::::::::::::::::::::::::::::::O::| '、::| く::::::::::::: ̄|
   |     /_..-'´ ̄`ー-、:::::::::::::::::::::::::::::::::::|/:/`‐'::\;;;;;;;_|
   |    |/::::::::::::::::::::::\:::::::::::::::::::::::::::::|::/::::|::::/:::::::::::/
    |   /:::::::::::::::::::::::::::::::::|:::::::::::::::::::::O::|::|::::::|:::::::::::::::/
856132人目の素数さん:2012/06/15(金) 17:57:34.88
>>854
0=0 の場合
857132人目の素数さん:2012/06/15(金) 18:42:33.72
          __ノ)-'´ ̄ ̄`ー- 、_
        , '´  _. -‐'''"二ニニ=-`ヽ、
      /   /:::::; -‐''"        `ーノ
     /   /:::::/           \
     /    /::::::/          | | |  |
     |   |:::::/ /     |  | | | |  |
      |   |::/ / / |  | ||  | | ,ハ .| ,ハ|
      |   |/ / / /| ,ハノ| /|ノレ,ニ|ル' 
     |   |  | / / レ',二、レ′ ,ィイ|゙/   私は只の数ヲタなんかとは付き合わないわ。
.     |   \ ∠イ  ,イイ|    ,`-' |      頭が良くて数学が出来てかっこいい人。それが必要条件よ。
     |     l^,人|  ` `-'     ゝ  |        さらに Ann.of Math に論文書けば十分条件にもなるわよ。
      |      ` -'\       ー'  人          一番嫌いなのは論文数を増やすためにくだらない論文を書いて
    |        /(l     __/  ヽ、           良い論文の出版を遅らせるお馬鹿な人。
     |       (:::::`‐-、__  |::::`、     ヒニニヽ、         あなたの論文が Ann of Math に accept される確率は?
    |      / `‐-、::::::::::`‐-、::::\   /,ニニ、\            それとも最近は Inv. Math. の方が上かしら?
   |      |::::::::::::::::::|` -、:::::::,ヘ ̄|'、  ヒニ二、 \
.   |      /::::::::::::::::::|::::::::\/:::O`、::\   | '、   \
   |      /:::::::::::::::::::/:::::::::::::::::::::::::::::'、::::\ノ  ヽ、  |
  |      |:::::/:::::::::/:::::::::::::::::::::::::::::::::::'、',::::'、  /:\__/‐、
  |      |/:::::::::::/::::::::::::::::::::::::::::::::::O::| '、::| く::::::::::::: ̄|
   |     /_..-'´ ̄`ー-、:::::::::::::::::::::::::::::::::::|/:/`‐'::\;;;;;;;_|
   |    |/::::::::::::::::::::::\:::::::::::::::::::::::::::::|::/::::|::::/:::::::::::/
    |   /:::::::::::::::::::::::::::::::::|:::::::::::::::::::::O::|::|::::::|:::::::::::::::/
858132人目の素数さん:2012/06/15(金) 20:09:38.30
累乗引く累乗 
例えば 3の5乗引く3の2乗
ってのは簡単にする方法無く値を出してから引くしかないの?
859132人目の素数さん:2012/06/15(金) 20:12:54.36
          __ノ)-'´ ̄ ̄`ー- 、_
        , '´  _. -‐'''"二ニニ=-`ヽ、
      /   /:::::; -‐''"        `ーノ
     /   /:::::/           \
     /    /::::::/          | | |  |
     |   |:::::/ /     |  | | | |  |
      |   |::/ / / |  | ||  | | ,ハ .| ,ハ|
      |   |/ / / /| ,ハノ| /|ノレ,ニ|ル' 
     |   |  | / / レ',二、レ′ ,ィイ|゙/   私は只の数ヲタなんかとは付き合わないわ。
.     |   \ ∠イ  ,イイ|    ,`-' |      頭が良くて数学が出来てかっこいい人。それが必要条件よ。
     |     l^,人|  ` `-'     ゝ  |        さらに Ann.of Math に論文書けば十分条件にもなるわよ。
      |      ` -'\       ー'  人          一番嫌いなのは論文数を増やすためにくだらない論文を書いて
    |        /(l     __/  ヽ、           良い論文の出版を遅らせるお馬鹿な人。
     |       (:::::`‐-、__  |::::`、     ヒニニヽ、         あなたの論文が Ann of Math に accept される確率は?
    |      / `‐-、::::::::::`‐-、::::\   /,ニニ、\            それとも最近は Inv. Math. の方が上かしら?
   |      |::::::::::::::::::|` -、:::::::,ヘ ̄|'、  ヒニ二、 \
.   |      /::::::::::::::::::|::::::::\/:::O`、::\   | '、   \
   |      /:::::::::::::::::::/:::::::::::::::::::::::::::::'、::::\ノ  ヽ、  |
  |      |:::::/:::::::::/:::::::::::::::::::::::::::::::::::'、',::::'、  /:\__/‐、
  |      |/:::::::::::/::::::::::::::::::::::::::::::::::O::| '、::| く::::::::::::: ̄|
   |     /_..-'´ ̄`ー-、:::::::::::::::::::::::::::::::::::|/:/`‐'::\;;;;;;;_|
   |    |/::::::::::::::::::::::\:::::::::::::::::::::::::::::|::/::::|::::/:::::::::::/
    |   /:::::::::::::::::::::::::::::::::|:::::::::::::::::::::O::|::|::::::|:::::::::::::::/
860あのこうちやんは始皇帝だった:2012/06/15(金) 20:21:58.10
>>859

 死ね!!!!!!!!!!!!!!!!!

 ニートの、甘ったれの、ゴミ・クズ・カスのクソガキ!!!!!!!!!!!!!!!

861あぼーん:2012/06/15(金) 22:36:03.70
あぼーん
862132人目の素数さん:2012/06/16(土) 02:35:58.50
>>858
冪が偶数なら a^2-b^2 の因数分解が使える。
冪が倍数なら a^n-b^n の因数分解が使える。
863132人目の素数さん:2012/06/16(土) 07:44:20.87
>>862
thx
因数分解確認してみます
864132人目の素数さん:2012/06/16(土) 12:42:55.18
いやいや、それやっても仕方ないだろ。
3^5-3^2=3^2*(3^3-1)で十分。これ以上因数分解したらむしろ計算量増える。
べき乗を効率的に計算したいならバイナリ法という方法がある。
865132人目の素数さん:2012/06/16(土) 16:34:36.50
学校で3の50乗引く3の20乗の1桁の数字はいくつかって問題があって
1、3、9、7のループから答えは出せたものの

普通に計算したらどうなるんだ?と思い
3^50-3^20=3^30か・・いや、それは分数の指数法則か、

あれ?簡単にする方法なくね?
そもそも3^5-3^2すら自力で累乗計算して引くっていう力技だけじゃん・・
と思い質問しました

いまだに得心が得られず、3^100-3^87等、数が大きくなると無理ゲーっぽいです
866132人目の素数さん:2012/06/16(土) 17:26:14.40
>>864のバイナリ法ってのは
3*3=9
9*9=81
81*81=6561
6561*6561=43046721

と言うように4回の掛け算で3^16( =3^2^4)が求まるというような方法で
15回掛けるよりも計算が少なくてすむ。
まあ、もしかすると既に使ってるかもしれないが一応。
867132人目の素数さん:2012/06/16(土) 17:50:22.05
3^10=9^5=(10-1)^5=100k-50k+10k-1k+50-1=50k+9k+49=59,049
これ以上は手計算だと扱いたくないな…
868132人目の素数さん:2012/06/16(土) 18:36:45.90
3^50-3^20 = 9^25-9^10 = (-1)^25-(-1)^10 = (-1)-1 = -2 = 8 (mod 10)
3^100-3^87 = 9^50-3*9^43 = (-1)^50-3*(-1)^43 = 1-3*(-1) = 4 (mod 10)
869132人目の素数さん:2012/06/16(土) 19:01:05.71
そりゃmod 10で考えていいなら手計算でも楽だけど
870132人目の素数さん:2012/06/17(日) 12:55:08.46
複素数の問題なんだけど

(1-i)^2(1+i√(3))

これを分かりやすく解説してほしい・・・
教科書じゃどこをどうすればいいのかさっぱり。
871132人目の素数さん:2012/06/17(日) 13:03:42.42
>>870
解説て何?
計算できないてこと?
872132人目の素数さん:2012/06/17(日) 13:08:34.99
>>871
計算のためにまずどこをどうすれば良いのかが良く分からない。
873132人目の素数さん:2012/06/17(日) 13:27:59.20
とりあえず、iをただの文字として計算して、最後にi^2を-1に置き換える
874132人目の素数さん:2012/06/17(日) 13:51:14.10
線形代数の問題です。行列{a1a2a3}={[1 2 3][‐1 0 1][1 0 0]}を正規直交化せよ
よろしくお願いします。
875132人目の素数さん:2012/06/17(日) 14:32:31.04
>>874
・マルチ
・努力したようすがない
・二番煎じ
・wolfmanにでも聞け
876132人目の素数さん:2012/06/17(日) 20:37:17.81
当たり確率1/170のスロットマシンが100回以内で当たる確率は

1 - (169/170)^100
=1 - 0.5543426881966389
=0.4456573118033611

で、あってる?
計算の方法と、あと精度が合ってるのか知りたい。
877132人目の素数さん:2012/06/17(日) 21:20:52.70
式はあってるが値が下…で合ってるかどうかは知らん
0.
445657311803362234208523062696128240128424783737792034683835
992612305183380549470950403874082754070913801773556856375010
185819744427338019699347691502252880442503024887123409037952
289186082723378171273477768933101384619427501785986446483144
786908216875846710537288113069253327719638198780123636110373
079031780508999964969363471219956579481683117959412534125886
065787217570808347213596050707268234181402154938500265324438
598977583662757700147793621062751887075388344271045805222297
522559172794353707361327018634319010696346590271447283313512
033539632770119136113666770623437369568401471325638929879696
870625145882820651031338000260471520076138525502648845457246
891074612720019614905405226031266143087032956492629046131988
064136685546445332247263496851429226571334769837048151010224
572210276848192235552770100048093184790639288522435533951685
087983007103941256593868843887175710432574038949168138467172
954043442429767925634379712653068848704698222360206536779719
228846227613039140246122720667966793906499418441677875557431
222676696400263800513619060989737734273780843432436163687914
149746186329563377939119282242987738255332539052772019988438
243306523060319258116978524365413304169433024011873354510582
631516839315373028182248777754421877604385630659179...

100回スロットするときの確率計算なら小数点以下4桁くらいで十分のような気がするが…
878132人目の素数さん:2012/06/17(日) 22:14:08.96
>>877
サンクス!
精度云々は適当にプログラム組んだんで、どれくらい合ってるか気になったんだ。
それじゃ「確率は100/170だ!」って言い張る友人を泣かしてくる!
879132人目の素数さん:2012/06/17(日) 23:05:02.83
まあスロットなんて、裏で何されてるかわかったもんじゃないと思うがなあ
いくら法律があっても…ねえ
たださすがに100/170との違いは感じられると思う
880132人目の素数さん:2012/06/19(火) 22:05:15.66
          __ノ)-'´ ̄ ̄`ー- 、_
        , '´  _. -‐'''"二ニニ=-`ヽ、
      /   /:::::; -‐''"        `ーノ
     /   /:::::/           \
     /    /::::::/          | | |  |
     |   |:::::/ /     |  | | | |  |
      |   |::/ / / |  | ||  | | ,ハ .| ,ハ|
      |   |/ / / /| ,ハノ| /|ノレ,ニ|ル' 
     |   |  | / / レ',二、レ′ ,ィイ|゙/   私は只の数ヲタなんかとは付き合わないわ。
.     |   \ ∠イ  ,イイ|    ,`-' |      頭が良くて数学が出来てかっこいい人。それが必要条件よ。
     |     l^,人|  ` `-'     ゝ  |        さらに Ann.of Math に論文書けば十分条件にもなるわよ。
      |      ` -'\       ー'  人          一番嫌いなのは論文数を増やすためにくだらない論文を書いて
    |        /(l     __/  ヽ、           良い論文の出版を遅らせるお馬鹿な人。
     |       (:::::`‐-、__  |::::`、     ヒニニヽ、         あなたの論文が Ann of Math に accept される確率は?
    |      / `‐-、::::::::::`‐-、::::\   /,ニニ、\            それとも最近は Inv. Math. の方が上かしら?
   |      |::::::::::::::::::|` -、:::::::,ヘ ̄|'、  ヒニ二、 \
.   |      /::::::::::::::::::|::::::::\/:::O`、::\   | '、   \
   |      /:::::::::::::::::::/:::::::::::::::::::::::::::::'、::::\ノ  ヽ、  |
  |      |:::::/:::::::::/:::::::::::::::::::::::::::::::::::'、',::::'、  /:\__/‐、
  |      |/:::::::::::/::::::::::::::::::::::::::::::::::O::| '、::| く::::::::::::: ̄|
   |     /_..-'´ ̄`ー-、:::::::::::::::::::::::::::::::::::|/:/`‐'::\;;;;;;;_|
   |    |/::::::::::::::::::::::\:::::::::::::::::::::::::::::|::/::::|::::/:::::::::::/
    |   /:::::::::::::::::::::::::::::::::|:::::::::::::::::::::O::|::|::::::|:::::::::::::::/
881132人目の素数さん:2012/06/26(火) 06:01:50.97
          __ノ)-'´ ̄ ̄`ー- 、_
        , '´  _. -‐'''"二ニニ=-`ヽ、
      /   /:::::; -‐''"        `ーノ
     /   /:::::/           \
     /    /::::::/          | | |  |
     |   |:::::/ /     |  | | | |  |
      |   |::/ / / |  | ||  | | ,ハ .| ,ハ|
      |   |/ / / /| ,ハノ| /|ノレ,ニ|ル' 
     |   |  | / / レ',二、レ′ ,ィイ|゙/   私は只の数ヲタなんかとは付き合わないわ。
.     |   \ ∠イ  ,イイ|    ,`-' |      頭が良くて数学が出来てかっこいい人。それが必要条件よ。
     |     l^,人|  ` `-'     ゝ  |        さらに Ann.of Math に論文書けば十分条件にもなるわよ。
      |      ` -'\       ー'  人          一番嫌いなのは論文数を増やすためにくだらない論文を書いて
    |        /(l     __/  ヽ、           良い論文の出版を遅らせるお馬鹿な人。
     |       (:::::`‐-、__  |::::`、     ヒニニヽ、         あなたの論文が Ann of Math に accept される確率は?
    |      / `‐-、::::::::::`‐-、::::\   /,ニニ、\            それとも最近は Inv. Math. の方が上かしら?
   |      |::::::::::::::::::|` -、:::::::,ヘ ̄|'、  ヒニ二、 \
.   |      /::::::::::::::::::|::::::::\/:::O`、::\   | '、   \
   |      /:::::::::::::::::::/:::::::::::::::::::::::::::::'、::::\ノ  ヽ、  |
  |      |:::::/:::::::::/:::::::::::::::::::::::::::::::::::'、',::::'、  /:\__/‐、
  |      |/:::::::::::/::::::::::::::::::::::::::::::::::O::| '、::| く::::::::::::: ̄|
   |     /_..-'´ ̄`ー-、:::::::::::::::::::::::::::::::::::|/:/`‐'::\;;;;;;;_|
   |    |/::::::::::::::::::::::\:::::::::::::::::::::::::::::|::/::::|::::/:::::::::::/
    |   /:::::::::::::::::::::::::::::::::|:::::::::::::::::::::O::|::|::::::|:::::::::::::::/
882132人目の素数さん:2012/06/26(火) 20:55:26.08
素体について質問です。
Fp={0、1、2、・・・p-1}とするとき(0、1、2・・・はpを法とする剰余類と思ってください)、
F8の素体はF2になると本に書いてあるのですが、
F2={0、1}となり1+1=2となりF2について閉じていないので、体になってない気がします。
なぜF2は体になると言えるのでしょうか。
883132人目の素数さん:2012/06/26(火) 21:01:35.51
          __ノ)-'´ ̄ ̄`ー- 、_
        , '´  _. -‐'''"二ニニ=-`ヽ、
      /   /:::::; -‐''"        `ーノ
     /   /:::::/           \
     /    /::::::/          | | |  |
     |   |:::::/ /     |  | | | |  |
      |   |::/ / / |  | ||  | | ,ハ .| ,ハ|
      |   |/ / / /| ,ハノ| /|ノレ,ニ|ル' 
     |   |  | / / レ',二、レ′ ,ィイ|゙/   私は只の数ヲタなんかとは付き合わないわ。
.     |   \ ∠イ  ,イイ|    ,`-' |      頭が良くて数学が出来てかっこいい人。それが必要条件よ。
     |     l^,人|  ` `-'     ゝ  |        さらに Ann.of Math に論文書けば十分条件にもなるわよ。
      |      ` -'\       ー'  人          一番嫌いなのは論文数を増やすためにくだらない論文を書いて
    |        /(l     __/  ヽ、           良い論文の出版を遅らせるお馬鹿な人。
     |       (:::::`‐-、__  |::::`、     ヒニニヽ、         あなたの論文が Ann of Math に accept される確率は?
    |      / `‐-、::::::::::`‐-、::::\   /,ニニ、\            それとも最近は Inv. Math. の方が上かしら?
   |      |::::::::::::::::::|` -、:::::::,ヘ ̄|'、  ヒニ二、 \
.   |      /::::::::::::::::::|::::::::\/:::O`、::\   | '、   \
   |      /:::::::::::::::::::/:::::::::::::::::::::::::::::'、::::\ノ  ヽ、  |
  |      |:::::/:::::::::/:::::::::::::::::::::::::::::::::::'、',::::'、  /:\__/‐、
  |      |/:::::::::::/::::::::::::::::::::::::::::::::::O::| '、::| く::::::::::::: ̄|
   |     /_..-'´ ̄`ー-、:::::::::::::::::::::::::::::::::::|/:/`‐'::\;;;;;;;_|
   |    |/::::::::::::::::::::::\:::::::::::::::::::::::::::::|::/::::|::::/:::::::::::/
    |   /:::::::::::::::::::::::::::::::::|:::::::::::::::::::::O::|::|::::::|:::::::::::::::/
884882:2012/06/26(火) 21:05:01.11
すみません。なんか勘違いしてました
スルーしてください
885132人目の素数さん:2012/06/26(火) 21:10:10.63
          __ノ)-'´ ̄ ̄`ー- 、_
        , '´  _. -‐'''"二ニニ=-`ヽ、
      /   /:::::; -‐''"        `ーノ
     /   /:::::/           \
     /    /::::::/          | | |  |
     |   |:::::/ /     |  | | | |  |
      |   |::/ / / |  | ||  | | ,ハ .| ,ハ|
      |   |/ / / /| ,ハノ| /|ノレ,ニ|ル' 
     |   |  | / / レ',二、レ′ ,ィイ|゙/   私は只の数ヲタなんかとは付き合わないわ。
.     |   \ ∠イ  ,イイ|    ,`-' |      頭が良くて数学が出来てかっこいい人。それが必要条件よ。
     |     l^,人|  ` `-'     ゝ  |        さらに Ann.of Math に論文書けば十分条件にもなるわよ。
      |      ` -'\       ー'  人          一番嫌いなのは論文数を増やすためにくだらない論文を書いて
    |        /(l     __/  ヽ、           良い論文の出版を遅らせるお馬鹿な人。
     |       (:::::`‐-、__  |::::`、     ヒニニヽ、         あなたの論文が Ann of Math に accept される確率は?
    |      / `‐-、::::::::::`‐-、::::\   /,ニニ、\            それとも最近は Inv. Math. の方が上かしら?
   |      |::::::::::::::::::|` -、:::::::,ヘ ̄|'、  ヒニ二、 \
.   |      /::::::::::::::::::|::::::::\/:::O`、::\   | '、   \
   |      /:::::::::::::::::::/:::::::::::::::::::::::::::::'、::::\ノ  ヽ、  |
  |      |:::::/:::::::::/:::::::::::::::::::::::::::::::::::'、',::::'、  /:\__/‐、
  |      |/:::::::::::/::::::::::::::::::::::::::::::::::O::| '、::| く::::::::::::: ̄|
   |     /_..-'´ ̄`ー-、:::::::::::::::::::::::::::::::::::|/:/`‐'::\;;;;;;;_|
   |    |/::::::::::::::::::::::\:::::::::::::::::::::::::::::|::/::::|::::/:::::::::::/
    |   /:::::::::::::::::::::::::::::::::|:::::::::::::::::::::O::|::|::::::|:::::::::::::::/
886132人目の素数さん:2012/06/26(火) 21:21:48.47
いや、やっぱり勘違いじゃないかも…
F8上の部分体で最小なものを考えているんだから
F2の元は8を法とする剰余類のうち、代表元が0と1のもので、
1+1は8を法とする剰余類うちの代表元が2のものだから、
F2は加法について閉じてなくありませんか?
887132人目の素数さん:2012/06/26(火) 21:23:21.19
          __ノ)-'´ ̄ ̄`ー- 、_
        , '´  _. -‐'''"二ニニ=-`ヽ、
      /   /:::::; -‐''"        `ーノ
     /   /:::::/           \
     /    /::::::/          | | |  |
     |   |:::::/ /     |  | | | |  |
      |   |::/ / / |  | ||  | | ,ハ .| ,ハ|
      |   |/ / / /| ,ハノ| /|ノレ,ニ|ル' 
     |   |  | / / レ',二、レ′ ,ィイ|゙/   私は只の数ヲタなんかとは付き合わないわ。
.     |   \ ∠イ  ,イイ|    ,`-' |      頭が良くて数学が出来てかっこいい人。それが必要条件よ。
     |     l^,人|  ` `-'     ゝ  |        さらに Ann.of Math に論文書けば十分条件にもなるわよ。
      |      ` -'\       ー'  人          一番嫌いなのは論文数を増やすためにくだらない論文を書いて
    |        /(l     __/  ヽ、           良い論文の出版を遅らせるお馬鹿な人。
     |       (:::::`‐-、__  |::::`、     ヒニニヽ、         あなたの論文が Ann of Math に accept される確率は?
    |      / `‐-、::::::::::`‐-、::::\   /,ニニ、\            それとも最近は Inv. Math. の方が上かしら?
   |      |::::::::::::::::::|` -、:::::::,ヘ ̄|'、  ヒニ二、 \
.   |      /::::::::::::::::::|::::::::\/:::O`、::\   | '、   \
   |      /:::::::::::::::::::/:::::::::::::::::::::::::::::'、::::\ノ  ヽ、  |
  |      |:::::/:::::::::/:::::::::::::::::::::::::::::::::::'、',::::'、  /:\__/‐、
  |      |/:::::::::::/::::::::::::::::::::::::::::::::::O::| '、::| く::::::::::::: ̄|
   |     /_..-'´ ̄`ー-、:::::::::::::::::::::::::::::::::::|/:/`‐'::\;;;;;;;_|
   |    |/::::::::::::::::::::::\:::::::::::::::::::::::::::::|::/::::|::::/:::::::::::/
    |   /:::::::::::::::::::::::::::::::::|:::::::::::::::::::::O::|::|::::::|:::::::::::::::/
888132人目の素数さん:2012/06/26(火) 21:23:59.46
>>886
1+1=0(mod2)じゃねーの
889132人目の素数さん:2012/06/26(火) 21:27:02.03
F8は体じゃない。
890132人目の素数さん:2012/06/26(火) 21:31:17.45
F8はZ/8Zじゃねーよカス
891132人目の素数さん:2012/06/26(火) 21:33:23.85
>>886
F8って、F2の3次拡大であって、{1,2,3,・・・,7}じゃないからね。
892132人目の素数さん:2012/06/26(火) 21:34:12.41
>>891
{0,1,2,3,・・・,7}
893132人目の素数さん:2012/06/26(火) 21:35:25.22
>>888
8を法とする集合の素体を考えているのに、
2を法とする剰余類を考えていいんですか?

>>889
F8が体でないのはわかるのですが、
例.F8の素体はF2であり〜 と書かれます
894132人目の素数さん:2012/06/26(火) 21:36:39.48
>>891
違うんですか!?
895132人目の素数さん:2012/06/26(火) 21:38:41.27
Fpがpを法とする云々はpが素数のときだけ
896132人目の素数さん:2012/06/26(火) 21:48:52.51
>>894
F8(あるいはGF(8))と書いたら、要素の数が8個の体を指すのが普通の約束。
Z8で、mod8の剰余系Z/8Zを指す。こちらは{0.1.2.・・・,7}を代表元とする剰余類のなす環。
897132人目の素数さん:2012/06/26(火) 21:53:03.78
みなさんありがとうございます!
もう一度考えてきます
898132人目の素数さん:2012/06/26(火) 22:05:37.39
          __ノ)-'´ ̄ ̄`ー- 、_
        , '´  _. -‐'''"二ニニ=-`ヽ、
      /   /:::::; -‐''"        `ーノ
     /   /:::::/           \
     /    /::::::/          | | |  |
     |   |:::::/ /     |  | | | |  |
      |   |::/ / / |  | ||  | | ,ハ .| ,ハ|
      |   |/ / / /| ,ハノ| /|ノレ,ニ|ル' 
     |   |  | / / レ',二、レ′ ,ィイ|゙/   私は只の数ヲタなんかとは付き合わないわ。
.     |   \ ∠イ  ,イイ|    ,`-' |      頭が良くて数学が出来てかっこいい人。それが必要条件よ。
     |     l^,人|  ` `-'     ゝ  |        さらに Ann.of Math に論文書けば十分条件にもなるわよ。
      |      ` -'\       ー'  人          一番嫌いなのは論文数を増やすためにくだらない論文を書いて
    |        /(l     __/  ヽ、           良い論文の出版を遅らせるお馬鹿な人。
     |       (:::::`‐-、__  |::::`、     ヒニニヽ、         あなたの論文が Ann of Math に accept される確率は?
    |      / `‐-、::::::::::`‐-、::::\   /,ニニ、\            それとも最近は Inv. Math. の方が上かしら?
   |      |::::::::::::::::::|` -、:::::::,ヘ ̄|'、  ヒニ二、 \
.   |      /::::::::::::::::::|::::::::\/:::O`、::\   | '、   \
   |      /:::::::::::::::::::/:::::::::::::::::::::::::::::'、::::\ノ  ヽ、  |
  |      |:::::/:::::::::/:::::::::::::::::::::::::::::::::::'、',::::'、  /:\__/‐、
  |      |/:::::::::::/::::::::::::::::::::::::::::::::::O::| '、::| く::::::::::::: ̄|
   |     /_..-'´ ̄`ー-、:::::::::::::::::::::::::::::::::::|/:/`‐'::\;;;;;;;_|
   |    |/::::::::::::::::::::::\:::::::::::::::::::::::::::::|::/::::|::::/:::::::::::/
    |   /:::::::::::::::::::::::::::::::::|:::::::::::::::::::::O::|::|::::::|:::::::::::::::/
899132人目の素数さん:2012/06/27(水) 00:51:30.06
m個の数列、というものを考えるにあたって、
各々の数列を{x[1][n]}, {x[2][n]}, ..., {x[m][n]}というように表したいのですが、
これを実際に紙に書くときはどのように書けばよいでしょうか?
例えば、{x[1][n]}なら単にxの右下に1を書き、さらにその右下にnを書けば良いでしょうか?
それとももっと普通な書き方がありますか?
900132人目の素数さん:2012/06/27(水) 01:03:15.01
2重数列でいいんでねえの。
901132人目の素数さん:2012/06/27(水) 01:08:46.29
一方のindexを特別に注目させたいのなら
 
(x^j)_i 上付きのjと下付きのi、1≦j≦m、 1≦i≦n

x^j をベキと間違えないようにx^<j> と書いたりね。
902132人目の素数さん:2012/06/27(水) 04:59:59.57
>>899
行列と同じように
903132人目の素数さん:2012/06/27(水) 05:18:29.66
>>899
x^n_1, x^n_2, ...

のように上付きと下付きで区別することもあるし、

x_{n1}, x_{n2}, ...

のように添字を並べることもある。
904132人目の素数さん:2012/06/27(水) 21:33:59.99
因数分解なんですが

x^2-2(y-3)x

(x-y-3)^2+(y-3)^2

に変形できるのに、どうやって気づけばいいのか分かりません。
教えていただけると嬉しいです。
905132人目の素数さん:2012/06/27(水) 21:37:35.43
>>904
間違ってる
906132人目の素数さん:2012/06/27(水) 21:38:18.20
>>904
気がつかない
907132人目の素数さん:2012/06/27(水) 21:38:18.55
二つの式の少なくとも一方は写し間違えているな。
908132人目の素数さん:2012/06/27(水) 21:39:00.14
>>904
それは因数分解じゃなくて平方完成。
909132人目の素数さん:2012/06/27(水) 21:51:03.16
>>904
A^2+2BA=(A+B)^2-B^2
910904:2012/06/27(水) 23:02:29.75
レスありがとうございます
すみません間違ってました・・

x^2-2(y-3)x

(x-y+3)^2+(y-3)^2

です。

気づくためのコツみたいなものはあるのでしょうか??
911132人目の素数さん:2012/06/27(水) 23:08:55.81
>>910
間違ってる。それぞれ展開してみ
912132人目の素数さん:2012/06/27(水) 23:11:40.73
>>910
気づかない、2nd
913132人目の素数さん:2012/06/27(水) 23:22:35.63
次の数列に現れるすべての数は合成数であることを示しなさい。
10001、100010001、1000100010001、…
914132人目の素数さん:2012/06/28(木) 00:02:54.15
何度もすいません

(x-y+3)^2-(y-3)^2

ですね。。
お恥ずかしいです。

>>908
因数の積の形にはなっていないので因数分解ではないのですね・・
ご指摘ありがとうございます。

>>909
なるほど!納得できました。
こういう公式とは違うけれどよく使うパターンみたいのがたくさんありそうですね・・
ありがとうございました。
915132人目の素数さん:2012/06/28(木) 00:17:20.01
tan(πx/2) 微分がわからないです
916132人目の素数さん:2012/06/28(木) 00:19:14.04
>>915
合成関数の微分公式
917132人目の素数さん:2012/06/28(木) 01:00:01.22
          __ノ)-'´ ̄ ̄`ー- 、_
        , '´  _. -‐'''"二ニニ=-`ヽ、
      /   /:::::; -‐''"        `ーノ
     /   /:::::/           \
     /    /::::::/          | | |  |
     |   |:::::/ /     |  | | | |  |
      |   |::/ / / |  | ||  | | ,ハ .| ,ハ|
      |   |/ / / /| ,ハノ| /|ノレ,ニ|ル' 
     |   |  | / / レ',二、レ′ ,ィイ|゙/   私は只の数ヲタなんかとは付き合わないわ。
.     |   \ ∠イ  ,イイ|    ,`-' |      頭が良くて数学が出来てかっこいい人。それが必要条件よ。
     |     l^,人|  ` `-'     ゝ  |        さらに Ann.of Math に論文書けば十分条件にもなるわよ。
      |      ` -'\       ー'  人          一番嫌いなのは論文数を増やすためにくだらない論文を書いて
    |        /(l     __/  ヽ、           良い論文の出版を遅らせるお馬鹿な人。
     |       (:::::`‐-、__  |::::`、     ヒニニヽ、         あなたの論文が Ann of Math に accept される確率は?
    |      / `‐-、::::::::::`‐-、::::\   /,ニニ、\            それとも最近は Inv. Math. の方が上かしら?
   |      |::::::::::::::::::|` -、:::::::,ヘ ̄|'、  ヒニ二、 \
.   |      /::::::::::::::::::|::::::::\/:::O`、::\   | '、   \
   |      /:::::::::::::::::::/:::::::::::::::::::::::::::::'、::::\ノ  ヽ、  |
  |      |:::::/:::::::::/:::::::::::::::::::::::::::::::::::'、',::::'、  /:\__/‐、
  |      |/:::::::::::/::::::::::::::::::::::::::::::::::O::| '、::| く::::::::::::: ̄|
   |     /_..-'´ ̄`ー-、:::::::::::::::::::::::::::::::::::|/:/`‐'::\;;;;;;;_|
   |    |/::::::::::::::::::::::\:::::::::::::::::::::::::::::|::/::::|::::/:::::::::::/
    |   /:::::::::::::::::::::::::::::::::|:::::::::::::::::::::O::|::|::::::|:::::::::::::::/
918132人目の素数さん:2012/06/28(木) 02:41:07.13
>>913

x=10^4 とおくと、与式は
 Σ[k=0, n-1] x^k = (x^n - 1)/(x-1),

n=pq(合成数)のとき
 Σ[k=0,pq-1] x^k = {x^(pq) - 1}/(x-1)
 = {x^(pq) -1}/(x^p -1) * (x^p -1)/(x-1)
となるから、nが素数の場合を考示せば十分。

n=2 のとき
 10001 = 73×173,

n=3
 100010001 = 3*7*13*37*9901

n=5 のとき
 10001000100010001 = 41×271×3541×9091×27961
919132人目の素数さん:2012/06/28(木) 23:21:21.60
>>913
(i)p=2のとき
10001=73*173より、合成数。
(ii)pが奇素数のとき
((10^4)^p-1)/(10^4-1)
=(10^p-1)/9*(10^p+1)/11*(100^p+1)/101
(10^p-1)/9、(10^p+1)/11、(100^p+1)/101は明らかに1より大。
さらに、整数kに対してx^k-1がx-1で割り切れ、kが奇数のときx^k+1がx+1で割り切れることを考えると、
これらは全て整数。よって、少なくとも3つの素因数をもつ。
920132人目の素数さん:2012/06/28(木) 23:48:07.83
>>919

(i) n=2 のとき
 10001 = 73×137,

(ii) nが奇数のとき

 Fn(x) = Σ[k=0, n-1] x^k = (x^n - 1)/(x-1),

 Fn(x^2) = Fn(x)Fn(-x),

 Fn(x^4) = Fn(x)Fn(-x)Fn(-x^2)

 Fn(x^8) = ・・・・
921132人目の素数さん:2012/06/29(金) 18:28:59.83
可換環Rにおいて、二つのイデアルI,Jがあり、
どの素イデアルもこの二つのどちらか一方のみを含むとする。
すると、Iの元とJの元の積ijに対し(ij)^N=0となるNが常にある。
これは真でしょう偽でしょうか?
922132人目の素数さん:2012/06/29(金) 19:25:11.95
真か偽です
923132人目の素数さん:2012/06/29(金) 20:37:28.76
921です。自己解決しました。
924132人目の素数さん:2012/06/29(金) 20:55:50.50
          __ノ)-'´ ̄ ̄`ー- 、_
        , '´  _. -‐'''"二ニニ=-`ヽ、
      /   /:::::; -‐''"        `ーノ
     /   /:::::/           \
     /    /::::::/          | | |  |
     |   |:::::/ /     |  | | | |  |
      |   |::/ / / |  | ||  | | ,ハ .| ,ハ|
      |   |/ / / /| ,ハノ| /|ノレ,ニ|ル' 
     |   |  | / / レ',二、レ′ ,ィイ|゙/   私は只の数ヲタなんかとは付き合わないわ。
.     |   \ ∠イ  ,イイ|    ,`-' |      頭が良くて数学が出来てかっこいい人。それが必要条件よ。
     |     l^,人|  ` `-'     ゝ  |        さらに Ann.of Math に論文書けば十分条件にもなるわよ。
      |      ` -'\       ー'  人          一番嫌いなのは論文数を増やすためにくだらない論文を書いて
    |        /(l     __/  ヽ、           良い論文の出版を遅らせるお馬鹿な人。
     |       (:::::`‐-、__  |::::`、     ヒニニヽ、         あなたの論文が Ann of Math に accept される確率は?
    |      / `‐-、::::::::::`‐-、::::\   /,ニニ、\            それとも最近は Inv. Math. の方が上かしら?
   |      |::::::::::::::::::|` -、:::::::,ヘ ̄|'、  ヒニ二、 \
.   |      /::::::::::::::::::|::::::::\/:::O`、::\   | '、   \
   |      /:::::::::::::::::::/:::::::::::::::::::::::::::::'、::::\ノ  ヽ、  |
  |      |:::::/:::::::::/:::::::::::::::::::::::::::::::::::'、',::::'、  /:\__/‐、
  |      |/:::::::::::/::::::::::::::::::::::::::::::::::O::| '、::| く::::::::::::: ̄|
   |     /_..-'´ ̄`ー-、:::::::::::::::::::::::::::::::::::|/:/`‐'::\;;;;;;;_|
   |    |/::::::::::::::::::::::\:::::::::::::::::::::::::::::|::/::::|::::/:::::::::::/
    |   /:::::::::::::::::::::::::::::::::|:::::::::::::::::::::O::|::|::::::|:::::::::::::::/
925132人目の素数さん:2012/06/29(金) 22:09:29.84
>>918-920

n>2 のとき、 Fn(x^2) = Σ[k=0,n-1] (x^2)^k は合成数であることを示しなさい。
926132人目の素数さん:2012/06/30(土) 05:40:08.33
重積分の極座標に変換をおこなう問題について
___________________________________

問題:重積分について以下の設問に答えなさい

I=∬[D]√(1-x^2-y^2)dxdy
D={(x,y)|(x^2+y^2)^2≦y^2-x^2,y≧0}

(1)極座標に変換してDを図示しなさい。
(2)Iで示される積分領域の立体の外形を図示しなさい。
(3)Iを極座標で書きなさい。
(4)Iを求めなさい。

___________________________________

以上が問題になります。

参考までに自分で途中まで考えてみたものです。煩雑ですいませんが、ご指摘おねがいします。

(1)x=rcosθ,y=rsinθとおき、
積分範囲は-√(sinθ^2-cosθ^2)≦r≦√(sinθ^2-cosθ^2)と、0≦θ≦πとなりました。
ですが、以上の図での表し方がわかりません。
(2)立体の外形を図示というのがイマイチピンときません。z=√(1-x^2-y^2)とおいて、0-xyz座標系で考えればいいのでしょうか?
(3) (1)の解が正しければ、∫[0,π]∫[0,√(sinθ^2-cosθ^2)]√(1-r^2)drdθとなると思います。
(4) (3)の解を解いていきますが、形が複雑になり途中で挫折しました。なのでおそらく(1)から、間違えてると思います。


問題には解答がありませんので、できるだけ詳細な回答をしていただければ助かります。
927132人目の素数さん:2012/06/30(土) 06:37:45.66
>>926
極座標では r ≧ 0 としてよい
その置き換えを D の不等式に代入整理すれば
  r^2 ≦ -cos(2θ)
となるはず(レムニスケートという有名な曲線である)
右辺も非負であることから θ の範囲がわかる
I は xy 平面の領域 D と 原点中心半径1の半球面とで挟まれた部分の体積
計算の仕方は教科書に出ているはず
928132人目の素数さん:2012/07/05(木) 18:38:13.63
x = cos 8π/6
のxを求める問題の解き方(考え方)が良く分かりません。
929132人目の素数さん:2012/07/05(木) 19:21:58.02
>>928
単位円の図を書いて考える
930132人目の素数さん:2012/07/05(木) 19:34:37.80
>>929
単位円を使わないとこの手の問題は解けないですよね・・・
931132人目の素数さん:2012/07/05(木) 20:09:50.30
「この手の問題」という程の問題か?
932132人目の素数さん:2012/07/05(木) 20:15:58.93
そんなことはない。
933132人目の素数さん:2012/07/07(土) 23:24:58.99
A1 10  A2 5 のとき、A1-A2は5で、移動範囲Rsというらしいのですが、
式で書くと
Xi-1-Xiと書かれてるんですが、
-1ってのがよくわからないのですが、わかりやすくせつめいおねがいします。
934132人目の素数さん:2012/07/07(土) 23:29:57.35
>>933
自分で読んで文章が分かるのか?
935132人目の素数さん:2012/07/08(日) 00:16:21.31
おそらくx[i-1]-x[i] ([ ]内は右下に小さく書いてある)のことを言っているんだろう
右下に書いてある数は「添字」と呼ばれるもので、その数値が何番目のデータかを示すのに使われる
だから、上の式でx[i]というのは「i番目のデータ」、x[i-1]というのは「i-1番目のデータ」を表している
つまり上の式は「i-1番目のデータからi番目のデータを引く」という意味になる
i-1番目という表現が理解しにくければiに具体的な数を当てはめてみるといい
i=2のときは「1番目のデータから2番目のデータを引く」
i=3のときは「2番目のデータから3番目のデータを引く」
i=4のときは「3番目のデータから4番目のデータを引く」

これらをまとめて示すとx[i-1]-x[i]という式になる
高校数学に「数列」という分野があり、そこで漸化式というものを扱うときにこういう記法が頻出する
だから、一回数列を学んでからだとスッキリ分かるようになると思う

すまん、エスパー検定準2級の俺にはこれが限界だ
936132人目の素数さん:2012/07/08(日) 00:49:56.35
エスパーすごい!
937132人目の素数さん:2012/07/08(日) 00:57:55.06
エクセルくさいな
938132人目の素数さん:2012/07/08(日) 01:01:35.84
Mn(C)の極小左イデアルの求め方を教えてください。
939132人目の素数さん:2012/07/08(日) 01:12:47.64
>935
まじすごいです。
完全に理解しました。実は他の本なんかでは式の書き方が
Xi=1 - Xi
というように、「=」になってたんです。
「=」を→「-」と書くこともあるんですね。
更に別な本では933の式が「-」でなく「+」になってるんで混乱してました。
本がまちがってるんですね。
940132人目の素数さん:2012/07/08(日) 03:53:47.21
根本的に変
941132人目の素数さん:2012/07/09(月) 20:11:20.34
関数系:1,cos(πx),sin(πx),cos(2πx),sin(2πx),cos(3πx),…
を用いて関数f(x)=|x|(xは実数)を[-1,1]上でフーリエ展開せよ、という問題において

f(x)=Σ[n=0〜n=∞]a[n]cos(nπx)+Σ[n=0〜n=∞]b[n]sin(nπx)
で答えは
a[0]=1,a[n]=2{-1+(-1)^n}/{(nπ)^2},b[n]=0となって
f(x)=1-{4/(π^2)}{cos(πx)+cos(3πx)/9+cos(5πx)/25+…)
と考えたのですが、解答によると
f(x)=1/2-{4/(π^2)}{cos(πx)+cos(3πx)/9+cos(5πx)/25+…)でした

どなたか何処が違うのかご指摘願えないでしょうか
942132人目の素数さん:2012/07/09(月) 20:21:27.56
60C30←答えは?
943132人目の素数さん:2012/07/09(月) 20:24:57.94
>>941
∫[-1,1]f(x)dx=?
944132人目の素数さん:2012/07/09(月) 20:32:57.21
>>943
a[0]=∫[-1,1]f(x)*cos(0)dx=∫[-1,1]|x|dx=2∫[0,1]xdx=1
だと思ったのですが、違うでしょうか…?
解答を見ると
2a[0]=∫[-1,1]f(x)*cos(0)dx=∫[-1,1]|x|dx=2∫[0,1]xdx=1
となっていまして…
このa[0]の係数"2"がどこから出てきたのかが分からないという感じです
945132人目の素数さん:2012/07/09(月) 20:36:30.29
>>944
∫[-1,1]1dx=?
946132人目の素数さん:2012/07/09(月) 20:41:58.41
>>945
! 成る程、問題の関数系が正規直交関数系でないからですね
ヒントを下さり有難う御座いました
947132人目の素数さん:2012/07/10(火) 00:33:52.38
定義がわかりませーんてか
948132人目の素数さん:2012/07/10(火) 12:33:01.59
pは素数でa,bは0でない。
a=Π[i=1;n]pie_i、b=Π[i=1;n]pif_iとなる整数ei≧0、fi≧0を取る時
@a|b⇒ei≦fi
Ab|a⇒⇒ei≧fi
を示せという問題を教えてください。
949132人目の素数さん:2012/07/10(火) 16:43:53.03
どっかの再掲か
950132人目の素数さん:2012/07/12(木) 09:18:00.86
f(x)を微分した時に、
f'(x) = a*f(x)+b (a,bは0以外の実数)
という形になる場合、
eを底とした指数関数以外で、これに当てはまるf(x)の例ってありますか?
951132人目の素数さん:2012/07/12(木) 10:14:21.23
>>950
意味不明
952132人目の素数さん:2012/07/12(木) 12:52:31.47
>>950
定数関数 f(x)=-b/a
953132人目の素数さん:2012/07/12(木) 20:02:34.22
おみごと
954132人目の素数さん:2012/07/13(金) 10:27:13.67
∫[o -> π] dx (xsinx )/ (1+cos^2(x))

高校範囲で解けると聞いたんだが解けない。
力を貸してくれ。
955132人目の素数さん:2012/07/13(金) 11:01:50.50
まるちすんなよ
956132人目の素数さん:2012/07/13(金) 15:52:23.09
質問っす

30%の確率で当たるくじが12個あって、それぞれのくじから一回ずつ引いた時に
6個のくじが当たる確率ってどのくらい?
また、仮にくじが12個から13,14と増えていった時に
同じようにそれぞれのくじから一回ずつ引いた時に6個のくじが当たる確率はどれくらいずつが上がっていくの?

大体でいいので教えていただけると幸いです
957132人目の素数さん:2012/07/13(金) 16:05:18.93
>>956
7.9%、10.3%、12.6%
958132人目の素数さん:2012/07/14(土) 05:31:38.15
>>954
 ( ゚∀゚)つ

f(π-x) = f(x) のとき、

 I = ∫[0,π] x f(x)dx = ∫[0,π] (π-y) f(y)dy,

 I = (π/2)∫[0,π] f(x)dx = π∫[0,π/2] f(x)dx,
959132人目の素数さん:2012/07/14(土) 23:40:42.83
>>954

 ∫ sin(x)/{1+cos(x)^2} dx = -∫ 1/(1+z^2) dz
   = - arctan(z)
   = - arctan(cos(x)),
960132人目の素数さん:2012/07/16(月) 12:46:07.18
http://squid.axfc.net/uploader/Img/l/3849849043/v/Img_146146.jpg

とあるゲームでの検証結果です。
威力、攻撃補正、防御補正、ダメージの関係式分かればお願いします。


追記:ダメージは下1けたに乱数があるようで、最悪繰り上げ等して
導けるのであればそれでも構いません。
961132人目の素数さん:2012/07/16(月) 15:49:59.05
>>960
Not Found

The requested URL /uploader/Img/l/3849849043/v/Img_146146.jpg was not found on this server.
だと
962馬鹿を焼く描写 ◆ghclfYsc82 :2012/07/16(月) 20:03:30.55
勉強や努力が足りなくて優秀になれない奴が惨めな思いをするのは当然
なんだよ。それを自分で何もせずに優秀な人間の足を引っ張るとは言語
道断である。他人を貶めるだけで自分は楽をする奴は恥を知れ。今後も
そういう馬鹿者を発見次第、即刻攻撃を掛けて当該スレを焼け野が原に
するので、覚悟をする様に願いたい。こういう考え方が国家を滅ぼす。
無能な馬鹿は自滅するに任せ、優秀な人材こそを選択的に抽出し、それ
を国家が意図して保護しなければならない。そうする事が国家が生き残
る唯一の道である。繰り返す。何の努力もしない馬鹿を無条件に保護す
れば、その結果として誰も努力しなくなるだけである。だから馬鹿を保
護しては絶対にならない。



>みんなで優秀な人間の足を引っ張って沈もうよ。
>そうすれば自分だけが馬鹿で惨めな思いをしなくて
>すむから楽チン。
>一億総白痴可で横並びになれば怖くは無い
>
963960:2012/07/16(月) 22:40:25.40
>>961
今気づいた・・・

一応上げなおしました。
http://www.uproda.net/down/uproda495153.jpg
964132人目の素数さん:2012/07/17(火) 02:45:01.00
>>926
今気づいた・・・

(3)では、dx dy → r dr dθ のはず。
  つまり、変換のヤコビアン |J| = r,
 √(1-r^2) = z とおけば
 ∫[(√2)cosθ, 1] z^2・dz = [ (1/3)z^3 ](z=(√2)cosθ, 1)
    = (1/3){1 - (2√2)(cosθ)^3}
    = (1/3){1 - (1/√2)[cos(3θ) + 3cosθ]},
これを π/4 < θ < 3π/4 で積分する。
965132人目の素数さん:2012/07/21(土) 09:38:17.90
任意の数直線内がある。
これに含まれる有理数と無理数は、どちらの方が多いか?
966132人目の素数さん:2012/07/21(土) 16:51:14.25
さあ
967132人目の素数さん:2012/07/21(土) 16:54:45.18
>>965
> 数直線内
ふーむ
968132人目の素数さん:2012/07/21(土) 17:00:34.80
>>965
有理数の全体は可算無限個、無理数の全体は非可算無限個なので無理数のが多い
969132人目の素数さん:2012/07/21(土) 19:51:17.28
>>968
横レス失礼 無理数全体の集合の濃度が可算無限でないことはどうやって示す?
970132人目の素数さん:2012/07/21(土) 19:53:36.40
可算濃度+可算濃度=可算濃度<連続体濃度
971132人目の素数さん:2012/07/21(土) 20:20:14.58
>>969
有名な対角線論法ではいかんの?
972132人目の素数さん:2012/07/24(火) 17:48:52.34
age
973132人目の素数さん:2012/07/24(火) 17:56:22.06
>>965
有理数と無理数の数を比べるなんて、
太陽系の広さと宇宙全体の広さを比べることくらいナンセンス
974132人目の素数さん:2012/07/24(火) 17:58:01.17
有理数が無理数の一部だと思ってる人がいるとは驚いた
975132人目の素数さん:2012/07/24(火) 17:59:20.18
yをn次元ベクトルとする。このとき、
「Eがボレル集合ならE+yもボレル集合」はどうやって示せばいいのかわからん。
教えて賢い人(´・ω・`)
976132人目の素数さん:2012/07/24(火) 18:00:18.05
平行移動は連続写像
977132人目の素数さん:2012/07/24(火) 18:12:35.64
>976
ごめん、もうちょっと詳しく(´・ω・`)
978132人目の素数さん:2012/07/24(火) 19:18:37.85
>>975
質問の文を変えたな

教科書嫁^2
979描者の品格 ◆ghclfYsc82 :2012/07/24(火) 19:24:09.16


>367 :匿名希望:2012/07/23(月) 17:23:38.69
> >>365
> 頭悪いのはお前の方だろ?
> 「猫」という字を「描」に間違えやがってwww
> 小学生並みの頭の悪さだぞ、お前!
> 焼かれるのはお前の方だろ?
> 外国では猫の丸焼きというゲテモノ料理もあるらしいぜwww
>
980132人目の素数さん:2012/07/25(水) 00:22:57.56
>>973
オマエの例え話はナンセンス
981描者の品格 ◆ghclfYsc82 :2012/07/25(水) 00:29:14.25


>367 :匿名希望:2012/07/23(月) 17:23:38.69
> >>365
> 頭悪いのはお前の方だろ?
> 「猫」という字を「描」に間違えやがってwww
> 小学生並みの頭の悪さだぞ、お前!
> 焼かれるのはお前の方だろ?
> 外国では猫の丸焼きというゲテモノ料理もあるらしいぜwww
>
982132人目の素数さん:2012/07/25(水) 22:33:37.53
二百七十九日。
983132人目の素数さん:2012/07/25(水) 23:48:42.90
難扇子
984132人目の素数さん:2012/07/26(木) 18:04:05.59
よく「初等的な解法」とか言いますが、初等的ってどのあたりまでを指すんですか?
985132人目の素数さん:2012/07/26(木) 19:33:36.16
>>984
微積分とか線型代数なんじゃねー
986132人目の素数さん:2012/07/26(木) 19:40:17.97
微分方程式って、今もやってんのかな?

一年上の先輩は高校生で習ったらしいが、
俺らは大学から習い始めた
987132人目の素数さん:2012/07/26(木) 22:33:37.25
二百八十日。
988132人目の素数さん:2012/07/26(木) 22:43:06.79
>>986
ゆとりがやってるわけねーだろ
989132人目の素数さん:2012/07/27(金) 00:02:08.95
いまの数3ではやってるよ
数3をやらない学校やコースもあるから
誰もがやるわけじゃないよ
990132人目の素数さん:2012/07/27(金) 00:52:41.06
暑いと変なのが湧くな
991132人目の素数さん:2012/07/27(金) 04:38:36.90
logを使わない関数で、
x→∞でf'(x)が0に収束するのにf(x)が無限大に発散する例ってありますか?
992132人目の素数さん:2012/07/27(金) 04:50:40.13
√x
993132人目の素数さん:2012/07/27(金) 05:01:38.11
おお〜、そうか
0<a<1、
f(x) = x^a
なら全部そうなりそうですね

ありがとうございました
994真描vs偽描 ◆ghclfYsc82 :2012/07/27(金) 15:37:48.30


>462 名前:132人目の素数さん :2012/07/26(木) 23:54:17.40
> >>461
> 専門学校生が
> 「あらやだイケメンに触られて気持ちいい」
> って思ってたら通報されなかっただろうに
> 気持ち悪いおじさんになるために努力を積み重ねてきた結果
> 「キモ顔のおじさんが、気持ち悪く触ってきて超キモい」
> って思わせることに成功し逮捕されたんだよね
> 努力を実らせた立派な人だと思う
>
>
> 努力して痴漢で逮捕される夢を叶えた描者さんはただ者じゃないと思います
> すばらしい
>
995132人目の素数さん:2012/07/27(金) 22:33:37.37
二百八十一日。
996132人目の素数さん:2012/07/28(土) 01:05:34.40
巾乗の定義は exp(a log x) やから、
log 使うとるがな。

997真描vs偽描 ◆ghclfYsc82 :2012/07/28(土) 06:17:46.42


訂正:
懲戒免職 → 懲戒解雇

>懲戒免職になって、ここまで堕ちたか。
>昔から現実を見れていなかったが、さらにひどくなっているようだ。
>現実と願望が乖離して、願望を現実だと思い込んできているね。
>
>勝手なことを言ったり実行したりしているから、助けてもらえずクビになる。
>ほんとに人生大損だね。
>
998132人目の素数さん:2012/07/28(土) 22:33:37.07
二百八十二日。
999132人目の素数さん:2012/07/29(日) 06:23:24.39
Aを環、Rをその部分環とし、mをRの極大イデアルとする。
このとき、mAは、極大環またはAである。

これは正しいでしょうか?
1000132人目の素数さん:2012/07/29(日) 12:11:41.66
極大環なんて知らん
10011001
このスレッドは1000を超えました。
もう書けないので、新しいスレッドを立ててくださいです。。。